Eureka 1998

Page 1

CONTEÚDO APRESENTAÇÃO

2

A NOVA OLIMPÍADA BRASILEIRA DE MATEMÁTICA Introdução

5

OLIMPÍADA BRASILEIRA DE MATEMÁTICA Problemas de treinamento

7

OLIMPÍADA BRASILEIRA DE MATEMÁTICA Problemas das provas das primeiras fases Júnior e Sênior 1997

12

A OLIMPÍADA DE MAIO Introdução

22

III OLIMPÍADA DE MAIO Primeiro nível

23

III OLIMPÍADA DE MAIO Segundo nível

29

9a. OLIMPÍADA DO CONE SUL Introdução

35

8a. OLIMPÍADA DO CONE SUL Problemas

36

ARTIGOS

NÚMEROS MÁGICOS E CONTAS DE DIVIDIR Carlos Gustavo Tamm de Araújo Moreira

38

COMO PERDER AMIGOS E ENGANAR PESSOAS Nicolau C. Saldanha

41

DOIS PROBLEMAS SOBRE GRAFOS Paulo Cezar Pinto Carvalho

51

PROBLEMAS PROPOSTOS

58

AGENDA OLÍMPICA

60

COORDENADORES REGIONAIS

61


Sociedade Brasileira de Matemática

APRESENTAÇÃO EUREKA!, a revista da Olimpíada Brasileira de Matemática faz parte de um grande projeto que tem como objetivo principal contribuir decisivamente para a melhoria de ensino de Matemática em nosso país. O que planejamos realizar é descrito (de forma resumida), nesta apresentação. DOS OBJETIVOS O ensino de Matemática hoje no Brasil difere pouco do ensino praticado há 20 anos. A cada ano, livros novos são editados repetindo quase sempre o mesmo estilo e os mesmos conteúdos dos anteriores. Existem hoje no Brasil bons livros de Matemática dedicados aos alunos tanto do ensino fundamental quanto do ensino médio. Entretanto, o que lhes falta é um ingrediente que, no mundo de hoje, é fundamental: o estímulo à criatividade. Entendemos que não é suficiente para a formação do futuro cidadão um aprendizado burocrático da Matemática e percebemos a importância de estimular os alunos desde tenra idade a resolver problemas novos e desafiantes, propiciando o desenvolvimento da imaginação e da criatividade. O programa de Olimpíadas de Matemática é reconhecido em todos os países do mundo desenvolvido como o mais eficiente instrumento para atingir esse objetivo. Aproveitando o natural gosto dos jovens pelas competições, as Olimpíadas de Matemática têm conseguido estimular alunos a estudar conteúdos além do currículo escolar e, também, por outro lado, aumentar e desenvolver a competência dos professores. DO PROJETO O programa de Olimpíadas de Matemática existe no país há 19 anos. Sempre foi pequeno e dedicado a encontrar jovens talentos para a Matemática ou para ciências afins e, neste aspecto, cumpriu sua finalidade. Temos hoje brilhantes matemáticos e cientistas de renome mundial que tiveram origem nas Olimpíadas de Matemática. Entretanto, reconhecemos que, com esta atividade, pode-se fazer muito mais. Com parceria do IMPA EUREKA! N° 1, 1998

2


Sociedade Brasileira de Matemática

(Instituto de Matemática Pura e Aplicada) e com a SBM (Sociedade Brasileira de Matemática), foi submetido ao CNPq um projeto que pretende contribuir para a melhoria do ensino de Matemática no Brasil utilizando as Olimpíadas de Matemática como mecanismo propagador. Este projeto teve boa acolhida e neste momento estamos iniciando um trabalho de grandes dimensões que, para ter seus objetivos cumpridos, necessitará também (e principalmente) do apoio e da ajuda de diversos segmentos da sociedade: alunos, professores, escolas, universidades, secretarias de educação etc. Nossa atividade estará centrada na resolução de problemas e atingirá alunos desde a 5a. série do ensino fundamental até a 3a. série do ensino médio e, naturalmente, seus professores. Para a divulgação deste material, utilizaremos esta revista, cartazes mensais com diversas informações sobre atividades olímpicas e um site na Internet. Para movimentar os jovens realizar-se-à anualmente uma nova Olimpíada Brasileira de Matemática, que estará dividida em níveis de acordo com a escolaridade do aluno. Além disso, estaremos apoiando a realização de competições de Matemática em nível regional. Para os professores, estão sendo planejados cursos de aperfeiçoamento em diversas regiões do país, também colocaremos à disposição, através do site da Internet, um vasto banco de problemas e uma biblioteca especializada localizados na nossa sede no IMPA. DA REVISTA EUREKA!, a revista da Olimpíada de Matemática é uma publicação dedicada principalmente aos alunos e professores da escola secundária a qual será editada quatro vezes ao ano e terá basicamente a seguinte estrutura: a)

Seção de problemas de treinamento com soluções, dividida, em três níveis: para os alunos de 5a. e 6a. séries, para os alunos de 7a. e 8a. séries e para os alunos de ensino médio. Esta seção pretende fornecer aos alunos material para estudo e pesquisa dirigidos à Olimpíada Brasileira, que será realizada nesses mesmos três níveis.

EUREKA! N° 1, 1998

3


Sociedade Brasileira de Matemática

b)

Seção de artigos de Matemática elementar, tratando de assuntos que complementem o currículo escolar e que também abordem novos conteúdos. Estes artigos estarão classificados em iniciante, intermediário ou avançado, de acordo com o estágio de desenvolvimento dos leitores aos quais se destinem os artigos.

c)

Seção de Problemas de diversos níveis, sem solução, para que os leitores possam pesquisar e enviar suas soluções para a revista, sendo as melhores publicadas nos números seguintes.

d)

Seção de Cartas dos Leitores, em que alunos e professores terão possibilidade de fazer quaisquer perguntas. Todas as cartas serão respondidas e as mais relevantes serão publicadas.

e)

Agenda, para informarmos todas as atividades ligadas às Olimpíadas de Matemática no Brasil e no exterior.

DOS CARTAZES Para que nossa atividade permaneça viva durante o ano, enviaremos todos os meses para as escolas cadastradas um cartaz da Olimpíada Brasileira de Matemática. Esse cartaz conterá todas as informações sobre as atividades olímpicas e também o Problema do Mês, em cada um dos três níveis. Contamos com que muitos alunos fiquem interessados nesse desafio e que nos enviem soluções. Todos os colégios cadastrados receberão gratuitamente a revista EUREKA! e os cartazes mensais. Para cadastrar um colégio, basta entrar em contato conosco dando o nome do colégio, endereço completo e o nome de um professor responsável para receber a correspondência.

Rio de Janeiro, abril de 1998

EUREKA! N° 1, 1998

4


Sociedade Brasileira de Matemática

A NOVA OLIMPÍADA BRASILEIRA DE MATEMÁTICA A Olimpíada Brasileira de Matemática será realizada a partir deste ano de 1998 de forma bastante diferente da que vinha sendo praticada nos últimos anos. Isto porque agora passa a atingir os alunos desde a 5a. série do ensino fundamental. Antes, a Olimpíada Brasileira de Matemática era principalmente um instrumento para detectar talentos e desenvolvê-los, mas, agora, tem também por objetivo promover em âmbito nacional a melhoria do ensino de Matemática nas escolas, com o desenvolvimento conjunto de alunos e professores. A Olimpíada Brasileira de Matemática, a partir deste ano, não será apenas uma competição. Para a preparação dos alunos e para o aperfeiçoamento dos professores, a OBM distribuirá aos colégios revistas e cartazes contendo farto material para estudo e pesquisa, dedicados a cada faixa de escolaridade e desenvolvimento dos alunos. A realização das provas é uma finalização (sempre parcial) dessa atividade. A Olimpíada Brasileira de Matemática será realizada em três fases e em três níveis. São eles: Nível 1 - para alunos da 5ª. e 6ª. séries do ensino fundamental. Nível 2 - para alunos da 7ª. e 8ª. séries do ensino fundamental. Nível 3 - para alunos do ensino médio (antigo 2º. grau). Para cada um dos níveis, a OBM terá três fases. Na primeira, qualquer aluno interessado poderá participar. Para participar das outras, existirá um critério de promoção. -

A prova da primeira fase será de múltipla escolha, contendo de 20 a 25 questões sobre conteúdo adequado a cada um dos níveis de escolaridade. Nestas questões, serão incluídas algumas que dependam de alguma criatividade, porém não fugindo dos conteúdos tradicionais das escolas.

-

A prova da segunda fase será discursiva e constará de 6 problemas, exigindo uma maior dose de iniciativa e criatividade.

-

A prova da terceira fase será também discursiva. As provas da primeira e segunda fases da OBM serão realizadas nas escolas que desejarem participar dessa atividade. A correção

EUREKA! N° 1, 1998

5


Sociedade Brasileira de Matemática

das provas também será realizada nas escolas, com o salutar envolvimento de seus professores, de acordo com critérios determinados pela organização. Os coordenadores oferecerão locais alternativos aos alunos que desejarem participar da Olimpíada, caso o colégio onde realizam seus estudos não venha a organizar a atividade. -

A prova da terceira fase será realizada em um local central designado pelo coordenador local e corrigida pelo comitê organizador da OBM.

Para tornar viável a realização de uma competição de Matemática em âmbito nacional, foi criada uma estrutura operativa. As atividades de elaboração das provas, edição da revista, publicação dos cartazes etc. serão centralizadas na Secretaria da Olimpíada Brasileira de Matemática, localizada no IMPA (Rio de Janeiro). Para apoiar as atividades no país, existem hoje cerca de 30 coordenadores regionais que darão assistência às escolas de sua área de atuação. Cada colégio participante da OBM ficará, portanto, ligado ao coordenador regional mais próximo, que fornecerá toda a assistência necessária. Em 1998, a Olimpíada Brasileira de Matemática será realizada nas seguintes datas: Primeira fase: Segunda fase: Terceira fase:

Sábado, 6 de junho Sábado, 12 de setembro Sábado, 24 de outubro (níveis 1, 2 e 3) e Domingo, 25 de outubro (nível 3)

A Olimpíada Brasileira de Matemática não é de forma alguma uma competição entre colégios. Ela pretende essencialmente despertar nos alunos o gosto pelo estudo da Matemática através da resolução de problemas novos, estimulando o desenvolvimento da imaginação e da criatividade. O aspecto da competição naturalmente existe, mas jamais estará ligado a grupos, equipes, colégios, cidades ou regiões. Desejamos deixar bem claro que uma medalha oferecida pela Olimpíada Brasileira de Matemática é um reconhecimento ao esforço individual do aluno premiado, mas representa também o coroamento de um trabalho em que centenas ou milhares de anônimos alunos também se desenvolveram. E isto, no fundo, é o que importa.

EUREKA! N° 1, 1998

6


Sociedade Brasileira de Matemática

OLIMPÍADA BRASILEIRA DE MATEMÁTICA Problemas de treinamento para a Primeira fase Primeiro nível 1)

Num quadrado formado por 9 quadrados menores e do mesmo tamanho, queremos escrever um X e um O, de forma que eles não fiquem vizinhos, isto é, os quadrados em que se encontram não podem ter um lado ou um vértice comum. O desenho abaixo mostra uma dessas possibilidades:

× De quantas maneiras podemos localizar os dois sinais, respeitadas as condições apresentadas? a) 32 2)

b) 20

c) 64

d) 18

e) 12

Jacira consegue datilografar 20 páginas de um manuscrito em 4 horas e Joana o faz em 5 horas. Ainda restam 900 páginas do manuscrito para datilografar. Se as duas começarem a datilografar no mesmo instante essas páginas, quantas páginas deverá pegar a mais lenta, de forma que ambas terminem juntas? a) 225 b) 500 c) 400 d) 450 e) 180

3)

O professor Epaminondas, no primeiro dia de aula, apostou que, entre os alunos daquela classe, pelo menos dois fariam aniversário no mesmo dia do mês. O professor tinha certeza de que ganharia a aposta, pois naquela classe o número de alunos era maior ou igual a: a) 15

b) 32

c) 28

d) 31

e) 30

EUREKA! N° 1, 1998

7


Sociedade Brasileira de Matemática

4)

Seu Pedro possui três lotes quadrados: um deles tem lado de 10 metros, e os outros dois têm lados de 20 metros cada. Seu Pedro quer trocar os três lotes por um outro lote quadrado, cuja área seja a soma das áreas daqueles três lotes. O novo lote deverá ter lado de medida: a) impossível de obter b) 24 metros d) 40 metros e) 30 metros

c) 25 metros

5)

Um jogo consiste em partir da casa 1 à casa 36 numa trilha com casas numeradas de 1 a 36. Os dois jogadores começam na casa 1 e o avanço de casas depende do lançamento de dois dados cúbicos comuns.

-

Se a soma dos pontos for par, o jogador avança 3 casas. Se a soma dos pontos for ímpar, o jogador avança 1 casa. Se o jogador ultrapassar a última casa, retorna à casa 1. A ordem com que os jogadores iniciam suas jogadas é definida por alguma forma de sorteio. Ganha quem parar primeiro na casa 36. O menor número de jogadas que alguém pode fazer e ganhar é a) 37

b) 13

c) 12

d) 14

e) 17

Segundo nível 1)

A equação do 2°. grau ax2 + bx – 3 = 0 tem –1 como uma de suas raízes. Sabendo que os coeficientes a e b são números primos positivos, podemos afirmar que a2 + b2 é igual a: a) 29

2)

b) 89

c) 17

d) 13

e) 53

Você já conhece o quadrado mágico de ordem 3: a soma dos números das linhas, das colunas e das diagonais é 15. A figura a se-

EUREKA! N° 1, 1998

8


Sociedade Brasileira de Matemática

guir mostra uma das oito possibilidades de escrever os números no quadrado:

O único número que não pode mudar de posição em todos esses quadrados mágicos é: a) 1 3)

d) 7

e) 9

b) 16

c) 8

d) 32

e) 68

Os pontos A, B e C são vértices de um triângulo cujos lados medem 3, 4 e 5 cm e pertencem ao interior de uma circunferência, da qual estão a uma distância de 1 cm. O raio da circunferência, em centímetros, é: a) 5

5)

c) 5

No modo SP, o aparelho de videocassete grava exatamente duas horas e, no modo EP, grava quatro horas de filme, com menor qualidade. Carlinhos quer gravar um filme de 136 minutos, com a melhor qualidade possível. Ele decidiu começar no modo EP e terminar no modo SP. Após quantos minutos de gravação no modo EP ele deve passar ao modo SP ? a) 20

4)

b) 3

b) 7

c) 2,5 d) 4,2 e) 3,5

Um número de dois algarismos não nulos é igual ao dobro do produto desses algarismos. Esse número pertence ao conjunto: a){11,12,..., 30} c){51,52,..., 70} e){91,92,..., 99}

b){31,32,..., 50} d){71,72,..., 90}

EUREKA! N° 1, 1998

9


Sociedade Brasileira de Matemática

Terceiro nível 1)

Considere três circunferências concêntricas ( mesmo centro T ) de raios 1, 2 e 3, respectivamente. Considere um triângulo cujos vértices pertencem, um a cada uma das circunferências. Sabendo que o triângulo tem área máxima sob essas condicões, podemos afirmar que, para este triângulo, o ponto T é o: a) baricentro b) incentro e) ex-incentro

2)

f(x) =

c) circuncentro d) ortocentro

Dada a função f: Z→ Z ( Z é o conjunto dos números inteiros) definida por x – 1 se x é ímpar e x + 1 se x é par, podemos afirmar que o número de soluções da equação f(x) = f(2x) é: a) 1

3)

c) 3

d) 4

e) 0

Na seqüência de inteiros positivos a1, a2,..., ak,…, para 1≤ i ≤ k,o termo ai é o i-ésimo ímpar positivo; para i > k, o termo ai é a média aritmética dos termos anteriores. Podemos concluir que a2k é igual a: a) k2

4)

b) 2

b)k

c)2k

d) 0

e)

k

Os vértices de um triângulo têm coordenadas (0,0), (3,1) e (1,7), respectivamente. As retas que passam pelos vértices e por um ponto T no interior do triângulo dividem-no em 6 triângulos de mesma área. Então: a) T = (3,6) d) T = (2,8)

b) T = (2,4) e) T = (2/3, 4/3)

EUREKA! N° 1, 1998

10

c) T = (4/3, 8/3)


Sociedade Brasileira de Matemática

5)

Para quantos valores reais de p a equação x3 – px2 + px –1 = 0 tem todas as raízes reais e inteiras ? a) 1

6)

b) 2

c) 3

d) 4

e) 5 ou mais

Considere o conjunto P dos pontos (x,y) do R2 tais que x e y sejam inteiros. Por exemplo, (1,1) ∈ P. Tome agora uma circunferência de diâmetro igual a 5, de forma que em seu interior haja o maior número possível de pontos de P. Esse número é: a) 10

b) 16

c) 20

d) 14

e) 21

Nota: Veja as respostas na página 21.

Você sabia… que os antigos egípcios não usavam frações com numerador maior que 1 nem somavam frações iguais de numerador 1 ? Assim por exemplo, eles se referiam ao número 2/5 como 1/3 + 1/15. Veja o problema 9 na página 60.

#OOO EUREKA! N° 1, 1998

11


Sociedade Brasileira de Matemática

OLIMPÍADA BRASILEIRA DE MATEMÁTICA Provas Júnior e Sênior 1997 ♦ Até o ano passado a Olimpíada Brasileira de Matemática era realizada em apenas dois níveis. PRIMEIRA FASE JÚNIOR PROBLEMA 1

O número N tem três algarismos. O produto dos algarismos de N é 126 e a soma dos dois últimos algarismos de N é 11. O algarismo das centenas de N é: a)2

b) 3

c)6

d)7

e)9

PROBLEMA 2

A fortuna de João foi dividida da seguinte forma: um quinto para seu irmão mais velho, um sexto do restante para seu irmão mais novo e partes iguais do restante para cada um de seus 12 filhos. Que fração da fortuna cada filho recebeu? a)

1

b)

20

1 18

c)

1

d)

16

1 15

e)

1 14

PROBLEMA 3

No alvo abaixo, uma certa pontuação é dada para a flecha que cai na região A e outra para a flecha que cai na região B. Alberto lançou 3 flechas: uma caiu em B e duas em A, e obteve 17 pontos. Carlos também lançou 3 flechas: uma caiu em A e duas em B, e obteve 22 pontos. Quantos pontos são atribuídos para uma flecha que cai na região A? a) 2 b) 3 c) 4 d) 5

A B

e) 6

EUREKA! N° 1, 1998

12


Sociedade Brasileira de Matemática

PROBLEMA 4

Seja f uma função definida para todo x real, satisfazendo as condições:

f(3) = 2  f(x + 3) = f(x) ⋅ f(3) Então, f(–3) vale: a)–6

b)0

c)

1 2

d)2

e)–1

PROBLEMA 5

Quatro carros, de cores amarela, verde, azul e preta, estão em fila. Sabe-se que o carro que está imediatamente antes do carro azul é menor do que o que está imediatamente depois do carro azul; que o carro verde é o menor de todos; que o carro verde está depois do carro azul; e que o carro amarelo está depois do preto. O primeiro carro da fila: a)é amarelo. b)é azul. c)é preto. d)é verde. e) não pode ser determinado apenas com esses dados. OBS: O primeiro da fila é o que vem antes de todos os outros.

PROBLEMA 6

64 jogadores de habilidades diferentes disputam um torneio de tênis. Na primeira rodada, são feitos 32 jogos (os emparelhamentos são por sorteio), e os perdedores são eliminados. Na segunda rodada, são feitos 16 jogos, os perdedores são eliminados, e assim por diante. Se os emparelhamentos são feitos por sorteio e não há surpresas ( se A é melhor que B, A vence B), qual o número máximo de jogos que o décimo melhor jogador consegue jogar? a)2

b)3

c)4

d)5

e)6

EUREKA! N° 1, 1998

13


Sociedade Brasileira de Matemática

PROBLEMA 7

O número de pares (x, y) de reais que satisfazem o sistema de equações

 x 2 − xy − y 2 + 1 = 0  3  x − x 2 y − xy 2 + x − y + 2 = 0 a)0

b)1

c)2

d)3

é igual a:

e)4

PROBLEMA 8

Seja y = x + 2 + x − 1 + x – 3 . Se 1 ≤ x < 2, então y é igual a: a)x + 4

b)3x – 2

c)x – 4

d)3x + 2

e)x – 2

PROBLEMA 9

Um gramado tem a forma de um quadrado com 10m de lado. Uma corda tem um dos extremos fixado em um dos vértices, e no outro extremo está amarrado um bode. Se o bode consegue comer metade da grama, então o comprimento da corda é de aproximadamente: a)8m

b)7,5m

c)7m

d)6,5m

e)6m

PROBLEMA 10

Se p e q são inteiros positivos tais que

7 10

pode ter é: a)6

b)7

c)25

d)30

e)60

EUREKA! N° 1, 1998

14

<

p q

<

11 15

, o menor valor que q


Sociedade Brasileira de Matemática

PROBLEMA 11

A equação

x + 10 − 2 x + 3 = 1 − 3 x :

a) não tem solução. b) tem uma única solução positiva. c) tem uma única solução negativa. d) tem duas soluções, uma positiva e outra negativa. e) tem duas soluções, ambas negativas.

PROBLEMA 12

Como o médico me recomendou caminhadas, todo dia de manhã dou uma volta (com velocidade constante) na quadra em que resido. Minha mulher aproveita para correr (com velocidade constante) em volta do quarteirão. Saímos juntos e chegamos juntos. Ela percorre a quadra no mesmo sentido que eu e me ultrapassa duas vezes durante o percurso. Se ela corresse no sentido contrário ao meu, quantas vezes ela cruzaria comigo? a)2

b)3

c)4

d)5

e)6

PROBLEMA 13

Em uma urna há 28 bolas azuis, 20 bolas verdes, 12 bolas amarelas, 10 bolas pretas e 8 bolas brancas. Qual é o número mínimo de bolas que devemos sacar dessa urna para termos certeza de que sacaremos pelo menos 15 bolas da mesma cor? a)58

b)59

c)60

d)71

e)72

PROBLEMA 14

Um ladrilho, em forma de polígono regular, foi retirado do lugar que ocupava em um painel. Observou-se, então, que esse ladrilho, se sofresse uma rotação de 40o ou de 600 em torno de seu centro, poderia ser encaixado perEUREKA! N° 1, 1998

15


Sociedade Brasileira de Matemática

feitamente no lugar que ficou vago no painel. O menor número de lados que pode ter esse ladrilho é: a)6

b)9

c)12

d)15

e)18

PROBLEMA 15

No triângulo retângulo ABC da figura abaixo, está inscrito um quadrado. Se AB = 20 e AC = 5, que porcentagem a área do quadrado representa da área do triângulo ABC? C

a) 25% b) 30% c) 32% d) 36% e) 40%

A

B

PROBLEMA 16

Em certo país, a unidade monetária é o pau. Há notas de 1 pau e moedas de meio pau, um terço de pau, um quarto de pau e um quinto de pau. Qual a maior quantia, em paus, que um cidadão pode ter em moedas sem que possa juntar algumas delas para formar exatamente um pau? a)

11

b) 1

12

5 12

c) 2

7 15

d) 2

13

e) 2

60

43 60

PROBLEMA 17

João e Pedro são vendedores e ganham R$ 1000,00 de salário e comissão de 8% sobre as vendas. Em setembro, João ganhou R$ 2000,00 e Pedro ganhou R$ 2 500,00. Nesse mês, as vendas de Pedro superaram as de João em: a) 20%

b) 25%

c) 30%

d) 40%

EUREKA! N° 1, 1998

16

e) 50%


Sociedade Brasileira de Matemática

PROBLEMA 18

Um triângulo ABC, de lados AB = c, AC = b e BC = a, tem perímetro 2p. Uma circunferência tangencia o lado BC e os prolongamentos dos lados AB e AC nos pontos P, Q e R, respectivamente. O comprimento AR é igual a: a)p − a

b)p − b

c)p − c

d)p

e) 2p

PROBLEMA 19

P é um ponto interior a um quadrado ABCD. As distâncias de P aos vértices A e D e ao lado BC são iguais a 10cm. O lado do quadrado mede: a)10cm

b)12cm

c)14cm

d)16cm

e)18cm

PROBLEMA 20

A figura ao lado mostra três dados iguais. O número da face que é a base inferior da coluna de dados: a) é 1. b) é 2. c) é 4. d) é 6. e) pode ser 1 ou 4. PRIMEIRA FASE SÊNIOR (*) (*) Na prova da primeira fase sênior apareceram os problemas 5, 6, 7, 10, 12, 13, 14, 16, 17, 18, 19 e 20 da primeira fase júnior. PROBLEMA 1

Quantos são os pares não-ordenados de inteiros positivos tais que, em cada par, a soma do produto dos números do par com a soma dos números do par com o módulo da diferença dos números do par seja igual a 20? a)1

b)2

c)3

d)4

e)5

EUREKA! N° 1, 1998

17


Sociedade Brasileira de Matemática

PROBLEMA 2

O número de pares (x, y) de inteiros que satisfazem a equação x + y + xy = 120 é: a)1

b)2

c)3

d)4

e)6

PROBLEMA 3

O conjunto-solução da inequação

1 1 é o conjunto: > x x−1

a) dos reais diferentes de 0 e de 1. b) dos reais positivos diferentes de 1. c) dos reais diferentes de zero e menores que 1. d) dos reais entre 0 e 1. e) vazio. PROBLEMA 4

O número de valores inteiros de m para os quais as raízes da equação x2 – (m + m2)x + m3 – 1 = 0 são inteiras é igual a: a)0

b)1

c)2

d)3

e)4

PROBLEMA 5

Os vértices de um decágono regular convexo ABC...J devem ser coloridos usando-se apenas as cores verde, amarela e azul. De quantos modos isso pode ser feito se vértices adjacentes não podem receber a mesma cor? a)1022

b)1024

c)1026

d)1524

e)1536

PROBLEMA 6

Uma das soluções inteiras e positivas da equação 19x + 97y = 1997 é, evidentemente, (x0, y0) = (100,1). Além desse, há apenas mais um par de números inteiros e positivos, (x1, y1), satisfazendo a equação. O valor de x1+y1 é: a)23

b)52

c)54

d)101 e)1997

EUREKA! N° 1, 1998

18


Sociedade Brasileira de Matemática

PROBLEMA 7

Selecionam-se 3 vértices de um cubo. Qual é a probabilidade de eles pertencerem a uma mesma face? 1 1 1 2 3 b) c) d) e) a) 5 6 7 7 7 PROBLEMA 8

Sendo k inteiro, o número de valores distintos de sen a)5

b)8

c)9

d)10

9

é igual a:

e)18

PROBLEMA 9

Para cobrir um terraço em forma de um retângulo ABCD, usa-se uma placa plana de alumínio apoiada em quatro estacas verticais fixadas nos vértices do retângulo. A placa fica inclinada em relação ao chão para escoar a água das chuvas. Se as estacas que partem dos vértices A, B e C têm comprimentos respectivamente iguais a 3, 4 e 5 metros, o comprimento da que parte de D é: a)3m

b)4m

c)5m

d)6m

e)8m

PROBLEMA 10

Se seu salário sobe 26% e os preços sobem 20%, de quanto aumenta o seu poder aquisitivo? a)5% b)6% c)7% d)8% e)9% PROBLEMA 11

O reservatório de um caminhão-tanque tem a forma de um cilindro de re3 volução com eixo horizontal e está cheio até da altura. A fração da ca4 pacidade total do resevatório que está ocupada é de aproximadamente: a)80%

b)75%

c)68%

EUREKA! N° 1, 1998

19

d)60%

e)56%


Sociedade Brasileira de Matemática

PROBLEMA 12

O preço de um estacionamento é formado por um valor fixo para as duas primeiras horas e um adicional por cada hora subseqüente. Se o estacionamento por 3 horas custa R$ 5,00 e por 5 horas custa R$ 6,00, quanto custa o estacionamento por 8 horas? a)R$ 7,00 e)R$ 13,33

b)R$ 7,50

c)R$ 9,60

d)R$ 12,00

PROBLEMA 13

O número de soluções reais da equação x2 = 2x é: a)0

1

b)1

c)2

d)3

e)4

Você sabia… que π é aproximadamente:

3,14159265358979323846264338327950288419716939937510582097494459230781 640628620899862803482534211706798214808651328230664709384460955058223 17253594081284811174502841027019385211055596446229489549303819644288109 756659334461284756482337867831652712019091456485669234603486104543266 482133936072602491412737245870066…?

EUREKA! N° 1, 1998

20


Sociedade Brasileira de Matemática

Respostas dos problemas de treinamento Primeira fase da Olimpíada Brasileira de Matemática. Primeiro nível

1) a

2) c

3) b

4) e

5) b

2) c

3) d

4) e

5) b

Segundo nível

1) a Terceiro nível

1) d

2) a

3) b

4) c

5) b

6) e

RESPOSTAS DA PRIMEIRA FASE -OBM JÚNIOR- 1997 1) d 2) b 3) c 4) c 5) c 6) e 7) c

8) a 9) a 10) b 11) e 12) c 13) b 14) e

15) c 16) d 17) e 18) d 19) d 20) c

RESPOSTAS DA PRIMEIRA FASE -OBM SÊNIOR- 1997 1) b 2) e 3) d 4) c 5) c

6) a 7) e 8) c 9) b 10) a

11) a 12) b 13) d

Você sabia que o erro da aproximação

de π por 355/113 é menor que 3×10-7?

EUREKA! N° 1, 1998

21


Sociedade Brasileira de Matemática

A OLIMPÍADA DE MAIO Introdução A Federação Iberoamericana de Competições de Matemática organizou pela primeira vez a Olimpíada de Maio no ano de 1995. A competição está dividida em dois níveis: estudantes menores de 13 anos e estudantes menores de 15 anos. O concurso se realiza por correspondência e está baseado no modelo que segue a Olimpíada de Matemática do Pacífico (APMO), concurso de longa distância com grande tradição. En maio deste ano se realizará a IV Olimpíada de maio, seguindo o calendário seguinte: •

Limite para o envio dos problemas.

31 de janeiro

Envio dos enunciados das provas aos delegados de cada país:

11 de abril

Prova:

Limite da chegada dos listados e provas para cada país:

09 de maio, 14h

Envio dos resultados e diplomas de honra aos delegados de cada país:

13 de junho 27 de junho

A seguir apresentamos as provas da III Olimpíada de Maio, realizada em maio de 1997, com as respectivas soluções.

EUREKA! N° 1, 1998

22


Sociedade Brasileira de Matemática

III OLIMPÍADA DE MAIO Primeiro nível Duração da prova: 3 horas. Cada problema vale 10 pontos. Não se pode usar máquina de calcular. Não se pode consultar livros nem notas.

1)

Num tabuleiro quadrado de 9 casas (de três por três), deve-se colocar nove elementos do conjunto S = {0, 1, 2, 3, 4, 5, 6, 7,8,9 }, distintos um do outro, de modo que cada um deles fique numa casa e se verifiquem as seguintes condições:

As somas dos números da segunda e terceira fileira sejam, respectivamente, o dobro e o triplo da soma dos números da primeira fileira. As somas dos números da segunda e terceira coluna sejam, respectivamente, o dobro e o triplo da soma dos números da primeira coluna.

Mostre todas as formas possíveis de colocar elementos de S no tabuleiro,cumprindo com as condições indicadas. 2) A

M

Q

D

B

No retângulo ABCD, M, N, P e Q são os pontos médios dos lados. Se a área do triângulo sombreado é 1, calcular a área do retângulo ABCD.

N

P

C

EUREKA! N° 1, 1998

23


Sociedade Brasileira de Matemática

3)

Num tabuleiro de 8 por 8, colocam-se 10 fichas que ocupam, cada uma, uma casa. Em cada casa sem ficha está escrito um número entre 0 e 8, que é igual à quantidade de fichas colocadas nas casas vizinhas. Casas vizinhas são as que têm um lado ou um vértice em comum.Mostre uma distribuição das fichas que faça que a soma dos números escritos no tabuleiro seja a maior possível.

4)

Joaquín e seu irmão Andrés vão todos os dias para a aula no ônibus da linha 62. Joaquín paga sempre as passagens. Cada passagem tem impresso um número de 5 dígitos. Um dia, Joaquín observa que os números das passagens, além de consecutivos, são tais que a soma dos dez dígitos é precisamente 62.

Andrés pergunta para ele se a soma dos dígitos de algum dos boletos é 35 e, ao saber a resposta, pôde dizer corretamente o número de cada boleto. Quais são estes números?

5)

Quando Pablo fez 15 anos, fez uma festa convidando 43 amigos. Ele tem uma torta com forma de polígono regular de 15 lados e sobre ela coloca 15 velas.

As velas são colocadas de modo que entre velas e vértices nunca há três alinhados (três velas quaisquer não estão alinhadas, nem duas velas quaisquer com um vértice do polígono, nem dois vértices quaisquer do polígono com uma vela). Logo depois, Pablo divide a torta em pedaços triangulares, mediante cortes que unem velas entre si ou velas e vértices, mas nunca se cruzam com outros já realizados. Por que, ao fazer isto, Pablo consegue distribuir um pedaço para cada um de seus amigos mas ele fica sem comer?

EUREKA! N° 1, 1998

24


Sociedade Brasileira de Matemática

SOLUÇÕES

Nas condições do problema, a soma de todos os elementos do quadrado deve ser um múltiplo de 6. Como 0 + 1 +2 +3 + 4 + 5 + 6 + 7 + 8 + 9 = 45, que deixa resto 3 quando dividido por 6, as únicas possibilidades para o conjunto dos números que aparecem no quadrado são 0, 1, 2, 3, 4, 5, 6, 7, 8 e 0, 1, 2, 4, 5, 6, 7, 8, 9 (note que retiramos respectivamente 3 e 9 que são os elementos que deixam resto 3 quando divididos por 6). No primeiro caso a soma dos elementos da primeira linha (e da primeira coluna) deve ser :

1)

6=

45 − 9 6

E no segundo: 7 =

45 − 3 6

No primeiro caso, as possibilidades para o conjunto resp. C1 elementos da primeira linha (resp. da primeira coluna) são:{0,1,5}, {0,2,4} e {1,2,3} 1.a)

Se L1 = {0,1,5} e C1 = {0,2,4}, temos a única solução

 015   024       236  , e, por simetria, se L1 = {0,2,4} e C1 = {0,1,5}, temos 138   487   567      a) Se L1 {0,1,5} e C1 = {1,2,3} ou L1 = {1,2,3} e C1 = {0,1,5}, temos

105  123       246  ou  048   387   567      b) Se L1 = {0,2,4} e C1 = {1,2,3} ou L1 = {1,2,3} e C1 = {0,1,5}, temos

EUREKA! N° 1, 1998

25


Sociedade Brasileira de Matemática

 204   213      156  ou  057   378   468      No segundo caso, as possibilidades para L1 (ou C1) são {0,1,6}, {0,2,5} e {1,2,4} (não pode aparecer o 3). Para cada escolha de L1 e C1 temos uma única possibilidade de solução, e as soluções são:

 016   025  106  124   205   214           248 , 149 ,  257 ,  059 , 167 e 068   597   687   498   678   489   579          2)

A

Q

D

M

O

B

T P

Sejam O o centro do retângulo e T a interseção de ON com BP. Os triângulos ∆OTP e ∆OTB são de áreas iguais, pois têm a mesma base e igual altura (OP = NB). Como T é o ponto médio, os triângulos ∆OTP e ∆NTB são iguais, ambos são de área 1.Então, a área do ∆OTP é 2 e, como é a metade da área de ONB, a área de ABCD é 16.

N

C

3)

Cada ficha soma 1 em cada uma das casas vizinhas que estão livres de ficha. Uma casa tem como máximo 8 vizinhas ( perde vizinhas se está numa borda do tabuleiro). Vejamos que é impossível colocar as 10 fichas em 10 casas isoladas, tais que nenhuma fique na borda do tabuleiro. Podemos pensar que temos um tabuleiro de 6 por 6 –pois as casas das bordas não EUREKA! N° 1, 1998

26


Sociedade Brasileira de Matemática

interessam– ou dividimos em 9 setores 2 por 2, mediante paralelas aos lados. Se queremos selecionar casas isoladas, em cada setor podemos escolher ao máximo 1. São, em total, no máximo 9 casas isoladas. Se uma casa fica na borda do tabuleiro, terá como máximo 5 vizinhas. Ou seja ao colocar uma ficha ali, somará no máximo 5. Por outro lado, podemse colocar 8 fichas isoladas mais 2 nas quais as casas se toquem num vértice; neste caso se perde só uma casa vizinha por cada uma delas. A soma total é: 8 × 8 + 2 × 7 = 78. Duas possíveis distribuições são as seguintes: 1 1 2 1 2 1 1 0

1 * 2 * 2 * 1 0

2 2 4 2 4 2 2 0

1 * 2 * 2 * 1 0

2 2 4 2 4 2 2 0

1 * 2 * 2 * 2 1

1 1 2 1 2 2 * 1

0 0 0 0 0 1 1 1

0 0 1 1 1 1 1 1

1 1 2 * 2 2 * 1

1 * 3 2 3 * 2 1

2 2 3 * 3 2 2 0

1 * 3 2 3 * 1 0

2 2 3 * 3 2 2 0

1 * 2 1 2 * 1 0

1 1 1 0 1 1 1 0

4)

Se o número menor é abcde, e deve ser 9, pois caso contrário o maior seria abcd (e + 1), e a soma dos dez dígitos é 2 ( a + b + c + d + e ) + 1, que é ímpar e não poderia ser nunca 62. Além disso, se o número menor acaba num número par de noves (99 ou 9999), a soma dos dez dígitos também seria um número ímpar. Assim, o número menor é abcd9 (d não é 9) ou ab999 (b não é 9).

No primeiro caso, o outro número será abc(d + 1)0, e a soma dos dez dígitos 2( a + b + c + d) + 10 = 62, ou seja, a + b + c + d = 26, e os dígitos EUREKA! N° 1, 1998

27


Sociedade Brasileira de Matemática

do número menor somam a + b + c + d +9 = 35. Haveria mais de um número de cinco dígitos nessas condições 85859, 77669, etc.,) pelo que a resposta que deu Joaquín à pergunta do seu irmão foi "não". Assim, os números serão:

ab999 e a(b + 1)000, a soma dos dez dígitos é 2(a + b) + 28 = 62; assim: a + b = 17, e como b não é 9, a = 9 e b = 8. Os números dos boletos são: 98999 e 99000 5)

Seja n o número de triângulos em que se pode dividir a torta com as condições dadas. Somaremos os ângulos interiores destes n triângulos de duas formas:

(1) (2)

180° n 360° × 15 + 180° (15 – 2)

Cada ponto interior (vela) contribui com 360°, e a soma dos ângulos interiores de um polígono convexo de L lados, é 180° (L – 2). Portanto: 180° n = 360° × 15 + 180° × 13, onde n = 43. Também pode utilizar-se a relação de Euler de um mapa plano: R + V = L + 1 ( R = regiões, V = vértices e L = lados). Então V = 30, 3R = 2L – 15 ( Todos os lados são comuns a duas regiões, exceto os 15 lados do contorno do polígono). Assim, pois: 2 L − 15 + 30 = L + 1 Portanto: L = 72 e R = 43 3

Você sabi@

que a página web da Olimpíada Brasileira de Matemática é

http://www.obm.org.br

EUREKA! N° 1, 1998

28


Sociedade Brasileira de Matemática

III OLIMPÍADA DE MAIO Segundo nível Duração da prova: 3 horas. Cada problema vale 10 pontos. Não se pode usar máquina de calcular. Não se pode consultar livros nem notas.

1)

Quantos são os números de sete algarismos que são múltiplos de 388 e terminam em 388?

2) A

B P M

D

3)

Q

Em um quadrado ABCD de lado k, colocam-se os pontos P e Q sobre os lados BC e CD , respectivamente, de forma que PC = 3PB e QD = 2QC. Sendo M o ponto de interseção de AQ e PD, determine a área do triângulo QMD em função de k.

C

Temos 10000 fichas iguais com a forma de um triângulo equilátero. Com esses pequenos triângulos se podem formar hexágonos regulares sem superposições de fichas ou vazios. Considere agora o hexágono regular que desperdiça a menor quantidade possível de triângulos. Quantos triângulos sobram?

4)

Nas figuras, assinalam-se os vértices com um círculo. Chamam-se caminhos aos segmentos que unem os vértices. Distribuem-se números inteiros não negativos nos vértices, e nos caminhos se assinalam as diferenças entre os números de seus extremos.

Diremos que uma distribuição é elegante se aparecem nos caminhos todos os números de 1 a n, em que n é o número de caminhos. EUREKA! N° 1, 1998

29


Sociedade Brasileira de Matemática

11

Veja um exemplo de distribuição elegante:

11 0

4 9 2 12

7 5 2 10 12

6

1

1

8

7 3

9

Dar –se possível– uma distribuição elegante para as seguintes figuras. Em caso de não ser possível, mostrar por quê.

5)

• •

Quais são as possíveis áreas de um hexágono com todos os ângulos iguais e cujos lados medem 1,2,3,4,5 e 6 em alguma ordem?

SOLUÇÕES 1) Solução A O número se expressa como: n. 103 + 388, em que n é um número de quatro cifras.

n. 103 + 388 = k. 388. n. 103 = (k – 1).388. Mas 388 = 22.97, então o número n de quatro cifras deve ser múltiplo de 97. EUREKA! N° 1, 1998

30


Sociedade Brasileira de MatemĂĄtica

N = t.97, com 11≤ t ≤ 103. São 93 números.

Solução B Para que um nĂşmero multiplicado por 388 termine em 388, as Ăşltimas cifras devem ser 001, 501, 251 ou 751. O menor mĂşltiplo de 388 que tem sete cifras ĂŠ 388 â‹… 2578, e o maior ĂŠ 388 â‹… 25773. Entre 2578 e 25773 temos: 23 nĂşmeros terminados en 001, desde 3001 atĂŠ 25001 23 nĂşmeros terminados en 501, desde 3501 atĂŠ 25501 23 nĂşmeros terminados en 251, desde 3251 atĂŠ 25251 24 nĂşmeros terminados en 751, desde 2751 atĂŠ 25751 SĂŁo em total: 23 + 23 + 23 + 24 = 93

VocĂŞ sabia que o maior nĂşmero primo conhecido ĂŠ 23021377-1, que tem 909529 dĂ­gitos e foi descoberto com a ajuda de um computador pessoal?

Consulte na Internet a pĂĄgina

http://www.mersenne.org/prime.htm

EUREKA! N° 1, 1998

31


Sociedade Brasileira de Matemática

2)

Sejam D = (0,0), C = (k,0), B = (k,k) e A = (0,k), temos  3k   2k  P =  k ,  e Q =  ,0   4  3  3 3 A equação de PD é y = x, e de AQ é y = k x . Se M= (x0,y0), temos 4 2 3 3 9 4k 3 k x =k− x ⇒ x =k⇒x = ⇒ y = x = , que é a altu0 0 0 0 0 0 2 4 9 4 3 4 ra de M em relação a BQ, donde a área do ∆QMD é

DQ ⋅ (k / 3) (2k / 3)(k / 3) k 2 = = 2 2 9

3)

Um hexágono é a união de 6 triângulos equiláteros iguais. Cada um destes triângulos, se tem lado n, decompõe-se em n2 triângulos pequenos. Lado 1 Lado 2 Lado 3 Lado n

1 triângulo pequeno 4 triângulos pequenos 9 triângulos pequenos n2 triângulos pequenos

O hexágono de lado n contem 6n2 destes triângulos pequenos.

100   = 40  6

Busca-se o maior n tais que 6n2 ≤ 10000⇒ n = 

Usam-se 6 × 402 triângulos pequenos. Perdem-se 400 = (1000 – 6 × 402) triângulos.

EUREKA! N° 1, 1998

32


Sociedade Brasileira de Matemática

4) 0 6 6

1 4

5 2

1 3 4

No segundo caso, não é possível, pois devem aparecer nas arestas os números 1,2,3,…,10 (5 pares e 5 ímpares). Se quatro ou cinco vértices recebem números com a mesma paridade, temos pelo menos 6 arestas pares, portanto a numeração não será elegante. Nos outros casos, teríamos seis arestas ímpares. 5)

Sejam x, y, z, u, v, w os lados consecutivos do hexágono. Prolongamos os lados y, u e w e obtemos um triângulo equilátero. A área é igual à área deste triângulo equilátero menos as áreas de três triângulos equiláteros de lados x, z e v.

Área do hexágono:

3  ( x + y + z )2 − x 2 − v 2 − z 2   4 

Vejamos quais são os possíveis valores de x, y, z, u, v, w. Seja x = 1,

temos w + x + v = y + x + z w + x + v = v + u + z (pois o triângulo de fora é

equilátero) Donde temos

w+v = y+z w+x = u+z E temos v + x = y – u. Não pode ser v – x = 5, porque os únicos dois números que têm diferença 5 são 1 e 6.

EUREKA! N° 1, 1998

33


Sociedade Brasileira de Matemática

Se v – x = 4, temos v = 5, y = 6, u = 2. De w + 6 = z + 7, resulta, além disso, w = 4, z = 3.

Se v – x = 3, então v = 4. Pode ser y = 5, u = 2 ou y = 6, u = 3. O primeiro caso é impossível, porque não quedam valores de w, z tais que w + 5 = z + 7. O segundo também é impossível, pois não restam valores de w, z tais que w + 4 = z +6

Se v – x = 2, temos v = 3 e pode ser y = 6, u = 4 ou y = 4, u = 2 No primeiro caso, w + 4 = z + 7, donde w = 5, z = 2.

Se v – x = 1, temos v = 2 e pode ser y = 4, u = 3 ou y = 5, u = 4 ou y = 6, u = 5. O primeiro e terceiro casos são impossíveis. No segundo caso, w + 3 = 6 + z, onde w = 6, z = 3.

3

Os possíveis valores da área são:

3 4

(81 − 1 − 4 − 9) = 67

4

(100 − 1 − 25 − 9) = 65

3 4

Os hexágonos são: 3 6

5

3 4

2

1

1 4

5

2

EUREKA! N° 1, 1998

34

6

3 4


Sociedade Brasileira de Matemática

9a. OLIMPÍADA DE MATEMÁTICA DO CONE SUL Salvador - BA, 13 a 21 de junho de 1998 A 9ª. Olimpíada de Matemática dos países do Cone Sul será realizada em Salvador, BA, no período de 13 a 21 de junho de 1998. Esta Olimpíada será realizada pela segunda vez no país (a primeira foi em 1993, em Petrópolis, RJ). Dela participam alunos de até 15 anos dos seguintes países: Argentina, Brasil, Bolívia, Chile, Paraguai, Peru e Uruguai. A organização da Olimpíada está a cargo da Professora Luzinalva Amorim, da Universidade Federal da Bahia. A equipe brasileira será selecionada através de provas realizadas em março e maio deste ano e será liderada pelos professores Paulo Cezar Pinto Carvalho, do IMPA, e Florêncio Ferreira Guimarães, da UFES. A competição consta de duas provas, realizadas em dois dias, cada uma com três problemas, valendo 10 pontos cada. Veja abaixo as provas da última Olimpíada de Matemática do Cone Sul, realizada em Assunção (Paraguai), em 1997, e os resultados obtidos pela equipe brasileira.

Você sabi@ que a Olimpíada Brasileira de Matemática já tem página web??

Visite-nos no endereço eletrônico

http://www.obm.org.br

EUREKA! N° 1, 1998

35


Sociedade Brasileira de Matemática

8a. OLIMPÍADA DO CONE SUL 21 a 25 de Abril de 1997. Assunção, Paraguai. Primeiro dia. Tempo: três horas. PROBLEMA 1

De cada número inteiro positivo n, n ≤ 99, subtraímos a soma dos quadrados de seus algarismos. Para que valores de n esta diferença é a maior possível? PROBLEMA 2

Seja C uma circunferência de centro O, AB um diâmetro dela e R um ponto qualquer em C distinto de A e de B. Seja P a interseção da perpendicular traçada por O a AR. Sobre a reta OP se marca o ponto Q, de maneira que QP é a metade de PO e Q não pertence ao segmento OP. Por Q traçamos a paralela a AB que corta a reta AR em T. Chamamos de H o ponto de interseção das retas AQ e OT. Provar que H, R e B são colineares. PROBLEMA 3

Demonstrar que existem infinitos ternos (a, b, c), com a, b, c números naturais, que satisfazem a relação: 2a2 + 3b2 – 5c2 = 1997.

Segundo dia. Tempo: três horas. PROBLEMA 4

Considere um tabuleiro de n linhas e 4 colunas. Na 1a. linha são escritos 4 zeros (um em cada casa). A seguir, cada linha é obtida a partir da linha anterior realizando a seguinte operação: uma das casas, a escolher, é mantida como na linha anterior; as outras três são troEUREKA! N° 1, 1998

36


Sociedade Brasileira de Matemática

cadas: se na linha anterior havia um 0, coloca-se 1; se havia 1, coloca-se 2; e se havia 2, coloca-se 0. Construa o maior tabuleiro possível com todas as suas linhas distintas e demonstre que é impossível construir um maior. PROBLEMA 5

Seja n um número natural, n > 3. Demonstrar que entre os múltiplos de 9 menores que 10n há mais números com a soma de seus dígitos igual a 9(n-2) que números com a soma de seus dígitos igual a 9(n-1). PROBLEMA 6

Considere un triângulo acutângulo ABC, e seja X um ponto do plano do triângulo. Sejam M, N e P as projeções ortogonais de X sobre as retas que contêm as alturas do triângulo ABC. Determinar para que posições de X o triângulo MNP é congruente a ABC.

Nota: a projeção ortogonal de um ponto X sobre uma reta l é a interseção de l com a perpendicular a ela que passa por X.

RESULTADOS OBTIDOS PELA EQUIPE BRASILEIRA

BRA 1 BRA 2 BRA 3 BRA 4

Murali Srinivasan Vajapeyam Rui Lopes Viana Filho Christian Iveson Daniele Véras de Andrade

EUREKA! N° 1, 1998

37

OURO OURO BRONZE BRONZE


Sociedade Brasileira de Matemática

NÚMEROS MÁGICOS E CONTAS DE DIVIDIR Carlos Gustavo Tamm de Araújo Moreira ♦ Nível Iniciante. Temas muito inocentes de aritmética básica, como contas de multiplicar, podem gerar resultados bastante interessantes e surprendentes, como ao multiplicar o número 142857 por 2, 3, 4, 5, 6 e 7: 142857 × 2 142857 × 3 142857 × 4 142857 × 5 142857 × 6

= = = = =

285714 428571 571428 714285 857142

Por que razão acontece essa repetição dos dígitos de 142857 ao multiplicálo por 2, 3, 4, 5 e 6, sempre com a mesma ordem circular? Será mera coincidência? Será possível obter outros exemplos desse tipo? A resposta tem a ver com o resultado de 142857 × 7, que é 999999. Isso quer dizer que o período da representação decimal de 1/7 é exatamente 142857. Vamos examinar com cuidado a conta de divisão de 1 por 7: 10 30 20 60 40 50 1

7 0,142857

repetindo o resto 1, o que quer dizer que todo o processo se repete e o resultado da divisão é 1/7 = 0,142857142857142857… Podemos reescrever o processo assim: 1 10 30 20 60 40

=0×7+1 =1×7+3 =4×7+2 =2×7+6 =8×7+4 =5×7+5

EUREKA! N° 1, 1998

38


Sociedade Brasileira de Matemática

50 = 7 × 7 + 1. Daí temos: 10 – 7 × 1 = 3, e portanto 100-7 × 10 = 30, e como 30 – 7 × 4 = 2 temos: 100 – 7 (10 + 4) = 2, e analogamente obtemos: 1000 – 7 (100 + 40 + 2) = 6 10000 – 7 (1000 + 400 + 20 +8) = 4 100000 – 7 (10000 + 4000 + 200 + 80 + 5) = 5 1000000 – 7 (100000 + 40000 + 2000 + 800 + 50 + 7 ) = 1 ( A última igualdade diz que 142857 × 7 = 999999) Desta forma, os restos sucessivos que aparecem na divisão de 1 por 7, que são 3, 2, 6, 4, 5, 1 são, respectivamente, os restos na divisão por 7 de 10, 100, 1000, 10000, 100000 e 1000000. Estes restos assumem todos os valores possíveis entre 1 e 6 e isso equivale ao fato de o período de 1/7 ter 6 casas. Desta forma, temos: 2 × 0,142857142857142857… = 2/7 = 100/7–14 = 100 × 0, 14285714 2857142857… – 14 = 0,285714285714285714…, e, portanto, temos 2 × 142857 = 285714 Da mesma maneira temos que 3/7 = 10/7 – 1 implica 3 × 142857 = 428571, e as outras igualdades seguem de modo análogo. Notemos agora que sempre que o período da representação decimal de 1/n tiver n –1 casas decimais (que é o máximo possível), o período (que será igual a (10n-1 –1) / n ) terá as mesmas propiedades de 142857. O primeiro valor de n maior que 7 para o qual isso acontece é 17, e o período de 1/17 é 0588235294117647. Multiplique esse número por 2, 3, 4, 5, 6, 7, 8, 9, 10, 11, 12, 13, 14, 15, 16 e 17 para conferir. Observe que, para que isso aconteça, n deve ser um número primo, pois se n = p × b, com b maior que 1 e p um número primo diferente de 2 e 5, então p nunca aparecerá como resto na divisão de 1 por n, pois em geral um fator primo comum de n e de um resto que aparece na divisão de 1 por n só pode ser 2 ou 5 ( de fato, um resto que aparece na divisão de 1 por n é resto da divisão de alguma potência de 10 por n ). Por outro lado, se os únicos fatores primos de n são 2 e 5, então 1/n tem representação decimal finita.

EUREKA! N° 1, 1998

39


Sociedade Brasileira de Matemática

Conclusão: Se o período de 1/n tiver n–1 casas decimais, ele terá propiedades análogas às de 142857: os dígitos de seus produtos por 1, 2, 3, 4, …, n–1 serão sempre os mesmos, na mesma ordem circular. Para que isso aconteça, n deve ser primo e a menor potência de 10 que deixa resto 1 quando dividida por n deve ser 10n–1. Dizemos que, nesse caso, 10 é raiz primitiva módulo n. Não se sabe se existem infinitos primos n com essa propriedade. Isso seguiria de uma famosa conjectura de teoria dos números, a conjectura de Artin (vide [V]). Os números primos n menores que 100 tais que o período de 1/n na base 10 tem n –1 casas são 7, 17, 19, 23, 29, 47, 59, 61 e 97. Por outro lado, para todo número primo n existem números naturais B entre 2 e n – 1 tais que o período de 1/n na base B tem exatamente n – 1 casas (nesses casos B é raiz primitiva módulo n). Se um número B tem essa propriedade, todas as bases da forma kn + B com k natural também têm. Nesses casos, o período de 1/n na base B ( ou seja, o número (Bn-1–1)/n ), quando multiplicado por 1, 2, 3, …, n – 1 terá representações na base B que serão permutações uma da outra com a mesma ordem circular. Por exemplo, com n = 5 e B = 8, temos que a representação de 1/5 na base 8 é 0,146314631463… Na base 8 temos: 2 × (1463)8 =(3146)8 , 3 × (1463)8 = (4631)8 , 4 × (1463)8 = (6314)8 , 5 × (1463)8 = (7777)8

Referências: [L] Lima, Elon L., Meu Professor de Matemática e outras histórias, pp. 158-170 – SBM, 1991. [T] Tahan, Malba, O homen que calculava, Ed. Record. [V] Voloch, José Felipe, Raizes Primitivas e a Conjectura de Artin, Revista Matemática Universitária Nº9/10, dezembro de 1989, pp. 153-158.

EUREKA! N° 1, 1998

40


Sociedade Brasileira de Matemática

COMO PERDER AMIGOS E ENGANAR PESSOAS Nicolau C. Saldanha ♦ Nível Avançado. Neste artigo apresentaremos quatro situações simples em que probabilidades enganam. Em alguns casos a probabilidade de certos eventos tem um valor diferente daquele que a maioria das pessoas parece julgar razoável, pelo menos de início; em um exemplo mostraremos como é facil chegar a conclusões absurdas. Para que o leitor possa pensar sozinho, apresentaremos primeiro quatro "enunciados", em que lançamos cada situação, e depois quatro "desenvolvimentos" em que voltamos a discutir as quatro situações na mesma ordem. Qualquer um pode usar estes exemplos para divertir-se às custas de seus amigos, mas em nenhum caso o autor tem responsabilidade pela integridade física daqueles que usarem a Matemática para o mal.

ENUNCIADOS 1. Em um programa de auditório, o convidado deve escolher uma dentre três portas. Atrás de uma das portas há um carro e atrás de cada uma das outras duas há um bode. O convidado ganhará o que estiver atrás da porta; devemos supor neste problema que o convidado prefere ganhar o carro. O procedimento para escolha da porta é o seguinte: o convidado escolhe inicialmente, em caráter provisório, uma das três portas. O apresentador do programa, que sabe o que há atrás de cada porta, abre neste momento uma das outras duas portas, sempre revelando um dos dois bodes. O convidado agora tem a opção de ficar com a primeira porta que ele escolheu ou trocar pela outra porta fechada. Que estratégia deve o convidado adotar? Com uma boa estratégia, que probabilidade tem o convidado de ganhar o carro? 2. Um móvel tem três gavetas iguais. Em uma gaveta há duas bolas brancas, em outra há duas bolas pretas, e na terceira há uma bola branca e outra preta. Abrimos uma gaveta ao acaso e tiramos uma bola ao acaso sem olhar a segunda bola que está na gaveta. A bola que tiramos é branca. Qual é a probabilidade de que a segunda bola que ficou sozinha na gaveta seja também branca? EUREKA! N° 1, 1998

41


Sociedade Brasileira de Matemática

3. Dois amigos querem decidir quem pagará a conta do restaurante com

uma aposta. Cada um deles escolhe uma seqüência de três caras ou coroas, e eles jogam uma moeda até que saia uma das duas seqüências: aquele que tiver escolhido a primeira seqüência a sair ganhou a aposta. Por exemplo, André (por ser o primeiro em ordem alfabética) é o primeiro a escolher e fica com a seqüência ckc (em que c representa cara e k coroa) enquanto Bernardo responde com cck. Eles jogam a moeda obtendo kckkckkkkccck, e neste momento Bernardo declara-se o vencedor. Esta aposta é justa? André leva vantagem ou desvantagem por ser o primeiro a escolher? Quais são as probabilidades de vitória de cada um?

4. Aqui novamente devemos nos imaginar em um programa de auditório. Eugênio foi sorteado e tem direito a um prêmio, mas ele deve escolher entre dois envelopes lacrados aparentemente iguais. O apresentador informa que cada envelope tem um cheque e que o valor de um cheque é o dobro do outro, mas não diz nada sobre o valor dos cheques, nem indica qual envelope contém o cheque de maior valor. Eugênio escolhe e abre um envelope que contém um cheque de, digamos, R$ 100. Neste momento, o apresentador sempre faz uma proposta ao convidado: ele pode trocar de envelope mediante uma multa de 5% do valor do cheque que ele tem em mãos, no caso, R$ 5. Assim, se Eugênio aceitar, ele pode ganhar R$ 45 (se o cheque no segundo envelope for de R$ 50) ou R$ 195 (se o outro cheque for de R$ 200). Suponhamos que Eugênio (que fez um curso de Introdução à Probabilidade no período anterior) queira maximizar o valor esperado de seu prêmio. Deve ele aceitar a troca? E se o valor do primeiro cheque tivesse sido outro, de que forma deveria isto influenciar a decisão de Eugênio? Se Eugênio trocar de envelope independentemente do valor do cheque, não vale mais a pena para ele trocar de envelope antes de abrir, evitando, assim, a multa? DESENVOLVIMENTOS

1. A resposta correta é que, trocando de porta, a probabilidade de ganhar o carro é 2/3, enquanto não trocando a probabilidade é apenas 1/3. Uma forma simples de ver isto é a seguinte: trocando de porta, o convidado ganha, desde que a primeira porta que ele escolher esconda um dos dois bodes, como se pode facilmente perceber. A melhor estratégia para o convidado é, EUREKA! N° 1, 1998

42


Sociedade Brasileira de Matemática

portanto, trocar sempre, e assim sua probabilidade de ganhar fica sendo 2/3. O erro comum aqui é achar que, após a eliminação de uma porta (que foi aberta pelo apresentador, revelando um bode), há uma simetria entre as duas outras portas e a probabilidade de cada uma esconder o carro é 1/2. Não existe, entretanto, tal simetria, pois a porta escolhida pelo convidado não poderia, pelas regras, ser trocada pelo apresentador, enquanto a outra poderia ter sido aberta, mas não foi. Este processo de fato era seguido em um programa nos Estados Unidos. Uma longa e áspera discussão ocorreu na imprensa quanto a qual era o valor correto da probabilidade, e pessoas que deveriam ser capazes de resolver um problema trivial como este passaram pela vergonha de publicar soluções erradas. Julgamos melhor esquecer os detalhes deste episódio deprimente.

2. A resposta correta é 2/3 (e não 1/2). As seis bolas seriam de início igualmente prováveis, mas sabemos que a primeira bola escolhida foi branca: assim, as três bolas brancas têm igual probabilidade. Estamos interessados em saber a cor da companheira de gaveta de cada bola branca: em dois casos é branca, em um caso é preta. Assim, a probabilidade de que a segunda bola seja branca é 2/3, como já afirmamos. Um raciocínio comum, mas errado, é dizer: as gavetas são igualmente prováveis, mas obviamente não escolhemos a gaveta que contém duas bolas pretas. Portanto, teríamos probabilidade 1/2 de termos escolhido a gaveta com duas bolas brancas e 1/2 de termos escolhido a gaveta com uma bola de cada cor; no primeiro caso, a segunda bola é branca e, no segundo caso, a bola é preta. Assim, a resposta seria 1/2. O que há de errado neste raciocínio? O erro está em dizer que as duas gavetas possíveis são igualmente prováveis. Inicialmente a probabilidade de cada gaveta é de fato a mesma (inclusive para a gaveta com duas bolas pretas), mas, ao tirarmos uma bola e constatarmos que ela é branca, isto deixa de ser verdade. Isto é bem óbvio para a gaveta com duas bolas pretas: passou a ser impossível termos escolhido esta gaveta. Entre as duas outras gavetas, entretanto, há uma diferença que está sendo ignorada no raciocínio do parágrafo anterior. Se pré-escolhermos a gaveta com duas bolas branEUREKA! N° 1, 1998

43


Sociedade Brasileira de Matemática

cas, temos certeza de passar no teste: uma bola escolhida ao acaso nesta gaveta será sempre branca. Por outro lado, se pré-escolhermos a gaveta com uma bola de cada cor, ainda temos probabilidade 1/2 de sacarmos uma bola preta, o que estaria em contradição com o enunciado. Assim, a probabilidade de termos escolhido cada uma destas duas gavetas é 2/3 e 1/3, respectivamente. Podemos, a partir deste ponto facilmente deduzir a resposta correta de 2/3. É fato empírico desencorajador que muitas pessoas teimam em dizer que a probabilidade é 1/2 mesmo após esta explicação. O seguinte exemplo serve como exercício para aqueles que entenderam a explicação e é uma espécie de redução ao absurdo do raciocínio "rival". Temos novamente três gavetas, uma com vinte bolas brancas, uma com vinte bolas pretas e a terceira com dez bolas de cada cor. Abrimos uma gaveta e, sem olhar, retiramos ao acaso dez bolas: elas são todas brancas. Qual a probabilidade de que as dez bolas restantes sejam também brancas?

3. No nosso exemplo, Bernardo tinha probabilidade 2/3 de ganhar. Em geral, o segundo a jogar leva uma vantagem considerável e, se escolher bem sua resposta, pode garantir uma probabilidade de vitória de pelo menos 2/3, mas às vezes até 7/8, dependendo da primeira jogada. A Tabela 1 dá a probabilidade de vitória de Bernardo para cada par de jogadas (a coluna é a jogada de André e a linha a de Bernardo).

ccc ccc ckc ckk kcc kck kkc kkk

ccc

cck

ckc

ckk

kcc

kck

kkc

kkk

_ 1/2 3/5 3/5 7/8 7/12 7/10 1/2

1/2 _ 1/3 1/3 3/4 3/8 1/2 3/10

2/5 2/3 _ 1/2 1/2 1/2 5/8 5/12

2/5 2/3 1/2 _ 1/2 1/2 1/4 1/8

1/8 1/4 1/2 1/2 _ 1/2 2/3 2/5

5/12 5/8 1/2 1/2 1/2 _ 2/3 2/5

3/10 1/2 3/8 3/4 1/3 1/3 _ 1/2

1/2 7/10 7/12 7/8 3/5 3/5 1/2 _ Tabela 1

Não reconstruiremos aqui toda a tabela: apresentaremos apenas como exemplo a situação descrita no enunciado. O leitor que estiver interessado em aprender mais sobre este problema pode consultar nosso Precisa-se de alguém para ganhar muito dinheiro, a ser publicado na Revista do ProfesEUREKA! N° 1, 1998

44


Sociedade Brasileira de Matemática

sor de Matemática do Chile, mas já disponível na home page do autor: http://www.mat.puc-rio.br/~nicolau/. O Diagrama 2 descreve bem a situação. Os seis vértices indicam as seis situações possíveis durante o processo de jogar a moeda. O ponto indica que nenhum jogador tem como esperar fazer uso das jogadas já feitas, ou seja, ou nenhum lance ainda foi feito, ou foi lançado apenas um k, ou os dois últimos lances foram kk; como o jogo sempre começa nesta situação, chamaremos este vértice de inicial. O c indica que o último lance foi um c mas o anterior ou não existiu ou foi um k. Os vértices cc e ck indicam que estes foram os dois últimos lances. Finalmente, os vértices cck e ckc indicam que o jogo terminou; chamaremos estes vértices de finais.

cc

cck

ck

ckc

c

Diagrama 2 As duas setas partindo de cada vértice (exceto os finais) indicam como a situação se modifica a cada lance de moeda: elas correspondem às possibilidades de tirar c ou k em um dado momento. Queremos agora calcular a probabilidade de vitória de Bernardo, dado que o jogo chegou a uma certa situação. Temos, assim, quatro probabilidades a serem calculadas: p., pc, pcc e pck; consideramos naturalmente pcck = 1 e pckc = 0. Como a partir de cada vértice não final as probabilidades associadas às duas setas são iguais, temos as seguintes equações:

p. = 1/2 (p. + pc) pc = 1/2 (pcc + pck) pcc= 1/2 (pcc + 1) pck= 1/2 p. . Resolvendo o sistema, temos p. = 2/3, conforme afirmamos. O erro mais natural aqui é achar que todas as seqüências são igualmente boas: isto não é verdade, pois os dois últimos lances em geral serviram, EUREKA! N° 1, 1998

45


Sociedade Brasileira de Matemática

sem sucesso, para tentar finalizar as seqüências e servirão agora para tentar iniciá-las. Mais surpreendente ainda é o fato de que o segundo jogador sempre tem uma boa resposta: este jogo é um pouco como jogar par-ouímpar ou pedra-papel-tesoura com um dos jogadores tendo o direito de escolher sua jogada só depois de ver a jogada do adversário.

4. Antes de mais nada gostaríamos de lembrar que Eugênio deseja, por hipótese, maximizar o valor esperado do prêmio. Este critério é razoável em algumas situações e em outras não. Outro convidado poderia precisar desesperadamente de uma certa quantia, talvez R$ 100, e gostaria, portanto, de maximizar a probabilidade de ganhar pelo menos este valor crítico. Ainda outro convidado pode ser tão curioso que deseja saber quanto há em cada envelope mais do maximizar seu prêmio. O leitor, se fosse o convidado, talvez julgasse interessante considerar ainda outros aspectos. Podemos imaginar inúmeros critérios diferentes e em princípio cada critério gera um novo problema. Nós nos propomos aqui a estudar o problema na forma em que foi proposto e não a discutir se Eugênio, com sua opção pelo valor esperado, é um homem verdadeiramente sábio. Neste problema, ao contrário dos outros, apresentaremos inicialmente um raciocínio falho e vamos segui-lo até chegarmos a um absurdo deixando a análise dos erros deste raciocínio para o final. Para tornar a discussão toda mais viva, acompanharemos o pensamento de Eugênio. Ao receber a proposta de troca, Eugênio pensa: Se ficar com este cheque, meu prêmio será de R$ 100. Se trocar de cheque, tenho probabilidade 1/2 de ganhar R$ 45 e probabilidade 1/2 de ganhar R$ 195: o valor esperado é de (1/2) ⋅ 45 + (1/2) ⋅ 195 = 120 reais. Como 120 é maior que 100, a troca é vantajosa. Eugênio troca de cheque e fica felicíssimo ao ver que o outro cheque é de R$ 200: ele ganhou R$ 195! Ao voltar para seu lugar no auditório, Eugênio continua pensando: Na verdade vale a pena trocar qualquer que seja o valor do primeiro cheque. Se chamarmos este valor de x, temos por um lado a opção de ficar com x e por outro lado a opção de arriscar, com probabilidade 1/2 de ganhar 0.45x e probabilidade 1/2 de ganhar 1.95x. No primeiro caso, o valor esperado é x e, no segundo caso, o valor esperado é 1.2x. Assim, como x > 0, EUREKA! N° 1, 1998

46


Sociedade Brasileira de Matemática

vale sempre a pena trocar. Eugênio fica feliz com sua conclusão e pensa como seu curso de Probabilidade foi útil. Mas um pouco mais tarde Eugênio começa a ter dúvidas quanto a suas conclusões: Se vale a pena trocar de envelope sempre, então não é necessário abrir o envelope e ler o valor do cheque para tomar a decisão de trocar. Neste caso, eu poderia ter trocado de envelope um minuto antes e ter evitado a multa. Eugênio fica irritado, pensando que poderia ter ganhado 5 reais a mais se apenas tivesse pensado mais rápido. Mas ele continua pensando: Ei, espere, há algo errado! Um minuto antes os dois envelopes estavam lacrados e pareciam iguais para mim: trocar significaria apenas escolher o outro. Mas, então, cada vez que eu penso em um envelope tenho que trocar e nunca posso escolher nada! Assim, ao invés de aproveitar seu prêmio, Eugênio passa a noite angustiado com seu paradoxo. Na manhã seguinte, Eugênio procura seus colegas do curso de Probabilidade com a pergunta: o que exatamente há de errado com este raciocínio? O erro de Eugênio está logo no início, quando aceita, sem aliás sequer questionar, que a probabilidade do segundo cheque ser maior é 1/2. O leitor deve estar muito surpreso: é quase como se de repente disséssemos que cara e coroa têm probabilidades diferentes. Por isso daremos uma explicação relativamente longa para tentar convencer. Começaremos fazendo algumas digressões considerando o que um outro convidado, o João, que nunca estudou probabilidade, mas que tem bom senso, faria em algumas situações extremas. João não acompanha todos os sorteios, mas mesmo assim ele certamente tem alguma noção, por vaga que seja, de qual a faixa dos prêmios. Assim, se o valor do primeiro cheque fosse muito baixo, João certamente pensaria: Não é possível, ou pelo menos não é provável, que o segundo cheque seja ainda menor. Assim, quase certamente eu peguei o envelope com o cheque de menor valor (além de ter tido o azar de vir em um dia em que os prêmios foram baixos) e aposto que o outro cheque é maior: vou trocar. Por outro lado, se o valor do primeiro cheque fosse muito alto, seu pensamento seria: Que sorte, hoje os prêmios estão ótimos! E é muito improvável que o segundo cheque seja ainda maior! Vou ficar com este cheque mesmo! Assim, João não atribui probabilidades iguais às duas possibilidades (o segundo cheque ser maior ou meEUREKA! N° 1, 1998

47


Sociedade Brasileira de Matemática

nor), e as probabilidades que ele atribui (inconscientemente) a estes dois eventos dependem do valor do primeiro cheque. Bem, este era o João e não o Eugênio: ao considerá-lo, desviamo-nos temporariamente do problema original e do contexto que nos impusemos no primeiro parágrafo deste desenvolvimento, pois João nem sabe o que é o valor esperado e seus critérios não são os de Eugênio. João atribuiu subjetivamente probabilidades diferentes aos dois eventos; Eugênio (que aliás não se defrontou com situações extremas) atribuiu probabilidades iguais. Será que em algum sentido é errado atribuir sempre probabilidades iguais? Sim, atribuir probabilidades sempre iguais é não apenas errado, mas contraditório com a Teoria da Probabilidade que Eugênio tenta usar. Para entender isto, vamos representar cada configuração inicial de envelopes por um par ordenado (x1, x2) de números reais positivos: x1 é o valor do cheque no primeiro envelope escolhido pelo convidado, e x2 é o valor do segundo cheque. Assim, o espaço amostral Ω ⊆ R2 é a união de duas semi-retas abertas partindo da origem, como mostrado na Figura 3. A história que contamos envolvendo Eugênio corresponde ao ponto (100, 200), também indicado. Ao abrir o primeiro envelope, definimos o valor de x1 e ficamos restritos à interseção de Ω com uma reta vertical, ou seja, aos dois pontos (x1, x2 = 2x1) e (x1, x2 = x1/2). Eugênio implicitamente aceita que a probabilidade condicional a um valor qualquer fixo para x1 destes dois pontos é 1/2. Assim, ele deve aceitar que:

P({(t, 2t); t ∈ [T, 2T)}) = P({(t, t/2); t ∈ [T, 2T)}) para qualquer número positivo T, em que P(C), C ⊆ Ω, denota a probabilidade de que (x1, x2) esteja em C. Por outro lado, a simetria inicial entre os envelopes diz que

P({(t, 2t); t ∈ [T, 2T)}) = P({(2t, t); t ∈ [T, 2T)}). Sejam

An = {(t, 2t); t ∈ [2n, 2n+1)}, Bn = {(2t, t); t ∈ [2n ,2n+1)},

EUREKA! N° 1, 1998

48


Sociedade Brasileira de Matemática

em que n é um inteiro qualquer; as identidades acima nos dão P(An) = P(Bn-1) e P(An) = P(Bn), respectivamente. Por indução, P(An) = P(Bn) = P(A0) para todo n. Observemos desde já que esta conclusão é no mínimo estranha: ela diz que a probabilidade de o valor de menor cheque estar entre 64 e 128 é igual à probabilidade de o menor cheque estar entre 264 e 265, ou entre 24199021 e 24199022; no próximo parágrafo veremos que esta conclusão é não apenas estranha, mas realmente absurda, mesmo ignorando o fato de que um prêmio de R$ 24199021 é uma impossibilidade prática.

X2

300

200

(100,200)

100 (100,50)

100

200

300

X1 Figura 3

Observemos que os conjuntos An e Bn são dois a dois disjuntos e sua união é Ω. Se P(A0)>0, podemos tomar N ∈ N tal que N P(A0)>1 e temos

P

 ∪ An    0 ≤ n < N  > 1,  

o que é absurdo. Por outro lado, se P(A0) = 0 temos

EUREKA! N° 1, 1998

49


Sociedade Brasileira de Matemática

 ∪ ( An − N < n < N 

P 

Bn ) 

 = 0 

para todo N, o que também é um absurdo, pois, quando N cresce, este conjunto também cresce, tendendo no limite para Ω, donde teríamos P(Ω)= 0, contradizendo P(Ω) = 1. Assim, em qualquer caso, temos um absurdo. Esta explicação é um pouco técnica, mas coincide perfeitamente com o "bom senso" de João: não podemos ignorar o primeiro cheque. Se seu valor for muito baixo, a probabilidade de que o segundo cheque seja maior deve em geral ser muito maior do que 1/2, pois P(An) deve tender a zero quando n tende a –∞. Por outro lado, se o seu valor for muito alto, a probabilidade de que o segundo cheque seja ainda maior deve ser muito menor do que 1/2, pois P(An) também deve tender a zero quando n tende a +∞. E Eugênio, afinal de contas, precisa fazer uma avaliação sutil, dependendo de que valores são plausíveis como prêmio: até um certo valor-limite vale a pena trocar, acima deste valor não.

Nicolau C. Saldanha Departamento de Matemática, PUC-RIO Gávea, Rio de Janeiro, RJ 22453-900, BRASIL nicolau@mat.puc-rio.br, http://www.mat.puc-rio.br/~nicolau/

EUREKA! N° 1, 1998

50


Sociedade Brasileira de Matemática

DOIS PROBLEMAS SOBRE GRAFOS Paulo Cezar Pinto Carvalho IMPA ♦ Nível Intermediario. INTRODUÇÃO A figura abaixo mostra um mapa rodoviário de um país fictício. Neste artigo vamos examinar dois problemas relativos a este mapa: 1. Um funcionário, encarregado de verificar, periodicamente, o estado das estradas, deseja planejar a sua rota de inspeção. Idealmente, esta rota deveria se iniciar na capital e percorrer cada estrada exatamente uma vez, voltando, então, ao ponto de partida. Existe tal rota? 2. Um representante de vendas de uma companhia deseja planejar uma rota na qual ele visite cada cidade exatamente uma vez, voltando ao ponto de partida. Existe tal rota?

Fig. 1 - Mapa rodoviário de um país fictício Há vários pontos em comum entre os dois problemas. Por exemplo: em ambos se deseja verificar a existência de um circuito (ou ciclo) no grafo determinado pelo mapa (um grafo é um par (V, A), em que V é o conjunto de vértices do grafo, e A é um conjunto de pares de vértices – os arcos do grafo). No primeiro problema, este circuito deve incluir exatamente uma vez cada arco do grafo. No segundo problema, o circuito deve incluir exatamente uma vez cada vértice do grafo. Embora os dois problemas sejam aparentemente semelhantes, há algumas diferenças fundamentais entre eles. Convidamos os leitores a refletir um pouco sobre cada um deles antes de prosseguir.

EUREKA! N° 1, 1998

51


Sociedade Brasileira de Matemática

CIRCUITOS EULERIANOS O primeiro problema – o do inspetor de estradas – foi estudado pela primeira vez por Euler (1707-1783). Por esta razão, um circuito que percorre cada arco de um grafo exatamente uma vez é chamado de circuito euleriano e um grafo que possui um tal circuito é chamado de grafo euleriano. A situação estudada por Euler ficou imortalizada como o Problema das Pontes de Könisberg, ilustrado na figura abaixo, e que possivelmente já é conhecido por muitos dos leitores. O objetivo é percorrer exatamente uma vez todas as sete pontes da cidade (hoje Kaliningrado), que conectam as duas ilhas entre si e com as margens do rio, voltando ao ponto de partida.

Fig. 2 – O Problema das Pontes de Könisberg

Em linguagem de grafos, trata-se de encontrar um circuito euleriano no grafo da figura acima, no qual os vértices representam as ilhas e as margens e os arcos são as pontes1. Euler mostrou a não-existência de tal circuito através de um argumento extremamente simples. Consideremos, por exemplo, a ilha da direita. Um circuito qualquer deve chegar à ilha e sair dela o mesmo número de vezes. Logo, para que exista um circuito euleriano, deve haver um número par de pontes com extremidade nesta ilha. Como existem três pontes nessas condições, concluímos que não é possível encontrar um circuito euleriano. De modo mais geral, temos o seguinte:

Teorema: Existe um circuito euleriano em um grafo se e somente se o grafo é conexo (isto é, existe um caminho ligando qualquer par de vértices) e cada vértice tem grau par (ou seja, o número de arcos que nele incidem é par).

1

A rigor, neste caso temos um multi-grafo, já que certos pares de vértices são ligados por mais de um arco.

EUREKA! N° 1, 1998

52


Sociedade Brasileira de Matemática

O argumento acima mostra a necessidade de se ter grau em cada vértice para existir um circuito euleriano. É também óbvio que o grafo precisa ser conexo. A prova de que essas duas condições implicam na existência de um circuito euleriano pode ser feita por indução finita no número de arcos do grafo e é deixada como um exercício para o leitor. [Sugestão: suponha a propriedade verdadeira para grafos com menos de n arcos e considere um grafo com n arcos, satisfazendo às duas condições. Começando em um vértice qualquer, percorra arcos do grafo, até voltar a um vértice já visitado (o caminho gerado possui, então, um ciclo). Retirando do grafo os arcos desse ciclo, obtém-se um ou mais grafos satisfazendo as duas condições e com menor número de arcos (portanto, com circuitos eulerianos, de acordo com a hipótese de indução). Basta explicar como “costurar” esses circuitos eulerianos ao ciclo descrito acima]. Podemos aplicar este teorema ao nosso problema de inspeção de estradas. Da mesma forma como no Problema das Pontes de Könisberg, não existe qualquer circuito euleriano no grafo determinado pelo mapa rodoviário, já que o vértice correspondente à capital tem grau 3. Assim, se o nosso inspetor de estradas recebesse de seu chefe a incumbência de elaborar um trajeto nas condições do problema 1, ele poderia facilmente convencê-lo da impossibilidade de fazê-lo. Como veremos a seguir, a situação do seu colega representante de vendas é bem pior...

CIRCUITOS HAMILTONIANOS Um circuito passando exatamente uma vez por cada vértice de um grafo é chamado de circuito hamiltoniano, em homenagem ao matemático irlandês William Rowan Hamilton (1805-1865), que estudou este problema no grafo determinado pelas arestas de um dodecaedro regular (existe ou não um circuito hamiltoniano neste caso?). Um grafo que possui um circuito hamiltoniano é chamado de grafo hamiltoniano. A situação do problema de verificar se um grafo é hamiltoniano é bem diferente da do problema anterior. Apesar de terem sido estudados por vários séculos, não há uma boa caracterização dos grafos hamiltonianos. Há diversas famílias de grafos para os quais existe um circuito hamiltoniano (um exemplo trivial é um grafo completo, em que cada vértice é EUREKA! N° 1, 1998

53


Sociedade Brasileira de Matemática

ligado a todos os outros); também é possível estabelecer certas condições que implicam na não-existência de um circuito. Mas uma caracterização geral não foi encontrada e, à luz de certos avanços em teoria da computação das últimas décadas, parece improvável que ela seja encontrada algum dia. O problema de decidir se um grafo é hamiltoniano está na companhia de diversos problemas ilustres, com as seguintes características em comum:

O problema possui uma assimetria fundamental: é muito fácil convencer alguém da existência de um circuito hamiltoniano em um grafo: basta exibir tal caminho. No entanto, é difícil, em geral, convencer alguém da não-existência de um tal circuito. Por exemplo, o grafo da figura abaixo (o leitor é capaz de reconhecê-lo?) tem um circuito hamiltoniano, de cuja existência o leitor fica imediatamente convencido pela figura. Já o grafo dado no início do artigo não tem circuito hamiltoniano, mas não existe um argumento simples e geral para demonstrar esse fato (assim, nosso amigo representante de vendas certamente terá mais trabalho para convencer seu chefe da impossibilidade de elaborar uma rota nas condições do problema 2 do que seu colega inspetor de estradas).

Fig. 3 – Um grafo hamiltoniano •

Não se conhece um algoritmo eficiente para verificar se um grafo é hamiltoniano (por eficiente, entendemos aqui um algoritmo em que o número de passos seja limitado por um polinômio no número de vértices do grafo). Além disso, parece improvável que um tal algoritmo possa algum dia ser encontrado, porque sua existência implicaria na existência de algoritmos eficientes para um grande número de outros

EUREKA! N° 1, 1998

54


Sociedade Brasileira de Matemática

problemas, para os quais também não se conhecem algoritmos eficientes. Estes problemas (incluindo o de verificar a existência de circuito hamiltoniano) formam uma classe de problemas chamados de NPcompletos. Um outro problema famoso da classe é o de determinar o número mínimo de cores que podem ser usadas para colorir os vértices de um grafo de modo que vértices de mesma cor não sejam ligados por um arco. O leitor poderá estar pensando assim: mas será que esta história de algoritmos eficientes tem relevância, numa era de computadores cada vez mais velozes? Afinal de contas, existe um algoritmo extremamente simples para verificar se um grafo possui um circuito hamiltoniano. Se existir um tal circuito, ele corresponderá a uma permutação (circular) dos vértices com a propriedade de que vértices consecutivos sejam ligados por um arco do grafo. Ora, para verificar a existência de circuito hamiltoniano basta gerar todas as permutações circulares dos vértices e testar se uma delas corresponde a um percurso no grafo. É claro que este algoritmo funciona para grafos de tamanho moderado (ele poderia ser o recurso usado pelo nosso vendedor: como são apenas 9 cidades, ele teria que testar “apenas” 8! = 40.320 caminhos, o que seria feito com rapidez em um computador). Mas o que ocorre com grafos maiores? Vejamos, por exemplo, uma situação em que o número de cidades cresce para 50 (o que representaria um tamanho ainda bastante razoável para uma situação real). Neste caso, o computador deveria examinar 49! circuitos potenciais. Tentemos estimar a magnitude deste número. A forma mais simples é usar a fórmula de Stirling, que fornece a estimativa n

n n! ≈ 2πn   . Mas, neste caso, podemos usar estimativas mais elemene tares. Por exemplo, podemos usar apenas potências de 2. Temos: 49! =

1 × 2 × 3 × 4 × 5 × 6 × 7 × 8 × ... × 15 × 16 × ... × 31 × 32 × … × 49 > 1 × 2 × 2 × 4 × 4 × 4 × 4 × 8 × ... × 8 × 16 × ... × 16 × 32 × … × 32 = 22 x 44 x 88 x 1616 x 3218 = 22+8+64+90 = 2164. Mas 210 = 1024 >103. Logo 49! > 16. 1048.

Ora, um computador moderno pode realizar cerca de 200 milhões de operações por segundo. Se em cada operação ele conseguir testar um circuito, ele ainda assim precisará de mais de 16. 1048 / 2. 106 = 8 × 1042 seEUREKA! N° 1, 1998

55


Sociedade Brasileira de Matemática

gundos, o que corresponde a aproximadamente a 2 × 1035 anos. Assim, trata-se claramente de uma missão impossível para o algoritmo de força bruta baseado na análise de cada permutação de vértices.

PROBLEMAS DIFÍCEIS QUE TAMBÉM SÃO ÚTEIS O resultado da discussão acima pode parecer bastante desanimador: não parece haver bons métodos para verificar a existência de um circuito hamiltoniano e algoritmos de força bruta só funcionam para problemas com pequeno número de vértices (é bom que se diga que existe um meio termo: há estratégias que permitem resolver o problema acima para valores razoáveis de n, reduzindo substancialmente o número de possibilidades a serem examinadas; mesmo estes algoritmos, no entanto, tornam-se impráticos a partir de um certo ponto). O mesmo ocorre com todos os chamados problemas NP-completos. No entanto, ao invés de ficarmos deprimidos com esta característica desses problemas, podemos explorá-la para uma importante finalidade em criptografia, que é a parte da Matemática que estuda métodos para criar e decifrar códigos. Para tal, é também muito importante a assimetria apontada acima (e que ocorre em todos os problemas NP-completos): é difícil encontrar um circuito hamiltoniano (ou mostrar que não existe um), mas é fácil testar se uma seqüência de vértices forma um circuito hamiltoniano. Suponhamos que você seja cliente de um banco. Para ter acesso aos serviços, você usa o número de sua conta (que é público) e uma senha, que em princípio deve ser conhecida apenas por você. O procedimento mais simples seria ter a sua senha armazenada no sistema do banco. Mas aí você correria o risco de que ela fosse descoberta, por exemplo, por um funcionário desonesto. Em lugar disto, o sistema do banco armazena uma versão codificada da senha, que não precisa ficar em segredo. Esta codificação deve ser feita de tal forma que seja simples verificar se sua senha está correta (para que você seja autorizado a retirar dinheiro do caixa eletrônico), mas seja praticamente impossível recuperar a senha a partir da versão codificada. Problemas NP-completos servem como uma luva para esta tarefa. Se quiséssemos usar o problema do circuito hamiltoniano, poderíamos agir mais ou menos da formadescrita a seguir. O cliente poderia escolher uma permutação dos números de 1 a 50, conhecida apenas por ele. A partir desEUREKA! N° 1, 1998

56


Sociedade Brasileira de Matemática

sa informação, seria gerado um grafo, contendo necessariamente os arcos correspondentes ao circuito (os demais poderiam, por exemplo, ser gerados por um método aleatório, em que cada um dos possíveis arcos teria uma certa probabilidade de sere escolhido). Este grafo seria armazenado no sistema. A figura a seguir mostra uma representação de uma permutação dos números de 1 a 50 e um grafo, gerado aleatoriamente, que possui um ciclo hamiltoniano dado por esta permutação.

Fig. 4 – Um ciclo hamiltoniano e um grafo gerado a partir dele Quando o cliente fosse utilizar sua conta, o sistema simplesmente verificaria se a permutação apresentada corresponde a um caminho no grafo. Como é improvável que um tal ciclo pudesse ser encontrado para um grafo deste tamanho, dificilmente um impostor conseguiria se fazer passar pelo cliente, ainda que conhecesse o grafo-problema. Na prática, são utilizados outros problemas NP-completos para se fazer codificação de senhas, mas a idéia é exatamente a mesma acima.

PALAVRAS FINAIS Grafos são uma fonte inesgotável de problemas com enunciado simples mas que escondem, muitas vezes, uma sofisticada estrutura matemática. Neste artigo abordamos apenas alguns aspectos de dois desses problemas. Certamente voltaremos a falar em grafos em outros artigos desta revista. Para o leitor que deseja saber mais sobre o assunto, recomendamos os livros a seguir:

• •

Jaime Luiz Szwarcfiter. Grafos e Algoritmos Computacionais. Editora Campus. Oynstein Ore. Graphs and Their Uses. The Mathematical Association of America.

EUREKA! N° 1, 1998

57


Sociedade Brasileira de Matemática

PROBLEMAS PROPOSTOS Convidamos o leitor a enviar soluções dos problemas propostos e sugestões de novos problemas para os próximos números. 1)

Mostre que, dado um conjunto de n pessoas, existem duas que possuem o mesmo número de amigos entre as pessoas do conjunto.

2)

Em uma pista circular há postos de gasolina, e o total de gasolina que há nos postos é exatamente o suficiente para um carro dar uma volta. Prove que existe um posto de onde um carro com o tanque inicialmente vazio pode partir e conseguir dar uma volta completa na pista (parando para reabastecer nos postos).

3)

Prove que existe n ∈ N tal que os últimos 1000 dígitos de n1998 são iguais a 1.

4)

Escreva 1998 como soma de (um número arbitrário de ) parcelas de modo que o produto das parcelas seja o maior possível.

5)

Sejam a > 0 e P1P2P3P4P5 uma poligonal aberta contida em um dos semi-planos determinados pela reta P1 P5 . Prove que é possível escolher pontos P6 e P7 no plano com P5 P6 = a de modo que é possível ladrilhar o plano com infinitos ladrilhos congruentes ao heptágono P1P2P3P4P5P6P7.

6)

Mostre que toda seqüência com n2 + 1 elementos possui uma subseqüência crescente com n + 1 elementos ou uma subseqüência decrescente com n + 1 elementos.

7)

Prove que

8)

Considere um torneio de xadrez envolvendo brasileiros e argentinos em que cada jogador joga contra todos os outros exatamente uma vez. Ao final do torneio, cada jogador obteve metade dos pon-

1 + 2 + 3 + ... + 1998 < 2

EUREKA! N° 1, 1998

58


Sociedade Brasileira de Matemática

tos que conquistou jogando contra brasileiros e metade jogando contra argentinos. Prove que o número total de jogadores do torneio é um quadrado perfeito (obs: cada vitória vale 1 ponto, empate 1/2 ponto e derrota 0 ponto).

9)

Prove que todo número racional positivo pode ser escrito como soma de um certo número de frações distintas de numerador 1.

Você sabia… que

cos

2π 5

cos

=

2π 17

5 −1 4

=

4

e

− 1 + 17 + 34 − 2 17 + 2 17 + 3 17 − 34 − 2 17 − 2 34 + 2 17

mas não é possível escrever cos

16

2π 7

e cos

EUREKA! N° 1, 1998

59

2π 9

usando radicais reais

?


Sociedade Brasileira de Matemática

AGENDA OLÍMPICA IV OLIMPÍADA DE MAIO 09 de maio, 14 h ♦ OLIMPÍADA DO CONE SUL 13 a 21 de junho de 1998 Salvador – BA. ♦ OLIMPÍADA BRASILEIRA DE MATEMÁTICA Primeira Fase – Sábado, 6 de junho Segunda Fase – Sábado, 12 de setembro Terceira Fase – Sábado, 24 de outubro (níveis 1,2 e 3) Domingo, 25 de outubro (nível 3). ♦ 39 a. OLIMPÍADA INTERNACIONAL DE MATEMÁTICA 10 a 21 de julho Taiwan. ♦ OLIMPÍADA IBEROAMERICANA DE MATEMÁTICA 13 a 20 de setembro de 1998 República Dominicana.

EUREKA! N° 1, 1998

60


Sociedade Brasileira de Matemática

COORDENADORES REGIONAIS Alberto Hassen Raad Antônio C. Rodrigues Monteiro Amarísio da Silva Araújo Angela Camargo Antônio C. do Patrocínio Benedito T. Vasconcelos Freire Carlos A. Bandeira Braga Claudio Arconcher Élio Mega Florêncio F. Guimarães F. Francisco Dutenhefner Gisele de A. Prateado G. João B. de Melo Neto José Carlos Pinto Leivas José Paulo Carneiro José Vieira Alves Leonardo Matteo D'orio Luzinalva M. de Amorim Marco Polo Marcondes Cavalcante França Mario Jorge Dias Carneiro Ma-To-Fú Pablo Rodrigo Ganassim Paulo H. Cruz Neiva de L. Jr. Reinaldo Gen Ichiro Arakaki Ricardo Amorim Sergio Claudio Ramos

(UFJF) (UFPE) (UFV) (Esc. Tec. Hermann Hering) (IMECC/UNICAMP) (UFRGDN) (UFPB) (Col. Leonardo da Vinci) (Col. ETAPA) (UFES) (UFMG ) (UFGO) (UFPI) (URG) (USU) (UFPB) (Parque de Material Aeronáutico de Belém) (UFBA) (Colégio Singular) (UF Ceará) (UFMG) (UEM) (L. Albert Einstein) (Esc. Tec.Everardo Passos) (INPE) (Centro Educ. Logos) (IM-UFRGS)

EUREKA! N° 1, 1998

61

Juiz de Fora-MG Recife-PE Viçosa-MG Blumenau-SC Campinas-SP Natal-RN João Pessoa-PB Jundiaí-SP São Paulo-SP Vitória-ES BH-MG Goiânia-GO Teresina-PI Rio Grande-RS Rio de Janeiro-RJ Campina Grande-PB Belém-PA L. de Freitas-BA Santo André-SP Fortaleza-CE BH-MG Maringá-PR Rio das Pedras-SP Piracicaba-SP S.J.Campos-SP Nova Iguaçu-RJ Porto Alegre-RS


CONTEÚDO AOS LEITORES

2

OLIMPÍADA BRASILEIRA DE MATEMÁTICA Problemas de treinamento para a Segunda Fase

4

XIX OLIMPÍADA BRASILEIRA DE MATEMÁTICA Problemas Júnior Segunda Fase e Soluções

10

IV OLIMPÍADA DE MAIO Resultados

16

IV OLIMPÍADA DE MAIO Prova

17

9a. OLIMPÍADA DE MATEMÁTICA DO CONE SUL

21

9a. OLIMPÍADA DE MATEMÁTICA DO CONE SUL Problemas e soluções

22

39a. OLIMPÍADA INTERNACIONAL DE MATEMÁTICA Resultados e problemas

30

ARTIGOS PARIDADE Eduardo Wagner

32

OS PROBLEMAS DO VISITANTE MATEMÁTICO

39

DIVISIBILIDADE, CONGRUÊNCIAS E ARITMÉTICA MÓDULO n 41 Carlos Gustavo Moreira SOLUÇÕES DE PROBLEMAS PROPOSTOS EUREKA N°1

53

PROBLEMAS PROPOSTOS

59

AGENDA OLÍMPICA

61

COORDENADORES REGIONAIS

62


Sociedade Brasileira de Matemática

AOS LEITORES Iniciamos este segundo número da revista EUREKA! transmitindo aos leitores nossa satisfação pela acolhida do primeiro número por alunos e professores. A comunidade estudantil e os professores das escolas passam a ter, de forma que esperamos permanente, uma publicação específica que, além de fornecer material para tornar as aulas mais ricas e interessantes, é um veículo de contato entre todos para expor experiências, dirimir dúvidas e nos aproximarmos cada vez mais. Já estamos recebendo correspondência de muitos alunos e alguns professores com respeito às soluções dos problemas propostos. Isto muito nos alegra e temos a certeza de que nos próximos números essa correspondência só tenderá a crescer. Entretanto, gostaríamos de pedir aos professores que nos enviem também colaborações para os números seguintes da revista: problemas interessantes com soluções, pequenos artigos, experiências em sala de aula, olimpíadas ou torneios regionais, enfim, material que seja adequado aos alunos da 5ª série do ensino fundamental à última série do ensino médio. Estas colaborações serão fundamentais para que nossa revista permaneça viva e seja sobretudo útil a toda a comunidade.

A Olimpíada Brasileira de Matemática de 1998 Realizamos a primeira fase da Olimpíada Brasileira de Matemática em mais de mil colégios do nosso país. Em nosso projeto pretendíamos atingir, nesta primeira etapa dessa nova atividade, cerca de 20 000 alunos mas, para nossa surpresa, esse número já superou o dobro do pretendido. Através dos relatórios enviados pelas escolas aos Coordenadores Regionais, estabelecemos as notas de corte para a promoção dos alunos à segunda fase que se realizará em setembro. A terceira fase, já mais centralizada, será feita em outubro e esperamos que no final de novembro possamos divulgar a lista dos alunos premiados. Como em toda competição, é natural que o número de premiados seja relativamente pequeno em relação ao número inicial de participantes. Porém, aqui não há perdedores. Todos são de alguma forma ganhadores: de EUREKA! N° 2, 1998

2


Sociedade Brasileira de Matemática

uma experiência nova, de um estímulo para estudar mais e crescer, ou da possibilidade de ver que objetivos que pareciam longínquos realmente podem ser atingidos. Devemos ainda relatar que alguns colégios não participaram da Olimpíada Brasileira de Matemática com receio de que, sem uma preparação adequada, seus alunos não tivessem um resultado satisfatório. Especialmente para estes colégios enviamos nossa mensagem final: A Olimpíada Brasileira de Matemática não é uma competição entre colégios. A OBM tem como objetivo principal estimular o estudo de Matemática entre os jovens, desenvolver professores e propiciar uma melhoria do ensino e do aprendizado desta matéria nas escolas brasileiras.

Comitê Editorial.

EUREKA! N° 2, 1998

3


Sociedade Brasileira de Matemática

OLIMPÍADA BRASILEIRA DE MATEMÁTICA Problemas de treinamento para a Segunda Fase

Primeiro Nível 1)

Determine o menor inteiro cuja representação decimal consiste somente de 1's e que é divisível pelo número 333…333 formado por 100 algarismos iguais a 3. (Problema proposto por Antonio Luiz Santos.)

2)

Numa gaveta há 6 meias pretas e 6 meias brancas. Qual é o número mínimo de meias a se retirar (no escuro) para garantir que:

a) b)

As meias retiradas contenham um par da mesma cor? As meias retiradas contenham um par de cor branca?

3)

Quando se escrevem os números 1, 2, 3, 4, 5, 6, 7, 8, 9, 10, 11, 12,…1998, qual é o dígito que ocupa o lugar 1998?

Segundo Nível 1)

Determine com quantos zeros consecutivos termina representação decimal do número 1 × 2 × 3 × …× 1998.

2)

Suponha que desejamos saber de qual janela de um prédio de 36 andares é seguro jogarmos ovos para baixo, de modo que os ovos não se quebrem ao atingirem o chão. Para tal, admitimos que:

• • • •

Um ovo que sobrevive a uma queda pode ser usado novamente. Um ovo quebrado deve ser descartado. O efeito da queda é o mesmo para todos os ovos. Se um ovo se quebra quando jogado de uma certa janela então ele quebrará se jogado de uma altura superior. Se um ovo sobrevive a uma queda então ele sobreviverá a uma queda menor.

EUREKA! N° 2, 1998

4

a


Sociedade Brasileira de Matemática

Não se sabe se da janela do primeiro andar os ovos quebram, e também não se sabe se da janela do último andar os ovos quebram.

Se temos apenas 1 ovo e queremos ter certeza de obter um resultado correto, o experimento deve ser guiado apenas por um único caminho: jogue o ovo pela janela do primeiro andar; se não se quebrar, jogue o ovo pela janela do segundo andar. Continue até que o ovo se quebre. Na pior das hipóteses, este método necessitará de 36 lançamentos para ser concluído. Suponha que 2 ovos estão disponíveis. Qual é o menor número de lançamentos de ovos necessários para garantir todos os casos? 3)

Considere cinco pontos quaisquer P1, P2, …, P5 no interior de um quadrado de lado 1. Mostre que pelo menos uma das distâncias dij entre Pi e Pj é menor que

2/2.

Terceiro Nível 1)

Determine quantos números naturais menores que 1998 têm um número ímpar de divisores positivos.

2)

Mostre que, dados 5 pontos do plano em posição geral (isto é três pontos quaisquer nunca estão em linha reta), há 4 que formam um quadrilátero convexo.

3)

Dois discos A e B são divididos em 2n setores iguais. No disco A, n setores são pintados de azul e n de vermelho. No disco B, os setores são pintados de azul ou vermelho de forma completamente arbitrária. Mostre que A e B podem ser superpostos de modo que pelo menos n setores tenham cores coincidentes.

EUREKA! N° 2, 1998

5


Sociedade Brasileira de Matemática

Soluções do Primeiro Nível 1)

É claro que d = 333...333 = 3 ⋅ 111...111 = 3n . Portanto, o número 100 três

 

100 uns

 

procurado N = 111...111 deve ser divisível por n e por 3 (n não é divisível k uns

por 3 porque a soma dos seus algarismos é igual a 100 que não é divisível por 3). Se k é um número da forma k = 100q + r onde 0 ≤ r < 100 então obviamente N = 111...111 000...00+ 111...11 = M + R . Como M é 100 q uns

r zeros

r uns

divisível por n então, N é divisível por n se, e somente se, R = 0 ou seja, se r = 0 e conseqüentemente se, e somente se, k for divisível por 100. Se k = 100q então a soma dos algarismos de N é igual a 100q e esta soma será divisível por 3 (e consequentemente também o número N)se, e somente se, q for divisivel por 3. Portanto, o menor número N = 111...111 divisível k uns

por d consiste em 300 uns.

2) a)

3 meias (necessariamente teremos 2 meias brancas ou 2 meias pretas; se tirarmos apenas duas pode ser que uma seja branca e a outra preta).

b)

8 meias (se tirassemos apenas 7 meias poderiam ser 6 pretas e apenas uma branca).

3) Quando se escrevem os números do 1 ao 99, usam-se 9 + 2 (99 – 9) = 189 dígitos. Ficam por preencher 1809 (1998 – 189) lugares. Para cada uma das centenas que seguem usam-se 300 dígitos. Como 1809 = 300 × 6 + 9, ao terminar de escrever os 1998 dígitos se escrevem todos os números desde o número 1 até completar 7 centenas (do número 1 até 699) e 9 dígitos mais: 700, 701 e 702. Portanto o dígito que ocupa o lugar 1998 é o número 2.

EUREKA! N° 2, 1998

6


Sociedade Brasileira de Matemática

Soluções do Segundo Nível 1) A resposta é 496. Se a decomposição de 1 × 2 × …× 1998 em fatores primos é 2a ⋅ 3b ⋅ 5c…, temos necessariamente c < a, pois para todo r natural há mais múltiplos de 2r que de 5r entre 1 e 1998. Assim, o número de zeros do final de 1 × 2 × …× 1998 é igual a c. Para determinar c, observamos que entre 1 e 1998 há 399 múltiplos de 5 (pois 399 × 5 < 1998 < 400 × 5), 79 múltiplos de 25, 15 múltiplos de 125, 3 múltiplos de 625 mas nenhum múltiplo de 3125, e portanto temos c = 399 + 79 + 15 + 3 = 496. (De fato, ao contar os múltiplos de 5, que são 399, já contamos os múltiplos de 25, mas estes devem ser contados pelo menos em dobro para calcular o exponente de 5, por isso somamos 79, mas é preciso contar os múltiplos de 125 pelo menos 3 vezes e só foram contados 2 vezes, por isso somamos 15. E assim por diante.)

2) 8 lançamentos. Jogamos o primeiro ovo do oitavo andar. Se quebrar, basta testar os 7 primeiros com o segundo ovo. Se não quebrar, o jogamos do 15°., depois do 21°., depois do 26°., depois do 30°., depois do 33°., depois do 35°. e finalmente do 36°.. Se ele quebrar por exemplo quando jogado do 26°. andar, basta testar o segundo ovo nos andares 22, 23, 24 e 25, para o que gastamos 4 + 4 = 8 lançamentos. A escolha dos andares se devem a 8 + 7 = 15, 8 + 7 + 6 = 21, 8 + 7 + 6 + 5 = 26, 8 + 7 + 6 + 5 + 4 = 30, 8 + 7 + 6 + 5 + 4 + 3 + 2 + 1 = 36. O resultado não pode ser melhorado, pois se o primeiro ovo quebra no n-ésimo lançamento, devemos testar com o ovo restante todos os andares entre os usados nos (n – 1)-ésimo e n-ésimo lançamentos, no pior caso. Tente generalizar este problema fazendo variar o número de ovos disponíveis e o número de andares do prédio.

EUREKA! N° 2, 1998

7


Sociedade Brasileira de Matemática

3) Dividimos o quadrado em 4 quadrados de lado 1/2. Necessariamente dois desses pontos, digamos Pi e Pj, estarão num mesmo quadradinho, e sua distância dij será menor que a diagonal do quadradinho (que é a maior 2 distância possível entre dois de seus pontos), ou seja . 2

Soluções do Terceiro Nível 1) α

Se n = p1α1 p 2α 2 ... p k k é a fatoração em primos de n, os divisores positivos β

de n são todos os números da forma p1β1 p 2β 2 ... p k k com 0 ≤ β1 ≤ α1, 0 ≤ β2

≤ α2, …, 0 ≤ βk ≤ αk, βi ∈ N, ∀i. Assim , o número de divisores positivos de n é (1 + α1) (1 + α2)…(1 + αk ). Para que este número seja ímpar é necessário e suficiente que todos os αi sejam pares, ou seja, que n seja quadrado perfeito. Como 442 = 1936 < 1998 < 2025 = 452, há 44 quadrados perfeitos entre 1 e 1998, portanto há 44 naturais menores que 1998 com um número ímpar de divisores positivos. 2) Se o menor polígono convexo que contém os 5 pontos tiver mais de 3 lados o problema é trivial. Caso contrário, dois dentre os 5 pontos (digamos D e E), estão dentro do triângulo cujos vértices são os outros 3. Ao prolongar a reta que une esses dois pontos cortamos dois dos lados do triângulo, digamos AB e AC. Nesse caso, é fácil ver que o quadrilátero BDEC é convexo.

EUREKA! N° 2, 1998

8


Sociedade Brasileira de Matemática

3) Sejam S1, S2, …, S2n os setores do disco B. Tentamos colocar o disco A sobre o disco B nas 2n posições possíveis (com os setores coincidindo). Para cada i com 1 ≤ i ≤ 2n , em exatamente n das posições do disco A o setor Si terá cor coincidente com o setor do disco A que está sobre ele. Assim, o número médio de setores com cores coincidentes nos dois discos para as 2n posições do disco A é 2n ⋅ n/2n = n, e necessariamente há posições do disco A para as quais há pelo menos n setores com cores coincidentes.

2fisdufiows uhf

Você sabia…

que há tantos números racionais quanto

números naturais, mas há estritamente mais números reais que racionais (isto é, existe uma bijeção f : N → Q mas não existe nenhuma bijeção g : Q → R) ? E que é impossível decidir se existe algum conjunto com estritamente mais elementos que os naturais mas estritamente menos elementos que os reais ??

EUREKA! N° 2, 1998

9


Sociedade Brasileira de Matemática

XIX OLIMPIADA BRASILEIRA DE MATEMÁTICA Problemas Júnior Segunda Fase e Soluções A Olimpíada Brasileira Júnior correspondia aproximadamente aos atuais níveis 1 e 2 da OBM. Estamos publicando a prova da segunda fase júnior do ano passado com soluções, a qual acreditamos ser bom material de treinamento tanto para a segunda fase da OBM quanto para a terceira nos níveis 1 e 2. No próximo número da EUREKA! publicaremos a segunda fase da Olimpíada Brasileira sênior do ano passado. PROBLEMA 1

No edifício mais alto de Terra Brasilis moram Eduardo e Augusto. O número do andar do apartamento de Eduardo coincide com o número do apartamento de Augusto. A soma dos números dos apartamentos dos dois é 2164. Calcule o número do apartamento de Eduardo sabendo que há 12 apartamentos por andar. (Por exemplo, no primeiro andar estão os apartamentos de 1 a 12, no segundo, de 13 a 24, e assim por diante.) PROBLEMA 2

A professora de Matemática propôs o seguinte problema para seus alunos: "Marquem 6 pontos sobre uma circunferência. Eu quero que vocês pintem o maior número de cordas determinadas por estes pontos, de modo que não existam quatro dos pontos sobre a circunferência determinando um quadrilátero com todos os lados e diagonais coloridos." a) Edmilson encontrou uma solução correta colorindo 12 cordas. Exiba uma maneira de como fazer isto. b) Gustavo afirmou ter encontrado uma solução na qual pintara 13 cordas. Mostre que a solução de Gustavo não está correta.

EUREKA! N° 2, 1998

10


Sociedade Brasileira de Matemática

PROBLEMA 3

Sejam ABCD um quadrado, M o ponto médio de AD e E um ponto sobre o lado AB. P é a interseção de EC e MB. Mostre que a reta DP divide o segmento EB em dois segmentos de mesma medida. PROBLEMA 4

Mostre que existem infinitos inteiros positivos n satisfazendo simultaneamente as seguintes condições: n é ímpar; i. n possui exatamente 1200 divisores positivos; ii. existem exatamente 1997 triângulos retângulos, dois a dois não iii. congruentes, de lados inteiros e n como medida de um dos catetos. PROBLEMA 5

Seja n ≥ 1 um inteiro. Temos n lâmpadas alinhadas e numeradas, da esquerda para a direita, de 1 a n. Cada lâmpada pode estar acesa ou apagada. A cada segundo, determina-se a lâmpada apagada de maior número e inverte-se o estado desta (de acesa para apagada ou de apagada para acesa) e das lâmpadas posteriores (as lâmpadas de maior número). a)

Mostre que em algum momento todas as lâmpadas estarão acesas (e o processo se encerrará).

b)

Suponha que inicialmente todas as lâmpadas estejam apagadas. Determine depois de quantos segundos todas as lâmpadas estarão acesas.

c)

Suponha agora n = 11 e que no início somente as lâmpadas de números 6, 7 e 10 estejam acesas. Mostre que após exatamente 1997 segundos todas as lâmpadas estarão acesas.

EUREKA! N° 2, 1998

11


Sociedade Brasileira de Matemática

SOLUÇÕES 1)

Seja a o andar do apartamento de Eduardo. Então o número de seu apartamento é 12 (a – 1) + b, com 1 ≤ b ≤ 12. Daí,

a + 12 ( a – 1 ) + b = 2164, b = 2176 – 13a 1 ≤ 2176 – 13a ≤ 12 a = 167, b = 5 Portanto, o número do apartamento de Eduardo é: 12 (a – 1) + b = 12 × 166 + 5 = 1997. 2)

a) Uma maneira é mostrada abaixo:

b) Suponha que a solução de Gustavo esteja correta. Sejam A, B, C, D, E, F os pontos. Então, como os 6 pontos determinam 15 cordas, somente dois segmentos não foram coloridos. Estes dois segmentos incidem em 3 ou 4 vértices. i.) ii.)

Se A é vértice comum de dois segmentos não coloridos, AB e AF, então caso existem 6 quadriláteros totalmente coloridos: ACDE, BCDE, BCDF, BCEF, BDEF e CDEF. Se os segmentos AB e EF não foram coloridos então existem 4 quadriláteros coloridos: CDAE, CDAF, CDBE, CDBF.

EUREKA! N° 2, 1998

12


Sociedade Brasileira de Matemática

3)

A

E

B P

M

N

D

C

Prolongue BM até encontrar o prolongamento do lado CD no ponto N. Claramente, ∆AMB ≡ ∆DMN , donde segue que AB = DN . Portanto, D é o ponto médio de CN. O resultado segue observando que os triângulos CPN e EPB são semelhantes e, como PD é mediana do triângulo CPN, conclui-se que o prolongamento de DP encontra EB em seu ponto médio. 4)

Seja n um número natural ímpar. Vamos calcular o número de triângulos retângulos de lados inteiros nos quais n é medida de um dos catetos. Para isso, devemos ter

n2 + x2 = y2 , n 2 = ( y − x)( y + x), com x e y inteiros positivos, x < y. Observe que (y – x) < (y + x). Se fizermos (y – x) = d, com d um divisor de n2, d será menor que n e (y + x) = n2/d será maior que n. Para qualquer d satisfazendo estas condições, podemos encontrar uma solução:

EUREKA! N° 2, 1998

13


Sociedade Brasileira de Matemática

  1  n2 x =  − d   2 d    y − x = d     n2 ⇒  y + x =   1  n2 d   + d  = y  2 d     Estas soluções são inteiras e positivas, pois n é ímpar (logo d também), e d < n . Portanto, o número de triângulos retângulos é o número de divisores de n2 menor que n. Mas para cada divisor de n2 menor que n, corresponde um divisor maior que n. Lembrando que n é também um divisor, concluímos que o número procurado é 1/2 (d(n2) – 1), onde d(n) é o número de divisores positivos de n. Portanto, é necessário e suficiente que n2 seja um número ímpar com d(n2) = 2 × 1997 + 1 = 3995 divisores. Uma das várias possibilidades para n2 ter 3995 divisores é ser da forma p4q798, com p e q primos distintos. Neste caso, n = p2q399, possui d(n) = (2 +1) × (399 +1) = 1200 divisores. 5)

Vamos representar por 1 uma lâmpada acesa, e por 0 uma lâmpada apagada e interpretar o número obtido na base 2. Veja que se, em algum passo, o último dígito for 0, ele será o único dígito alterado no próximo passo. Isto significa que o número aumentará 1 unidade. Caso contrário, o número terminará com um bloco de 1's antecipado por um 0:…011…1. No próximo passo, o número será …100…0. Mas observe que (…011…1) + 1 = …100…0. Portanto, em qualquer caso, o número k é sucedido pelo número k + 1. a) Dada qualquer disposição inicial das lâmpadas, ou seja, qualquer número binário de no máximo n dígitos, em algum momento todos os dígitos serão iguais a 1, pois este é o maior número de n dígitos na base 2. EUREKA! N° 2, 1998

14


Sociedade Brasileira de Matemática

b) Existem 2n números de no máximo n dígitos na base 2. Começando com 0, devemos chegar a 2n–1, passando por todos os naturais intermediários. São necessários, então, 2n–1 segundos. c) Observe que a configuração inicial representa o número 25 + 24 + 2 = 50. Para n = 11, todas as lâmpadas estarão acesas depois de (211–1) – 50 = 1997 segundos.

Você sabia…

Que um polígono regular com

um número ímpar de lados só pode ser construído exatamente com régua e compasso se o número de lados for um produto 2k

de primos distintos da forma 2 + 1 (esses primos são chamados primos de Fermat) ? E que só são conhecidos 5 primos de Fermat: 3, 5, 17, 257 e 65537, apesar de Fermat ter conjecturado que todo número da forma 32

(isso já é falso para k = 5 : 2

+ 1 é divisível por 641.) ??

EUREKA! N° 2, 1998

15

k

22 + 1

é primo


Sociedade Brasileira de Matemática

IV OLIMPÍADA DE MAIO Resultados Primeiro nível Fabio Dias Moreira Davi M. Alexandrino Nogueira Lyussei Abe Cibele Norie Sakai Uyhara Pedro Davoli Ometto Kelly Correa de Paula Marcelo Kenji Honda Rafael Martins Gomes Nascimento Priscila Carrara Thiago Pimentel Nykiel Rodrigo Evangelista Delgado Luiz Eduardo de Godoi

Ouro Prata Prata Prata Bronze Bronze Bronze Bronze Menção Menção Menção Menção

Coord. Est. Militar Etapa Integrado Koelle M.Schledorn Pioneiro S. Dumont Cass. Ricardo Militar Militar Cass. Ricardo

Rio de Janeiro-RJ Fortaleza-CE São Paulo-SP Itatiba-SP Rio Claro-SP Jundiaí-SP São Paulo-SP Fortaleza-CE S. J. Campos-SP Juiz de Fora-MG Juiz de Fora-MG S. J. Campos-SP

Ouro Prata Prata Bronze Bronze Bronze Bronze Menção Menção Menção

SETA Militar S. Dumont Bandeirantes Militar Progresso 7 de setembro Evolutivo Etapa Bandeirantes

S.J.Rio Preto-SP Fortaleza-CE Fortaleza-CE São Paulo-SP Juiz de Fora-MG Araraquara-SP Fortaleza-CE Fortaleza-CE São Paulo-SP São Paulo-SP

Segundo nível Hugo Pinto Iwata Ulisses Medeiros de Albuquerque Afonso de Paula P. Rocha Artur D. Nelmi Luiz Fernando Mendes Correa Andre de Almeira Bosso Fabricio Siqueira Benevides Luiz Brizeno Firmeza Neto Daniel Nobuo Uno Juliana Regina C. Zucare

Os alunos Fabio Dias Moreira (Rio de Janeiro, RJ) e Hugo Pinto Iwata (São José do Rio Preto, SP) receberam medalha de ouro na Olimpíada e com isso, ganharam uma viagem de uma semana para a Argentina onde se reunirão com os ouros dos outros países para diversas atividades turísticas e culturais. Esta viagem será realizada em outubro, em data que ainda será marcada. A seguir, publicamos a prova da IV Olimpíada de maio, com as respostas dos problemas.

EUREKA! N° 2, 1998

16


Sociedade Brasileira de Matemática

IV OLIMPÍADA DE MAIO Primeiro nível Duração da prova: 3 horas. Cada problema vale 10 pontos. Não se pode usar máquina de calcular. Não se pode consultar livros nem notas.

PROBLEMA 1

Com seis varetas se construiu uma peça como a da figura. As três varetas exteriores são iguais entre si. As três varetas interiores são iguais entre si. Desejase pintar cada vareta de uma cor só de modo que, em cada ponto de união, as três varetas que chegam tenham cores diferentes.As varetas só podem ser pintadas de azul, branco, vermelho ou verde. De quantas maneiras pode-se pintar a peça? PROBLEMA 2

Têm-se 1998 peças retangulares de 2cm de altura e 3cm de comprimento e com elas se armam quadrados (sem superposições nem buracos). Qual é a maior quantidade de quadrados diferentes que se pode ter ao mesmo tempo? PROBLEMA 3

Existem quatro botes numa margem de um rio; seus nomes são Oito, Quatro, Dois e Um, porque essas são as quantidades de horas que cada um deles demora para cruzar o rio. Pode-se atar um bote a outro, porém não mais de um, e então o tempo que demoram em cruzar é igual ao do mais lento dos botes. Um só marinheiro deve levar todos os botes até à outra margem do rio. Qual é o menor tempo necessário para completar o translado?

EUREKA! N° 2, 1998

17


Sociedade Brasileira de Matemática

PROBLEMA 4

ABCD é um quadrado de centro O. Sobre os lados DC e AD foram construidos os triângulos equiláteros DAF e DCE. Decida se a área do triângulo EDF é maior do que, menor do que ou igual à área do triângulo DOC.

E

D F

C

O A

B

PROBLEMA 5

Escolha um número de quatro dígitos ( nenhum deles zero) e começando com ele construa uma lista de 21 números distintos, de quatro dígitos cada um, que satisfaça a seguinte regra: depois de escrever cada novo número da lista devem-se calcular todas as médias entre dois dígitos desse número, descartando-se as médias que não dão um número inteiro, e com os que restam se forma um número de quatro dígitos que ocupará o lugar seguinte na lista. Por exemplo, se na lista se escreveu o número 2946, o seguinte pode ser 3333 ou 3434 ou 5345 ou qualquer outro número armado com os dígitos 3, 4 ou 5.

Segundo nível PROBLEMA 1

Inês escolheu quatro dígitos distintos do conjunto {1,2,3,4,5,6,7,8,9}. Formou com eles todos os possíveis números de quatro dígitos distintos e somou todos eses números de quatro dígitos. O resultado é 193314. Encontre os quatro dígitos que Inês escolheu.

EUREKA! N° 2, 1998

18


Sociedade Brasileira de Matemática

PROBLEMA 2

ABC é um triângulo equilátero. N é um ponto do lado AC tal que AC = 7.AN , M é um ponto do lado AB tal que MN é paralelo a BC e P é

um ponto do lado BC tal que MP é paralelo a AC. Encontre a fração

área( MNP) . área( ABC ) PROBLEMA 3

Dado um tabuleiro quadriculado de 4 × 4, com cada casa pintada de uma cor distinta, deseja-se cortá-lo em dois pedaços de igual área mediante um só corte, que siga os lados das casas do tabuleiro. De quantas maneiras se pode fazer isto? Obs. Os pedaços em que se divide o tabuleiro devem ser peças inteiras; não devem ser desconectados pelo corte. PROBLEMA 4

O chão do pátio tem desenhado um octógono regular. Emiliano escreve nos vértices deste os números de 1 a 8 em qualquer ordem. Deixa uma pedra no ponto 1. Caminha em direção ao ponto 2 e, havendo percorrido 1/2 do caminho, se detém e deixa a segunda pedra. Daí caminha em direção ao ponto 3 e, havendo percorrido 1/3 do caminho, se detém e deixa a terceira pedra. Daí caminha em direção ao ponto 4 e, havendo percorrido 1/4 do caminho, se detém e deixa a quarta pedra. Deste modo segue até que, depois de deixar a sétima pedra, caminha em direção ao ponto 8, e havendo percorrido 1/8 do caminho, deixa a oitava pedra. A quantidade de pedras que ficarem no centro do octógono depende da ordem em que ele escreveu os números nos vértices. Qual é a maior quantidade de pedras que podem ficar no centro?

EUREKA! N° 2, 1998

19


Sociedade Brasileira de Matemática

PROBLEMA 5

O planeta X31 tem só dois tipos de notas, mas o sistema não é tão mau já que só há quinze preços inteiros para os quais o pagamento não pode ser feito de forma exata (nesses casos deve-se pagar a mais e receber o troco). Se 18 é um dos preços para os quais não se pode fazer pagamento exato, encontre o valor de cada tipo de nota.

RESPOSTAS IV OLIMPÍADA DE MAIO Primeiro nível 1998 PROBLEMA 1: 16 formas. PROBLEMA 2 : 9 PROBLEMA 3: 15h PROBLEMA 4 : As áreas são iguais. PROBLEMA 5 : Há muitas soluções.

IV OLIMPÍADA DE MAIO Segundo nível 1998 PROBLEMA 1 : 5, 7, 8, e 9 PROBLEMA 2 : 6/49 PROBLEMA 3: 70 maneiras. PROBLEMA 4 : 4 pedras. PROBLEMA 5: 4 e 11

Você sabia…

Que a todo momento há dois

pontos antípodas na terra com a mesma temperatura e a mesma pressão (admitindo que temperatura e pressão dependem continuamente do ponto)??

EUREKA! N° 2, 1998

20


Sociedade Brasileira de Matemática

9a. OLIMPÍADA DE MATEMÁTICA DO CONE SUL A 9ª. Olimpíada de Matemática do Cone Sul foi realizada em Salvador, BA, no período de 13 a 21 de junho de 1998. Esta Olimpíada foi realizada pela segunda vez no país (a primeira foi em 1993, em Petrópolis, RJ). Dela participaram alunos de até 15 anos dos seguintes países: Argentina, Brasil, Bolívia, Chile, Paraguai, Peru e Uruguai. A organização da Olimpíada esteve a cargo da Professora Luzinalva Amorim, da Universidade Federal da Bahia. A equipe brasileira foi selecionada através de provas realizadas em março e maio deste ano e foi liderada pelos professores Paulo Cezar Pinto Carvalho, do IMPA, e Florêncio Ferreira Guimarães, da UFES. A competição constou de duas provas, realizadas em dois dias, cada uma com três problemas, valendo 10 pontos cada. Veja a seguir os resultados obtidos pela equipe brasileira e as provas da 9a. Olimpíada de Matemática do Cone Sul.

RESULTADOS DA EQUIPE BRASILEIRA BRA 1 BRA 2 BRA 3 BRA 4

Mila Lopes Viana Pedro Paulo Gouveia Fabricio Siqueira Benevides Jônathas Diógenes Castelo Branco

Você sabi@

que a

Olimpíada Brasileira de Matemática tem página web?? Visite-nos no endereço eletrônico

http://www.obm.org.br

EUREKA! N° 2, 1998

21

Bronze Prata Prata Bronze


Sociedade Brasileira de Matemática

9a. OLIMPÍADA DE MATEMÁTICA DO CONE SUL Problemas e soluções Primeiro dia. Tempo: 4 horas 30 min. PROBLEMA 1

São dados 98 cartões. Em cada um deles está escrito um dos números 1, 2, 3, …, 98 (não existem números repetidos). Pode-se ordenar os 98 cartões de tal modo que ao considerar dois cartões consecutivos a diferença entre o número maior e o número menor escritos neles seja sempre maior que 48. Indicar como e de quantas formas é possível efetuar a ordenação. PROBLEMA 2

Sejam H o ortocentro (interseção das alturas) do triângulo acutângulo ABC e M o ponto médio do lado BC. Seja X o ponto em que a reta HM intersecta o arco BC (que não contém A) da circunsferência circunscrita a ABC. Seja Y o ponto de interseção da reta BH com a circunsferência, distinto de B. Demonstre que XY = BC. PROBLEMA 3

Prove que, pelo menos para 30% dos naturais n entre 1 e 1.000.000, o primeiro dígito de 2n é 1.

Segundo dia. Tempo: 4 horas 30 minutos. PROBLEMA 4

Determine todas as funções f tais que

f ( x 2 ) − f ( y 2 ) + 2 x + 1 = f ( x + y) ⋅ f ( x − y) quaisquer que sejam os números reais x, y.

EUREKA! N° 2, 1998

22


Sociedade Brasileira de Matemática

PROBLEMA 5

Em Terra Brasilis existem n casas onde vivem n duendes, cada um em uma casa. Existem estradas de mão única de tal modo que:

• • •

cada estrada liga duas casas; em cada casa começa exatamente uma estrada; em cada casa termina exatamente uma estrada.

Todos os dias, a partir do dia 1, cada duende sai da casa onde está e chega à casa vizinha. Uma lenda de Terra Brasilis diz que, quando todos os duendes regressarem à posição original, o mundo acabará. (a) (b)

Demonstre que o mundo acabará. Se n = 98, demonstre que é possível que os duendes construam e orientem as estradas de modo que o mundo não se acabe antes de 300.000 anos.

PROBLEMA 6

O Prefeito de uma cidade deseja estabelecer um sistema de transportes com pelo menos uma linha de ônibus, no qual: (i) (ii) (iii)

cada linha passe exatamente por três paradas; cada duas linhas distintas tenham exatamente uma parada em comum; para cada duas paradas de ônibus distintas exista exatamente uma linha que passe por ambas.

Determine o número de paradas de ônibus da cidade.

EUREKA! N° 2, 1998

23


Sociedade Brasileira de Matemática

SOLUÇÕES 1)

Vamos provar uma versão um pouco mais geral do problema: Seja k um número natural. Encontrar todas as permutações a1, a2, …a2k dos números 1, 2, …, 2k que verificam ai − ai +1 ≥ k para todo i = 1, 2, …, 2k –1.

Solução Em primeiro lugar observamos que, se dois números entre 1, 2, …, 2k diferem de pelo menos k, então o maior dos números está entre k + 1, k + 2, … 2k e o menor, entre 1, 2, …, k . Chamemos simplesmente os números destes dois conjuntos de "grandes" e "pequenos", respectivamente. Suponhamos que a1, a2, …a2k é uma permutação com a propriedade em questão. Pelo que dissemos acima, seus termos com índice ímpar (par) devem ser todos grandes ou todos pequenos. Sejam, por exemplo, a1, a3, …a2k – 1 pequenos e a2, a4, …a2k grandes. Consideremos a soma

S = a1 − a 2 + a 2 + a3 + ... + a 2 k − 2 − a 2 k −1 + a 2 k −1 − a 2 k Como cada termo de índice par é maior do que seus vizinhos,

S = (a 2 − a1 ) + (a 2 − a3 ) + ... + (a 2 k − 2 − a 2 k −1) + (a 2 k − a 2 k −1 ) = 2(a 2 + a 4 + ... + a 2 k ) − 2(a1 + a3 + ...a 2 k −1 ) − a 2 k + a1 = 2((k + 1) + (k + 2) + ... + 2k ) − 2(1 + 2 + ... + k ) − a 2 k + a1

= 2k 2 − (a 2 k − a1 ) Notemos que a condição ai − ai +1 ≥ k determina a escolha de a1 e a2k . Os únicos vizinhos possíveis de k e k +1 são 2k e 1, respectivamente. Logo k e k+1 devem ser o primeiro e o último termos da permutação. E como escolhemos começar com a1 pequeno, a1 = k, a2k = k +1. Então a2 = 2k, a2k –1 = 1. EUREKA! N° 2, 1998

24


Sociedade Brasileira de Matemática

Regressando à soma S, vemos que ela é igual a 2k2 – (( k +1) – k ) = 2k2 – 1. Por outro lado, cada dois somandos da forma ai −1 − ai + ai − ai +1 contribui com pelo menos k + ( k + 1) = 2k + 1. Isto se deve ao fato de ser impossível que

ai −1 − ai = ai − ai +1 = k , pois

teríamos, neste caso, ai – 1 = ai + 1. Assim, temos

S ≥ (2k + 1) + (2k + 1) + ... + (2k + 1) + k = 2k 2 − 1

k −1

Então ai −1 − a i + a i − a i +1 = 2k + 1 para todo i = 1, 3, … 2k – 1. Isto é, as diferenças consecutivas são k, k + 1, k, k + 1, …, k. Começando com a1 = k, a2 = 2k (que diferem em k ), podemos determinar todos os ai da seqüência:

a 3 = 2k − ( k + 1) = k − 1 a 4 = ( k − 1) + k = 2k − 1 a 5 = ( 2k − 1) − ( k + 1) = k − 2

# Portanto a1, a2 , …, a2k é

k, 2k, k – 1, 2k – 1, k – 2, …, 2, k + 2, 1, k + 1 Por simetria, existe exatamente uma solução além desta: a que obtemos tomando a solução acima na ordem inversa.

EUREKA! N° 2, 1998

25


Sociedade Brasileira de Matemática

2)

Seja {L} = PM ∩ AH. Mostraremos que L coincide com H. Inicialmente, observe que ∠ NPA = 90° (pois AM é diâmetro). Prolonguemos PM até encontrar a circunsferência circunscrita no ponto N, diametralmente oposto ao vértice A (pois ∠ NPA = 90°.) Logo o circuncentro O é o ponto médio de AN e, como OM  AL, segue que M é o ponto médio de LN; como m é ponto médio de BC , segue que LBNC é um paralelogramo, de modo que BL  NC . Mas ∠ NCA = 90° (pois AN é diâmetro), ou seja, NC ⊥ AC. Daí segue que BL ⊥ AC e, como AL ⊥ BC, concluimos que L ≡ H. 3)

Vamos provar que para cada inteiro positivo k existe uma potência de 2 com exatamente k dígitos (na base 10) e cujo primeiro dígito é 1. De fato, se considerarmos a menor potência de 2 maior que 10k + 1, devemos ter 2 n < 10 k + 1 ≤ 2n + 1, ou 10 k + 1 ≤ 2n + 1 < 2 × 10 k + 1 . 6

Portanto basta calcular quantos dígitos possui 210 . Mas, de 103 < 210, 5

6

6

obtemos 10 3×10 < 210 , donde segue que 210 tem mais de 300.000 algarismos e segue que no mínimo 300.000/1.000.000 = 30% de tais potências começam com o algarismo 1.

EUREKA! N° 2, 1998

26


Sociedade Brasileira de Matemática

Observações: 1. Utilizamos somente que existe uma potência de 2 que começa com o dígito 1 e possui exatamente k dígitos. Como verifica-se imediatamente, existe exatamente uma potência de 2 com k dígitos que começa com 1. 2.

6

6

210 possui exatamente 301.030 algarismos, pois se 10t < 210 < 10t+1,

aplicando logaritmos, vem t < 106 log 2 < t + 1, donde t + 1 = 301.030.

3. Utilizando as idéias de 1 e 2, é possível mostrar que a probabilidade de uma potência de 2 começar com o algarismo 1 é log 2. Mais precisamente, se f (n) é o número de inteiros k (1 ≤ k ≤ n ) tais que 2k que iniciam com o algarismo 1, então

f ( n) lim = log 2 ≈ 0,301029995664. n→∞ n 4)

Fazendo x = y temos

f (2x) ⋅ f (0)= 2x + 1 Logo, para x = 0, (f (0))2 = 1 ⇔ f (0) = ± 1 Assim, f (2x) = ± (2x +1) e, portanto, f (x) = x + 1 ou f (x) = – (x + 1). Substituindo as funções encontradas na equação funcional original, verificamos que apenas f (x) = x + 1 satisfaz as condições do problema. 5)

(a)

Numere os duendes de 1 a n e seja f(i) o vizinho do duende número i. A função f é claramente uma bijeção. Em algum momento cada duende retornará a sua casa pois a seqüência f (i), f ( f (i) ), f ( f ( f (i))),…assume um número finito de valores, donde existirão inteiros positivos r < s tais que f s (i) = f r(i), portanto f s – r (i) = i (pois f é bijetora). Seja g(i) o menor inteiro positivo tal que o duende i retorna à sua casa depois de g(i) dias. Depois de

EUREKA! N° 2, 1998

27


Sociedade Brasileira de Matemática

mmc(g(1), g (2),…,g(n)) dias, todos os duendes retornarão à posição original e o mundo acabará. (b)

Divida os 98 duendes em 8 ciclos de tamanhos 3, 5, 7, 11, 13, 17, 19, 23 (98 = 3 + 5 + 7 + 11 + 13 + 17 + 19 + 23). Os duendes retornarão à posição inicial depois de 3 × 5 × 7 × 11 × 13 × 17 × 19 × 23 = 111546435 > 366 × 300.000. Alternativamente, podemos dividir os duendes em ciclos de tamanhos 3, 8, 9, 5, 7, 11, 13, 19 e 23, e eles retornarão à posição original em mmc (3, 8, 9, 5, 7, 11, 13, 19, 23) = 8 × 9 × 5 × 7 × 11 × 13 × 19 × 23 = 157.477.320

6)

Um exemplo de tal sistema é aquele que tem uma só linha com exatamente 3 pontos. Para o que segue, suponhamos que haja pelo menos 4 pontos 1, 2, 3, 4 e que uma das linhas é R1 = 123 (aqui, e no que segue, R = abc significa que a linha R passa pelos pontos a, b, c, não importando a ordem. Assim, por exemplo, R = bca é a mesma linha.) Por (iii), devem existir linhas R2, R3 e R4 que passam pelos pares de pontos {1, 4}, {2, 4} e {3, 4}, respectivamente. Notemos que R2 , R3 e R4 devem ser distintas. De fato, se, digamos, os pares {2, 4} e {3, 4} estão na mesma linha R2 , então R2 = 234, logo R1 e R2 têm duas paradas em comum e isto é impossível por (ii). Novamente por (ii), cada uma das linhas R2 , R3 e R4 tem exatamente um ponto em comum com R1 = 123. Como não podem haver dois pontos entre 1, 2, 3, que estão em R2 , R3 e R4 (novamente por (ii)), devemos ter R2 =14a, R3 = 24b, R4 = 34c para pontos distintos a, b, c que são por sua vez distintos de 1, 2, 3, 4. Para manter uma notação consistente, sejam a = 5, b = 6 e c = 7. Logo R2 = 145, R3 = 246 e R4 = 347. Com isso, provamos que há pelo menos 7 pontos. Agora, suponhamos que exista pelo menos um ponto a mais, digamos 8. Por (iii), existe uma linha S que passa por 1 e 8. Como S tem uma parada em comum com R3 = 246, concluímos que S = 128, S = 148 ou S = 168. Nenhuma destas é possível, pois 1. 2.

as linhas 128 e 148 têm dois pontos em comum com R1 = 123 e R2 = 145, respectivamente. 168 não tem ponto em comum com R4 = 347.

EUREKA! N° 2, 1998

28


Sociedade Brasileira de Matemática

Esta contradição é devida a termos suposto que existem mais de 7 pontos. Completamos a construção do sistema de transportes com 7 pontos de ônibus. Devem haver linhas R5, R6, R7 por {1, 6}, {2, 5}, {3, 5}, respectivamente (pois as linhas não estão entre as já existentes R1 , R2 , R2 , R3 e R4). Pode-se verificar que a escolha R5 = 167, R6 = 257 e R7 = 356 funciona. As 7 linhas 123, 145, 246, 347, 167, 257, 356 formam um exemplo de tal sistema. Concluímos, então, que a cidade pode ter exatamente 3 ou exatamente 7 pontos de ônibus.

Observação: Este problema é equivalente a particionar as arestas de um grafo completo Kn em triângulos de modo que quaisquer dois triângulos tenham exatamente um vértice em comum. Observando que, satisfeitas as condições do problema, cada vértice de um triângulo é comum a (n – 3)/2 outros triângulos, o total de triângulos em Kn é n − 3 1 n( n − 1) , o que só se verifica quando n = 3 ou n = 7. 1+ 3⋅ = 2 3 2 Para concluir a resolução, basta obter as partições nestes casos.

EUREKA! N° 2, 1998

29


Sociedade Brasileira de Matemática

39a. OLIMPÍADA INTERNACIONAL DE MATEMÁTICA Resultados e problemas

No mês de tantas expectativas dos brasileiros, o Brasil consegue uma medalha de ouro na 39a. Olimpíada Internacional de Matemática realizada com a presença de 76 países em Taiwan nos dias 10 a 21 de julho último. O estudante Rui Lopes Viana Filho (SP) foi ganhador de uma medalha de ouro. Também foram premiados os estudantes Emanuel Carneiro (CE) medalha de bronze, Murali Vajapeyam (PB) menção honrosa e Mauricio Carrari (SP) menção honrosa. Trata-se de feito muito importante, visto que países como Alemanha, Inglaterra, Israel, Suécia, Australia e muitos outros não conquistaram medalhas de ouro. Merece também elogios o fato da equipe brasileira ter sido uma das que tiveram melhor desempenho na questão 5 da prova, superando por exemplo, as equipes dos EUA e da Rússia. Veja a seguir as questões da 39a.Olimpíada Internacional de Matemática.

Primeiro dia Duração da Prova: 4 horas 30 min. PROBLEMA 1

No quadrilátero convexo ABCD, as diagonais AC e BD são perpendiculares e os lados opostos AB e DC não são paralelos. Sabemos que o ponto P, onde se intersectam as mediatrizes de AB e DC, está no interior de ABCD. Prove que ABCD é um quadrilátero inscritível se, e somente se, os triângulos ABP e CDP têm áreas iguais. PROBLEMA 2

Numa competição, existem a concorrentes e b juízes, onde b ≥ 3 é um inteiro ímpar. Cada juiz avalia cada um dos concorrentes, classificando-o como "aprovado" ou "reprovado". Suponha que k é um número tal que as EUREKA! N° 2, 1998

30


Sociedade Brasileira de Matemática

classificações dadas por dois juízes quaisquer coincidem no máximo para k k b −1 . concorrentes. Prove que ≥ a 2b PROBLEMA 3

Para qualquer inteiro positivo n, seja d(n) o número de divisores positivos de n (incluindo 1 e n). Determine todos os inteiros positivos k tais que

d (n 2 ) = k para algum n. d ( n)

Segundo dia Duração da Prova: 4 horas 30 min. PROBLEMA 4

Determine todos os pares (a, b) de inteiros positivos tais que ab2 + b + 7 divide a2b + a + b. PROBLEMA 5

Seja I o incentro do triângulo ABC. A circunferência inscrita no triângulo ABC é tangente aos lados BC, CA e AB nos pontos K, L e M, respectivamente. A reta que passa por B, paralela ao segmento MK, intersecta as retas LM e LK nos pontos R e S, respectivamente. Prove que o ângulo ∠RIS é agudo. PROBLEMA 6

Considere todas as funções f definidas no conjunto N dos inteiros positivos, com valores no mesmo conjunto, que satisfazem f (t 2 f ( s )) = s ( f (t )) 2 , para todos s e t em N. Determine o menor valor possível de f(1998)

EUREKA! N° 2, 1998

31


Sociedade Brasileira de Matemática

PARIDADE Eduardo Wagner ♦ Nível Iniciante. Todo número natural é par ou ímpar. Elementar, não? A afirmação acima, que é uma das mais simples e óbvias da Matemática, é também uma ferramenta de grande utilidade na resolução de muitos problemas envolvendo números naturais. Vamos comentar neste artigo alguns deles, em graus diferentes de dificuldade, mas inicialmente precisamos recordar três importantes propriedades: a) a soma de dois números pares é par. b) a soma de dois números ímpares é par. c) a soma de um número par com um número ímpar é ímpar. Dizemos que dois números inteiros têm mesma paridade, quando são ambos pares ou ambos ímpares. Assim, podemos dizer que a soma de dois números inteiros é par se, e somente se, eles têm mesma paridade. Vamos aos problemas. PROBLEMA 1

Em um quartel existem 100 soldados e, todas as noites, três deles são escolhidos para trabalhar de sentinela. É possível que após certo tempo um dos soldados tenha trabalhado com cada um dos outros exatamente uma vez? RESPOSTA : Não.

Escolha um soldado. Em cada noite em que trabalha, ele está em companhia de dois outros. Como 99 é um número ímpar, não podemos formar pares de soldados sempre diferentes para trabalhar com o escolhido. PROBLEMA 2

Um jogo consiste de 9 botões luminosos (de cor verde ou vermelha) dispostos da seguinte forma:

EUREKA! N° 2, 1998

32


Sociedade Brasileira de Matemática

1

2

3

4

5

6

7

8

9

Apertando um botão do bordo do retângulo, trocam de cor ele e seus vizinhos (do lado ou em diagonal). Apertando o botão do centro, trocam de cor todos os seus 8 vizinhos porém ele não. Exemplos: Apertando 1, trocam de cor 1, 2, 4 e 5. Apertando 2, trocam de cor 1, 2, 3, 4, 5 e 6. Apertando 5, trocam de cor 1, 2, 3, 4, 6, 7, 8 e 9. Inicialmente todos os botões estão verdes. É possível, apertando sucessivamente alguns botões, torná-los todos vermelhos? RESPOSTA : Não é possível.

Observe que apertando um botão do vértice do retângulo, trocam de cor 4 botões. Apertando um botão do meio de um lado, trocam de cor 6 botões e apertando um botão do centro trocam de cor 8 botões. Assim, cada vez que apertamos um botão trocam de cor um número par de botões. Como existem 9 botões, não é possível que todos troquem de cor. PROBLEMA 3

Escrevemos abaixo os números naturais de 1 a 10. 1

2

3

4

5

6

7

8

9

10.

Antes de cada um deles, coloque sinais “+” ou “–” de forma que a soma de todos seja zero. SOLUÇÃO: Não é possível fazer isto. Imaginando que fosse possível, deveríamos separar os números dados em dois grupos com a mesma soma. Então colocaríamos sinais negativos nos EUREKA! N° 2, 1998

33


Sociedade Brasileira de Matemática

números de um dos grupos e sinais positivos nos números do outro. Teríamos então uma soma igual a zero. Acontece que a soma dos números naturais de 1 a 10 é igual a 55. Como este número é ímpar, não podemos separar os números dados em dois grupos que tenham a mesma soma. Como o leitor deve estar percebendo, os argumentos utilizados permitiram concluir que as respostas dos três problemas propostos foram iguais: “não é possível fazer tal coisa”. Na maioria das vezes, um argumento de paridade serve exatamente para isto. Mostrar que um determinado fato não pode ocorrer e isto não é desanimador, muito pelo contrário. Serve para nos convencer que não adianta ficar gastando tempo demais fazendo tentativas inúteis. As experiências são valiosas no sentido de nos abrir os olhos para a possibilidade do problema não ter solução e, a partir daí, buscar um argumento que resolva definitivamente a questão. É muito importante também explorar um problema, ou seja, imaginar pequenas modificações no enunciado e verificar o que ocorre com sua resposta. Por exemplo, o problema 3 não tem solução porque a soma dos naturais de 1 até 10 é 55 (ímpar). O que ocorreria se a soma fosse par? Este é um novo e atrativo problema. Vamos enunciá-lo: PROBLEMA 3A: Escrevemos abaixo os números naturais de 1 a 11.

1

2

3

4

5

6

7

8

9

10

11

Antes de cada um deles, coloque sinais “+” ou “–” de forma que a soma de todos seja zero. SOLUÇÃO: A soma dos números naturais de 1 a 11 é 66. Como podemos separá-los em dois grupos de soma 33? Começando pelos maiores observe que 11 + 10 + 9 = 30. Logo, 11 + 10 + 9 + 3 = 33. O problema 3A tem como uma solução possível:

+1 + 2 – 3 + 4 + 5 + 6 + 7 + 8 – 9 – 10 – 11 = 0

EUREKA! N° 2, 1998

34


Sociedade Brasileira de Matemática

Fica ao encargo do leitor mostrar que sempre que a soma dos naturais de 1 até n é par então podemos separá-los em dois grupos de igual soma. Você pode utilizar o caminho que utilizamos acima, ou buscar uma outra forma. Para saber mais e intrigar seus colegas

Você pode propor aos seus amigos os problemas 3 ou 3A com uma lista grande de números naturais consecutivos. O problema terá ou não solução caso a soma desses números seja par ou ímpar, respectivamente. Entretanto, é possível encontrar o resultado desta soma rapidamente, sem precisar somar todas as parcelas. A soma de todos os naturais de 1 até n é (1 + n) n . Por exemplo, a soma de todos os naturais de 1 até 10 é igual a 2 (1 + 10)10 11 ⋅ 10 = = 55 . Procure demonstrar este fato e, se não conseguir, 2 2 pergunte ao seu professor ou escreva para a EUREKA!

PROBLEMA 4 2

Mostre que se a, b e c são inteiros ímpares, a equação ax + bx + c = 0 não tem raiz racional. Comentários: 1) Um número é raiz de uma equação dada se quando for substituído no 2 é raiz (ou lugar do “x” a igualdade ficar correta. Por exemplo, x = 3 2 solução) da equação 3 x − 2 = 0 porque 3 ⋅ − 2 = 0 . Ainda, x = 2 é 3 4 3 solução da equação x − x + x − 10 = 0 porque 2 4 − 2 3 + 2 − 10 = 0 . Freqüentemente não sabemos como resolver uma equação mas, em geral, podemos verificar se um certo valor de x é ou não uma de suas raízes.

EUREKA! N° 2, 1998

35


Sociedade Brasileira de Matemática

2) Um número é racional quando puder ser escrito como uma fração de 2 4 e são exemplos de numerador e denominador inteiros. Por exemplo, 7 1 números racionais. 3) Quando desejamos demonstrar que certo fato é impossível utilizamos freqüentemente o método da redução ao absurdo. Este método consiste em imaginar o contrário, ou seja, que tal fato seja possível. A partir daí procuramos chegar a uma contradição, a um absurdo. Conseguindo isso, teremos mostrado que nossa hipótese (a do contrário) é falsa e conseqüentemente, que a afirmação inicial é verdadeira. Vamos ver tudo isso na solução do problema. Não se preocupe se você ainda não sabe resolver uma equação do segundo grau. Isto não será necessário. Tudo o que precisamos é verificar se um número racional pode ser uma raiz.

Solução do problema 4 Imaginemos que o número racional

p q

seja raiz da equação

ax 2 + bx + c = 0 onde a, b e c são inteiros ímpares. Logo, fazendo a substituição, devemos ter, 2

 p  p a  + b  + c = 0 q q a

p2 p +b +c = 0 2 q q

ap 2 + bpq + cq 2 = 0 Vamos acrescentar agora uma hipótese importante para facilitar nosso p trabalho. Vamos supor que a nossa fração seja irredutível, ou seja, que q EUREKA! N° 2, 1998

36


Sociedade Brasileira de Matemática

ela já foi simplificada ao máximo. Por exemplo, no lugar de considerando

4 6

estaremos

2

o que é a mesma coisa. Consideramos então, para a 3 solução do problema, que p e q não são ambos pares. 2

2

Observe agora a equação ap + bpq + cq = 0 nos seguintes casos: 2

2

a) p e q são ímpares: neste caso, ap é ímpar, bpq é ímpar e cq é ímpar. Como a soma de três números ímpares é ímpar, o resultado não pode ser zero. 2

2

b) p é par e q é ímpar: neste caso, ap é par, bpq é par e cq é ímpar. Como a soma de dois números pares e um ímpar é ímpar, o resultado não pode ser zero. c) p é ímpar e q é par: vale o mesmo argumento do caso b). Demonstramos então que nenhuma fração de numerador e denominador inteiros pode ser raiz da equação ax 2 + bx + c = 0 onde a, b e c são inteiros ímpares. PROBLEMA 5

Um tabuleiro 6 × 6 está coberto com dominós 2 × 1. Mostre que existe uma reta que separa as peças do tabuleiro sem cortar nenhum dominó. SOLUÇÃO: Cada dominó é formado por dois quadrados e portanto, se o tabuleiro está inteiramente coberto, 18 dominós foram utilizados. Imagine agora uma reta (horizontal, por exemplo) que separe o tabuleiro em duas partes. Se ela não corta nenhum dominó, está resolvido o problema. Suponha então que ela corte ao meio um dominó. Neste caso, acima desta reta teremos n dominós inteiros mais meio dominó, ou seja, teremos acima desta reta 2n + 1 quadrados, que é um número ímpar. Mas isto é impossível porque se o tabuleiro tem 6 unidades de largura, qualquer reta o dividirá em partes que contém números pares de quadrados acima e abaixo dela. Assim, se uma reta corta um dominó, deverá cortar um outro dominó. Para a divisão do EUREKA! N° 2, 1998

37


Sociedade Brasileira de Matemática

tabuleiro, existem 10 retas possíveis e, se cada uma delas cortar dois dominós, deveríamos ter 20 dominós no tabuleiro. Como eles são apenas 18 então existe uma reta (pelo menos) que não corta nenhum dominó.

Problemas para pesquisa PROBLEMA 6

Os números naturais de 1 até 1998 são escritos em um imenso quadro negro. Em seguida, um aluno apaga dois quaisquer colocando no lugar sua diferença (não negativa). Depois de muitas operações, um único número ficará escrito no quadro. É possível que esse número seja zero?

PROBLEMA 7

Em uma ilha plana existem 11 cidades numeradas de 1 a 11. Estradas retas ligam 1 a 2, 2 a 3, 3 a 4, ..., 10 a 11 e 11 a 1. É possível que uma reta corte todas as estradas?

EUREKA! N° 2, 1998

38


Sociedade Brasileira de Matemática

OS PROBLEMAS DO VISITANTE MATEMÁTICO The Mathematical Visitor foi um periódico que existiu nos Estados Unidos entre 1877 e 1896. Era uma revista destinada aos amantes da arte de resolver problemas de Matemática. Publicava problemas propostos pelo seu abnegado editor ou leitores e, em números subseqüentes, trazia as melhores soluções apresentadas. Procurava fortalecer entre os norteamericanos, na época em que sua nação lutava para se inserir no rol dos países mais desenvolvidos, uma tradição há muito existente na Europa: a prática das saudáveis competições matemáticas públicas, instituídas por revistas como a famosa Ladies Diary, da Inglaterra. Os problemas do The Mathematical Visitor eram, em sua grande maioria, de nível elementar, embora alguns deles exigissem o uso de integrais em sua resolução. Quanto à criatividade e à elegância das questões propostas, a qualidade variava bastante. Num período em que faltavam calculadoras eletrônicas e sobrava lazer para as pessoas, eram muito freqüentes problemas cuja solução requeria muito mais paciência e tempo disponível de que engenhosidade e talento. Um exemplo de problemas desse tipo é o seguinte, que foi proposto em 1887: 1)

Considere a seqüência dos triângulos pitagóricos (triângulos retângulos de lados inteiros) nos quais os catetos são inteiros consecutivos. Ache a expressão geral para os lados n-ésimo triângulo e calcule explicitamente os lados do centésimo. (A resposta da segunda parte envolve números com 76 algarismos.) Outros problemas computacionais são: 4

2)

Calcular 4 4 .

3)

Obter a raiz cúbica de 2 com 100 algarismos decimais!

Mas não se pense que The Mathematical Visitor só trazia perguntas sem graça. Alguns problemas bem elementares lá propostos ainda guardam interesse a são apresentados aqui como desafio aos nossos leitores. EUREKA! N° 2, 1998

39


Sociedade Brasileira de Matemática

4)

Com apenas dois cortes retilíneos e recolagem, transforme um retângulo num quadrado de mesma área, supondo que a razão entre o maior e o menor lado do retângulo é menor do que ou igual a 4.

5)

Comprei na feira um queijo que pesou 9 quilos. Desconfiei da pesagem e o vendedor propôs, como compensação, vender-me um queijo igual, desta vez pesado no outro prato da balança. O peso foi de 4 quilos. Ganhei ou perdi na transação? Qual é o verdadeiro peso do queijo?

6)

Ache três números inteiros cuja soma é um cubo e a soma de dois quaisquer deles também é um cubo.

7)

O doutor A mata 3 pacientes em cada 7 que trata; o doutor B mata 4 em cada 13 e o doutor C mata 5 em cada 19. Qual é a probabilidade de um doente sobreviver se for tratado por esses 3 médicos ao mesmo tempo?

8)

Ache quatro inteiros que são quadrados e a soma de dois quaisquer deles ainda é um quadrado. (Observação: nem a redação da revista nem o autor do problema sabiam como resolvê-lo.) Um dos problemas mais interessantes, propostos em 1881, foi o seguinte:

9)

Um vaso de vinho está suspenso sobre outro, de igual capacidade (digamos 1 litro), cheio de água. Por um orifício no fundo de cada, o vinho escorre sobre o vaso de água e a mistura se esvai na mesma velocidade. Quando o vaso de vinho estiver vazio, qual é o volume de água no vaso inferior?

A coleção de problemas do The Mathematical Visitor foi reeditada em 1996 pela Math Pro Press, Westford, Mass., sob o título "Problems and Solutions from The Mathematical Visitor". Nossa revista aguarda respostas de nossos leitores para os problemas acima propostos, especialmente os de números 4, 5 e 9.

EUREKA! N° 2, 1998

40


Sociedade Brasileira de Matemática

DIVISIBILIDADE, CONGRUÊNCIAS E ARITMÉTICA MÓDULO n Carlos Gustavo Moreira ♦ Nível Avançado INTRODUÇÃO

Este artigo se propõe a ser uma referência sobre os temas citados no título, que aparecem naturalmente em diversos problemas de Matemática elementar, alguns dos quais serão explicitamente tratados aqui. O estilo é mais conciso do que a maioria dos outros artigos desta revista, o que pode tornar a leitura mais difícil, mas não desanime! Procure entender os enunciados das proposições e os problemas resolvidos e buscar sua propria solução para eles, além de pensar nos problemas propostos e enviar-nos suas soluções. Em caso de qualquer dúvida não deixe de escrever-nos.

Seção 1: Divisão euclidiana e o teorema fundamental da aritmética Os resultados que seguem têm como base o seguinte fato sobre os inteiros: Dados a ∈ Z, b ∈ N* existem q, r ∈ Z com 0 ≤ r < b e a = bq + r. Tais q e r estão unicamente determinados. De fato, q = [a/b] e r = a – bq (aqui [x] denota o único inteiro k tal que k ≤ x < k + 1). Como conseqüência temos a Proposição 0 (Divisão Euclidiana): Dados a ∈ Z, b ∈ Z* existem q, r ∈ Z unicamente determinados tais que 0 ≤ r < be a = bq + r

Definição: Dados dois inteiros a e b , com a ≠ 0 dizemos que a divide b (denotamos ab) se existe c inteiro tal que b = ac. Proposição 1: Dados a, b ∈ Z não ambos nulos existe d ∈ N* tal que da, db e, para todo c ∈ N*, ca, cb ⇒ cd. Além disso existem x, y ∈ Z com d = ax + by. (Esse d é chamado o máximo divisor comum entre a e b : d = mdc (a, b). )

EUREKA! N° 2, 1998

41


Sociedade Brasileira de Matemática

Demonstração: Seja A = {k > 0∃ x, y ∈ Z tais que k = ax + by} e seja d = ax0 + by0 o menor elemento de A. Mostraremos que da. Como d ∈ N*, existem q, r ∈ Z com a = dq + r e 0 ≤ r < d. Queremos mostrar que r = 0. De fato, se r > 0, r = a – dq = a (1 – qx0) + b(– qy0) ∈ A, contradizendo o fato de d ser o menor elemento de A. Portanto, r = 0 e a = dq ⇒ da. Do mesmo modo prova-se que db. Suponha agora que ca e cb. Então cax0 + by0 = d, como queríamos provar ❏ Lema: Se mdc (q, n) = 1 e nqk então nk. Prova do Lema: Como mdc(q, n) = 1, existem x, y ∈ Z com qx + ny = 1, logo qkx + nky = k, portanto nk (pois qkx e nnky)

Corolário: Sejam p um número primo e a, b ∈ Z. Se pab então pa ou pb

Teorema fundamental da aritmética: Todo número natural n ≥ 2 possui uma única fatoração (a menos da ordem dos fatores), como produto de primos. Demonstração: n = 2 é primo. Vamos mostrar a existencia da fatoração por primos por indução: Se n é primo não há o que provar. Se n é composto, n = ab, a, b ∈ N, a < n, b < n e, por hipótese de indução, a e b se decompõem como produto de primos, portanto n se decompõe como produto de primos. Vamos agora mostrar a unicidade, também por indução: Suponha que n admita duas fatorações n = p1p2…pr e n = q1q2…qs como produto de primos. O Corolário acima mostra que, como p1q1q2…qr, p1 deve dividir algum qi e portanto p1 = qi (pois são ambos números primos) e, como n/p1 = n/qi < n admite uma única fatoração prima, por hipótese de indução, concluímos que a fatoração de n é única ❏ Proposição 2: O conjunto dos números primos é infinito.

EUREKA! N° 2, 1998

42


Sociedade Brasileira de Matemática

Demonstração: Suponha que o conjunto dos números primos seja finito, digamos { p1, p2,…, pn}. Nesse caso, o número N = p1p2…pn +1 seria maior que todos os primos, mas não divisível por nenhum deles, pois pi( p1p2…pn + 1) ⇒ pi 1, absurdo. Teríamos então um natural N > 2 que não seria múltiplo de nenhum primo, contradizendo o teorema fundamental da aritmética ❏ Obs.: As idéias desta seção podem ser utilizadas em situações mais gerais, como no estudo de polinômios (por exemplo com coeficientes racionais), onde existe um algoritmo de divisão, a partir do qual pode-se provar de modo análogo resultados correspondentes aos aqui apresentados sobre máximo divisor comum, existência e unicidade de fatoração.

Seção 2: Congruências Definição: Sejam a, b, n ∈ Z, n > 0. Dizemos que a é congruente a b (módulo n) (denota-se a ≡ b (módulo n)) se n(b – a) Obs: a ≡ a (módulo n), a ≡ b (módulo n) ⇔ b ≡ a (módulo n), a ≡ b (módulo n), b ≡ c (módulo n) ⇒ a ≡ c (módulo n), ou seja, congruência (módulo n) é uma relação de equivalência. Proposição: Se a ≡ b (módulo n) e c ≡ d (módulo n) então a + c ≡ b + d (módulo n) e ac ≡ bd (módulo n). Demonstração: n(b – a), n (d – c) ⇒ n  (b + d) – (a + c) ⇒ (a + c) ≡ (b + d) (módulo n), e bd – ac = b(d – c) + ((b – a) ⇒ n(bd – ac) ⇒ bd ≡ ac (módulo n) ❏ Definição: Dados n, a ∈ Z n > 0, definimos a =a (módulo n) = = {k ∈ Zk ≡ a (módulo n)}. Dados a, b ∈ Z definimos a +b = a + b ea ⋅b = ab ( estas operações de soma e produto estão bem definidas pela proposição anterior). Definimos ainda Z/nZ = {a (módulo n), a ∈ Z}={0, 1, 2,… n − 1 }. Cada a é chamada uma classe de congruência módulo n.

EUREKA! N° 2, 1998

43


Sociedade Brasileira de Matemática

Definição: Sejam n, a ∈ Z, n > 0. Dizemos que a é invertível módulo n se existe b ∈ Z com ab ≡ 1(módulo n) (ou seja, tal que a ⋅b = 1). Dizemos que b é o inverso de a em Z/nZ. Definição: (Z/nZ)* ={a a ∈ Z e a é invertível (módulo n)}. Obs. a é invertível (módulo n) ⇔ mdc (a, n) =1. De fato, mdc (a, n) = 1 ⇔ ∃ x, y ∈ Z tais que ax + ny = 1⇔a ⋅x = 1 (módulo n). Notação: Dado um conjunto finito X, escrevemos # X para significar o número de elementos de X. Definição: A função ϕ de Euler, ϕ: N → N é definida por ϕ(n) = # (Z/nZ)* = # {k ∈ Z  0 ≤ k < n e mdc (k, n) = 1}. Notemos que se p é um número primo e k ∈ N então ϕ(pk) = pk – pk–1 =

pk (1–1/p). De fato, mdc (r, pk ) = 1 se e só se p não divide r. Logo ϕ(pk) = #{r ∈ Z0 ≤ r < pk e mdc (r, pk ) =1} = # {r ∈ Z 0 ≤ r < pk } – # {r ∈ Z 0 ≤ r < pk e pr} = pk – pk – 1. Definição: n números inteiros a1, a2,…an formam um sistema completo de resíduos (s.c.r.) módulo n se {a1 , a2,…, an}= Z/nZ isto é, se os a representam todas as classes de congruência módulo n ( por exemplo, 0,1,2,…n –1 formam um s.c.r. (módulo n)). ϕ(n) números inteiros b1, b2,…bϕ(n) formam um sistema completo de invertíveis (s.c.i.) módulo n se {b1, b2,…bϕ(n)} = (Z/nZ)*, isto é, se os bi representam todas as classes de congruências invertíveis módulo n. Proposição: Sejam q, r, n ∈ Z, n > 0, q invertível módulo n, a1, a2,…,an um s.c.r. (módulo n) e b1, b2,…,bϕ(n) um s.c.i. (módulo n). Então qa1 + r, qa2 + r,…, qan + r formam um s.c.r. (módulo n) e qb1, qb2,…qbϕ(n) formam um s.c.i. (módulo n).

EUREKA! N° 2, 1998

44


Sociedade Brasileira de Matemática

Demonstração: Vamos provar que se a1, …an formam um s.c.r. (módulo n) então qa1 + r, …qan + r formam um s.c.r. (módulo n). Basta provar que qai + r ≡ qaj + r (módulo n) ⇒ i = j, pois nesse caso teremos n classes de congruências distintas módulo n, que devem ser todas as classes de Z/nZ. Seja y ∈ Z tal que qy ≡ 1 (módulo n). Temos qai = qai + r – r ≡ qaj + r – r = qaj (módulo n) ⇒ qyai ≡ qyaj (módulo n) ⇒ ai ≡ aj (módulo n) ⇒ i = j. Seja agora b1, b2,…bϕ(n) um s.c.r. (módulo n). Temos que qbi é invertível módulo n. para todo i,1 ≤ i ≤ ϕ(n), pois se xi é tal que bi xi ≡ 1 (módulo n). então (qbi) (xiy) = (qy) (bixi) ≡ 1 (módulo n). Por outro lado, se qbi ≡ qbj (módulo n) então bi ≡ yqbi ≡ yqbj ≡ bj (módulo n) ⇒ i = j, e portanto qb1, qb2,…qbϕ(n) é um s.c.i. (módulo n) ❏

Teorema (Euler): Sejam a, n ∈ Z, n > 0, tais que mdc (a, n) = 1. Então aϕ(n) ≡1 (módulo n). Demonstração: Seja b1, b2,…bϕ(n) um s.c.i. (módulo n) Pela proposição anterior, (ab1), (ab2),…,(abϕ(n)) formam um s.c.i. (módulo n), e temos {b1, b2,…,bϕ(n)} = { ab1 , ab 2 ,… ab ϕ(n)} = (Z/nZ)*⇒b1 ⋅b2.…bϕ(n) = ab 1 ⋅ ab 2 ….abϕ(n) =aϕ(n) ⋅b1b2 …bϕ(n) ⇒ b1 ⋅b2 …bϕ(n) (aϕ(n) –1) = 0 ⇒aϕ(n) =1 pois b1, b2,…bϕ(n) são invertíveis (módulo n) ⇒ aϕ(n) ≡ 1 (módulo n) ❏

Corolário: (Pequeno Teorema de Fermat): Se a ∈ Z e p é primo então ap ≡ a (módulo p). Prova : Se pa, então ap ≡ a ≡ 0 (módulo p). Se p não divide a, então mdc (a, p) =1 ⇒ a (módulo p) ❏

p–1

≡ 1 (módulo p) ⇒ ap ≡ a

Exercício resolvido: Exiba n ∈ N tal que 2n tenha mais de duas mil casas decimais e tenha entre suas 2000 últimas casas decimais 1000 zeros consecutivos. Solução: 2ϕ ( 5

2000

)

≡ 1 (módulo 52000), pelo teorema de Euler. Portanto,

EUREKA! N° 2, 1998

45


Sociedade Brasileira de Matemática

existe b ∈ N com 2ϕ ( 5

2000

)

= 52000 b + 1, e teremos 2 2000 +ϕ ( 5

2000

)

= 102000 b +

2000

22000, e portanto os 2000 últimos dígitos de 2 2000+ϕ (5 ) coincidem com a representação decimal de 22000, que tem no máximo 667 dígitos, pois 23 < 10 ⇒ 22000 < 23.667 < 10667. Desta forma , 2

2000 +ϕ ( 52000 )

tem pelo menos 2000 – 667 = 1333 zeros consecutivos dentre as 2000 últimas casas decimais, de modo que n = 4⋅51999 + 2000 satisfaz as condições do enunciado (pois ϕ(52000) = 4⋅5 1999) ❏

Teorema Chinês dos restos: Se mdc (m, n) = 1 então f: Z/mnZ→ Z/mZ × Z/nZ, f (a (módulo n)) = (a (módulo m), a (módulo n)) é uma bijeção. Demonstração: f está bem definida, pois se a = b (módulo mn) então a ≡ b (módulo m) e a ≡ b (módulo n). Como Z/mnZ e Z/mZ × Z/nZ têm mn elementos cada, é suficiente verificar que f é injetiva. E, de fato, se a ≡ b (módulo m) e a ≡ b (módulo n) então m  (b – a) e n(b – a) ⇒ b – a = = mk, nmk ⇒ nk, pois mdc (m, n) = 1 ⇒ mn(b – a) ⇒ a ≡ b (módulo mn) ❏ Corolário: Se m1, m2,…, mr ≥ 1 são inteiros, e mdc (mi, mj) = 1 para i ≠ j então f: Z/m1 m2,…mrZ → Z/m1Z × Z/m2Z ×…× Z/mrZ, f (a (módulo m1 ⋅ m2 . …. mr)) = (a (módulo m1), … , a (módulo mr)) é uma bijeção ❏ Notemos que este Corolário mostra que, dados inteiros a1, a2,…ar, existe um inteiro n com n = a1 (módulo m1), n ≡ a2 (módulo m2), …, n ≡ ar (módulo mr).

Proposição: Temos f ((Z/mnZ)*) = (Z/mZ)* × (Z/nZ)* para a função f definida acima. Demonstração: Isto segue do fato de que a é primo com mn se e só se a é primo com m e a é primo com n ❏

EUREKA! N° 2, 1998

46


Sociedade Brasileira de Matemática

Corolário: mdc (m, n) = 1 ⇒ ϕ (mn) = ϕ (m) ⋅ ϕ (n) ❏ α α α Como conseqüência, se n = p1 1 , p 2 2 ... p k k onde p1, p2,… pk são primos distintos, α1, α2,… αk ∈ N* então ϕ(n) = n (1–1/p1) (1–1/p2)…(1–1/pk). Em particular, se n ≥ 3 então ϕ(n) é par ❏ Vamos mostrar um problema cuja solução usa de modo não trivial o teorema chinês dos restos:

Problema: Prove que dado n ∈ N existe um conjunto de n elementos

A ⊂ N tal que para todo B ⊂ A, B ≠ ∅, ∑ x é uma potência não trivial x∈B

(isto é, um número da forma mk, onde m, k são inteiros maiores ou iguais a 2), ou seja, A = {x1, x2,… xn} tal que x1, x2,…xn, x1 + x2, x1 +x3,…, x n −1 + x n , …,x1 + x2 +…xn são todos potências não triviais.

Solução: A = {4} é solução para n = 1, A = {9,16} é solução para n = 2. Vamos provar a existencia de um tal conjunto por indução em n. Suponha que A={x1,…, xn} é um conjunto com n elementos e para todo B ⊂ A, B ≠ ∅, x = m Bk B Vamos mostrar que existe c ∈ N tal que o conjunto

∑ x∈B

à ={cx1, cx2, …, cxn, c} satisfaz o enunciado. Seja A = mmc {kB, B ⊂ A, B ≠ ∅} o mínimo múltiplo comum de todos os exponentes kB. Para cada B ⊂ A, B ≠ ∅ associamos um número primo pB > A , de forma que B1 ≠ B2 ⇒ p B1 ≠ p B2 , e associamos um natural r com rB ≡ 0 (módulo px ), ∀X ≠ B, A rB + 1 ≡ 0 (módulo pB) (tal rB existe pelo teorema chinês dos restos), e tomamos

c = ∏ (1 + mBk B ) ArB B⊂ A B ≠∅

Como c é uma potência A -ésima, c é uma potência kB-ésima para todo B ⊂ A, B ≠ ∅, portanto, para B’⊂ {cx1, cx2,…,cxn}, B’≠∅, teremos B’= {cxx ∈ B} para algum B ⊂ A, B ≠ ∅. Logo x será uma potência

x∈B '

kB-ésima.

EUREKA! N° 2, 1998

47


Sociedade Brasileira de Matemática

Além disso,

∑ x = c(1 + m

X ∈B 'U {c}

KB B

  K X ArX   ) = ∏ (1 + m X ) ⋅ (1 + m BK B ) ArB +1 , X ⊂A   X ≠∅ , B 

que é uma potência pB-ésima, pois rX é múltiplo de pB para X ≠ B e A rB + 1 é múltiplo de pB

Seção 3: Ordens e raízes primitivas. Dados n ∈ N* e a ∈ Z com mcd (a, n) = 1, definimos a ordem de a módulo n, ordn a: = min {t ∈ N*at ≡ 1(módulo n)}. Dado a ∈ (Z/nZ)* definidos orda = ord n a. Proposição: {t ∈ N*a t ≡ 1(módulo n)}={k.ord n a, k ∈ N*}. Demonstração: Como a

a

k .ord n a

= (a

ord n a

≡ 1 (módulo n), para todo k ∈ N tem-se

) ≡ 1 = 1 (módulo n). Por outro lado, se t ∈ n, at ≡ 1

ord n a k

k

(módulo n), existe k ∈ N com

t = k ⋅ ord n a + r ,0 ≤ r < ord n a ⇒ a t = a k .ord n a ⋅ a r ≡ 1.a r ≡ a r (módulo n) ⇒ a r ≡ 1 (módulo n), portanto r = 0 ( pois 0 < r < ord n a contradiria a minimalidade de ord n a ), e t = k. ord n a

Corolário: ord n a ϕ(n) ❏ Definição: Se ord n a = ϕ(n), dizemos que a é raiz primitiva módulo n. Exemplos: 2 é raiz primitiva módulo 5, pois 21 = 2, 22 = 4, 23 = 8, 24 = 16, que é a primeira potência de 2 congruente a 1 módulo 5 e 4 = ϕ(5). • 1 é raiz primitiva módulo 2, pois ord2 1 =1 = ϕ(2). • 3 é raiz primitiva módulo 4, pois ord4 3 = 2 = ϕ(4). Proposição 3.1: a é raiz primitiva módulo n ⇔ {a t, t ∈ N} = (Z/nZ)*.

EUREKA! N° 2, 1998

48


Sociedade Brasileira de Matemática

Demonstração: Para todo a ∈ Z com mdc (a, n) = 1 temos {a t, t ∈ N} ⊂ (Z/nZ)*. Se a é raiz primitiva módulo n então os números 1, a, a2,…aϕ(n)–1 são distintos (módulo n) pois ai = aj (módulo n), com 0 ≤ i < j < ϕ(n) ⇒ aj–i ≡ 1 (módulo n) com 0 < j – i < ϕ(n), absurdo ⇒ {at, t ∈ N} = (Z/nZ)*. Por outro lado, #{a t, t ∈ N}≤ ord n a (o argumento acima mostra que de fato vale a igualdade), e portanto {a t, t ∈ N} = (Z/nZ)* ⇒ ord n a = ϕ(n) ❏

Corolário 1: Se m divide n e a é a raiz primitiva módulo n então a é raiz primitiva módulo m ❏ Corolário 2: Se k ≥ 3, então não existe nenhuma raiz primitiva módulo 2k. Prova: Pelo corolário anterior, basta provar que não existe raiz primitiva módulo 8, e isso segue do fato que se a é ímpar, a = 2r + 1, r ∈ Z ⇒ a2 = 4r ( r + 1) + 1 ≡ 1 (módulo 8) ❏ Proposição 3.2: Sejam p um número primo, e a ∈ Z raiz primitiva módulo p. Então a ou a + p é raiz primitiva módulo p2. Demonstração: Por hipótese, ord p a = ordp(a+ p) = ϕ(p) = p – 1. Portanto p – 1 ord p 2 a (pois at ≡ 1 (módulo p2) ⇒ at ≡ 1(módulo p)), e, como

ord p 2 a ϕ(p2) = p( p – 1), devemos ter ord p 2 a = p – 1 ou ord p 2 a = p ( p – 1) = ϕ(p2). Do mesmo modo, ord p 2 (a + p) = p – 1 ou

ord p 2 (a + p) = p(p – 1) = ϕ(p2). Basta provar, portanto, que ord p 2 a ≠ p – 1 ou ord p 2 ( a + p) ≠ p – 1. Suponha que ord p 2 a = p – 1. Portanto, a p–1 ≡ 1 (módulo p2), e então (a + p)

p–1

=a

p–1

+ (p – 1) pa

p–2

+ C p2 −1 ap–3. p2 +… ≡ 1 + (p – 1) pa

p–2

(módulo p2), portanto (a + p) p–1 não é congruente a 1(módulo p2), pois p2 não divide (p – 1) pa p –2, donde ord p 2 (a + p) ≠ p – 1 ❏ EUREKA! N° 2, 1998

49


Sociedade Brasileira de Matemática

Proposição 3.3: Se p é um número primo ímpar e a é raiz primitiva módulo p2 então a é raiz primitiva módulo pk para todo k ∈ N. Demonstração: temos ap–1 ≡ 1 (módulo p), mas ap–1 não é congruente a 1 (módulo p2), portanto ap–1 = 1 + b1 p, onde p não divide b1. Vamos mostrar por indução que a p

k ≥ 1: Temos a p

k

k −1

( p −1)

( p −1)

= 1 + bk p k , onde p não divide bk, para todo

= (ap

k −1

( p −1)

) p = (1 + bk p k −1 ) p =

= 1 + bk p k +1 + C p2 bk2 p 2 k + ... ≡ 1 + bk p k +1 (módulo pk + 2). Logo

ap

k

( p −1)

= 1 + bk p k +1 , com bk

+1

≡ bk (módulo p). Segue-se que p não

divide bk +1. Vamos agora mostrar por indução que a é raiz primitiva módulo pk para todo k ≥ 2. Suponha que a seja raiz primitiva módulo pk. Então temos pk –1( p – 1) = ϕ(pk) = ord p k a  ord p k +1 a ϕ(pk +1) = pk( p – 1). Portanto,

ord p k +1 a = pk –1(p – 1) ou ord p k +1 a = pk (p – 1) = ϕ(pk+1), mas o primeiro k −1

( p −1)

= 1 + bk p k ,que não é congruente a 1 módulo pk+1, pois p não divide bk. Portanto ord p k +1 a = ϕ(pk+1) e a é raiz caso é impossível, pois a p

primitiva módulo p k+1

Exemplo: 2 é raiz primitiva módulo 5k, ∀k ∈ N. De fato, 2 é raiz primitiva módulo 5, e, como 24 = 16 ≠ 1 (módulo 25), 2 é raiz primitiva módulo 25 = 52 (como na proposição 3.2). Portanto, pela proposição 3.3, 2 é raiz primitiva módulo 5k, ∀k ∈ N. Exercício resolvido: Mostre que existe n natural tal que os mil últimos dígitos de 2n pertencem a {1, 2}.

Solução: Observamos inicialmente que para todo k ∈ N existe um número mk de k algarismos, todos 1 ou 2, divisível por 2k. De fato, m1 = 2 e m2 = 12 satisfazem o enunciado. Seja mk = 2 k⋅ rk , rk ∈ N. Se rk é par, tome mk+1 = 2⋅10k + mk = 2k+1 (5k + rk /2), e se rk é ímpar, tome mk+1 = 10k + mk=2k+1(5k + rk)/2. EUREKA! N° 2, 1998

50


Sociedade Brasileira de Matemática

Como m1000 ≡ 2 (módulo 10), 5 não divide r1000 = m1000/21000. Como 2 é raiz primitiva módulo 51000, existe k ∈ N com 2k ≡ r1000 (módulo 51000). Logo 2k = b ⋅ 51000 + r1000, para algum b ∈ N. Portanto, 2k+1000 = b ⋅ 101000 + 21000 ⋅ r1000 = b ⋅ 101000 + m1000, e as 1000 últimas casas de 2k+1000 são as 1000 casas de m1000, que pertencem todas a {1, 2} ❏

Observação: Se p é primo ímpar, k ∈ N e a é um inteiro ímpar tal que a é raiz primitiva módulo pk então a é raiz primitiva módulo 2pk, pois ϕ(pk)= ord p k a  ord 2 p k a ϕ(2pk) = ϕ (pk) ⇒ ord 2 p k a = ϕ(2pk). Isso implica que se a é raiz primitiva módulo pk então a ou a + pk é raiz primitiva módulo 2pk ( pois a e a + pk são raízes primitivas módulo pk e um deles é ímpar.) ❏

Proposição 3.4: Se n = ab, com a ≥ 3 e b ≥ 3 inteiros tais que mdc(a, b) = 1, então não existe raiz primitiva módulo n. Demonstração: Temos ϕ(n) = ϕ(a)⋅ ϕ(b) e, como a ≥ 3 e b ≥ 3 , ϕ(a) e ϕ(b) são pares. Se mdc (k, n) = 1 então temos kϕ(n)/2 = (kϕ(b)/2)ϕ(a) ≡ 1 (módulo a), e kϕ(n)/2 = (kϕ(a)/2)ϕ(b) ≡ 1 (módulo b). Assim, kϕ(n)/2 = 1(módulo n), e portanto ord n k  ϕ(n)/2 < ϕ(n) ❏ Teorema: Existe alguma raiz primitiva módulo n se, e só se, n = 2, n = 4, n = pk ou n = 2pk onde p é primo ímpar. Prova: Pelos resultados anteriores, basta provar que se p é primo ímpar então existe raiz primitiva módulo p, ou seja, existe a ∈ (Z/pZ)* com ordp a = p – 1. Para cada a ∈ (Z/pZ)*, tem-se ordp a( p – 1). Seja d um divisor de p – 1. Definimos N(d) = # {a ∈ (Z/pZ)*ordp a = d}. Temos portanto p – 1 =

N(d). O resultado seguirá dos dois lemas

d p −1

seguintes:

Lema 1: N(d) ≤ ϕ(d) para todo d divisor de p – 1.

EUREKA! N° 2, 1998

51


Sociedade Brasileira de Matemática

Prova: Se N(d) > 0 então existe a ∈ (Z/pZ)* com ordp a. Se ordp a = d, entãoad = 1 e, para 0 ≤ k < d, as classes de ak são todas distintas módulo p, e (ak)d =1. Como a equação xd –1 = 0 tem no máximo d raízes distintas em Z/pZ (pois Z/pZ é um corpo), suas raízes são exatamente ak, 0 ≤ k < d. Por outro lado, ordp ak = d ⇒ mcd(k, d) = 1, pois se r > 1 é tal que rk e rd então (ak)d/r = (ad)k/r ≡ 1(módulo p), logo ord p(ak) ≤ d/r < d. Desta forma, {b ∈ (Z/pZ)* ord pb = d} ⊂ {ak, 0 ≤ k < d e mcd (k,d) = 1}, portanto N(d) ≤ ϕ(d) ❏ Lema 2: ϕ(d) = n, para todo n ∈ N.

∑ d n

Prova do Lema 2: Considere os n números racionais 1/n, 2/n, …,n/n. Ao simplificá-los, aparecem exatamente ϕ(d) deles com denominador d, para cada divisor d de n. Portanto, ϕ(d) = n ❏

∑ d n

Fim da prova do teorema: Do Lema 2 segue que ϕ(d) = p – 1 e, como p – 1=

d p −1

N(d) e N(d)

d p −1

≤ ϕ(d) para todo d, devemos ter N(d) = ϕ(d) para todo d. Em particular, N(p – 1) = ϕ(p – 1) > 0 ⇒ existem raízes primitivas módulo p ❏ PROBLEMAS

1)

2) 3) 4) 5)

Prove que existem infinitos números primos congruentes a 3 módulo 4. Determine todos os n naturais tais que (2n – 1)/n é inteiro. Determine todos os n naturais tais que (2n + 1)/n2 é inteiro. Prove que se a e b são naturais e (a2 + b2) / (ab + 1) é inteiro então (a2 + b2) / (ab + 1) é quadrado perfeito. Sejam a, n ∈ N*. Considere a sequência (xn) definida por x1 = a, x

xk+1 = a k , ∀ k ∈ N. Mostre que existe N ∈ N tal que xk+1 ≡ xk (módulo n), para todo k ≥ N.

Obs.: Os problemas 3 e 4 foram propostos na 31a. e na 29a. Olimpíada Internacional de Matemática (1990 e 1988) respectivamente. EUREKA! N° 2, 1998

52


Sociedade Brasileira de Matemática

SOLUÇÕES DE PROBLEMAS PROPOSTOS EUREKA! N°1 Publicamos aqui algumas das respostas enviadas por nossos leitores. 1)

Mostre que, dado um conjunto de n pessoas, existem duas que possuem o mesmo número de amigos entre as pessoas do conjunto.

SOLUÇÃO

Primeira Hipótese: há apenas uma única pessoa sem amigos; logo entre as n – 1 pessoas restantes, cada pessoa é amiga de no mínimo uma pessoa e no máximo n – 2 pessoas. Seja f : P → Q onde P = conjunto das pessoas restantes e Q = conjunto dos possíveis números de amigos de uma determinada pessoa em P, ou seja: P = { p1, p2, …, pn–1} Q = { 1, 2, 3, …, n – 2} Observe que há n – 2 valores no conjunto Q para n – 1 valores em P ; isto quer dizer que ∃ n1, n2 ∈ P tais que f ( n1 ) = f ( n2 ). Segunda Hipótese: Suponha que todas as n pessoas tenham amigos entre si, ou seja: P = { p1, p2, …, pn} e Q = { 1, 2, 3, …, n – 1} Observe que agora o conjunto Q possui n – 1 valores, pois cada pessoa de P possui no mínimo 1 amigo e no máximo (n – 1) amigos entre as (n – 1) pessoas restantes. Pelo mesmo motivo da primeira hipótese ∃ n1, n2 ∈ P tais que f ( n1 ) = f ( n2 ). Conclusão: há pelo menos duas pessoas com a mesma quantidade de amigos. 2)

Em uma pista circular há postos de gasolina, e o total de gasolina que há nos postos é exatamente o suficiente para um carro dar uma volta. Prove que existe um posto de onde um carro com o tanque inicialmente vazio pode partir e conseguir dar uma volta completa na pista (parando para reabastecer nos postos).

3)

O Professor Carlos Alberto da Silva Victor observou que o problema 3 estava com o enunciado errado (de fato, n1998 é um quadrado

EUREKA! N° 2, 1998

53


Sociedade Brasileira de Matemática

perfeito e portanto deve ser congruente a 0 ou a 1 módulo 4, não podendo pois terminar por 11 na representação decimal.) O enunciado correto é: Prove que existe n ∈ N tal que os 1000 primeiros dígitos de n1998 são iguais a 1. 4)

Escreva 1998 como soma de (um número arbitrário de ) parcelas de modo que o produto das parcelas seja o maior possível.

SOLUÇÃO

Observe inicialmente que, dado n ∈ N, n n (i) se n (n > 4) é par, temos ⋅ > n 2 2  n −1  n +1 (ii) se n (n > 3) é ímpar, temos  ⋅ >n  2   2  Sejam 1998 = n1 + n2 + n3 + … nk e P = n1 ⋅ n2 ⋅ n3 ⋅ … nk Com as observações (i) e (ii) devemos ter ni ∈ { 1, 2, 3, 4} e como 4 = 2 ⋅ 2 podemos substituir 4 por "2 + 2" e teremos ni ∈ { 1, 2, 3}; logo P = 1α ⋅ 2β ⋅ 3δ . É evidente que α = 0; pois se α = 1, "1 + 2" pode ser substituído por um 3 e "1 + 3" pode ser substituído por "2 + 2". Também β ≤ 2, pois "2 + 2 + 2" pode ser substituído por "3 +3" ( 3 ⋅ 3 > 2 ⋅ 2 ⋅ 2) e conseqüentemente P = 2β ⋅ 3δ com (β = 1 ou 2 ). Como 1998 = 3 ⋅ 666 + 0, P = 3666 e S = 3 + 3 + 3 + ... + 3 666 vezes

5)

Sejam a > 0 e P1P2P3P4P5 uma poligonal aberta contida em um dos semiplanos determinados pela reta P1 P5 . Prove que existem pontos P6 e P7 no plano, com P5 P6 = a, de modo que é possível ladrilhar o plano com infinitos ladrilhos congruentes ao heptágono P1P2P3P4P5P6P7.

6)

Mostre que toda seqüência com n2 + 1 elementos possui uma subseqüência crescente com n + 1 elementos ou uma subseqüência decrescente com n +1 elementos.

EUREKA! N° 2, 1998

54


Sociedade Brasileira de Matemática

7)

Prove que 1 + 2 + 3 + ... + 1998 < 2

SOLUÇÃO

Definamos a função ϕ : N – {0} → Z tal que ϕ (1) = 2 ϕ (n + 1) = ϕ (n)2 – n, n ≥ 1 Temos que

1< ϕ (1) = 2 2< ϕ (2) = ϕ (1)2 – 1 = 22 – 1 = 3

Mostraremos que agora por indução que n < ϕ (n) para todo n ≥ 3

ϕ (3) = ϕ (2)2 – 2 = 32 – 2 = 7. Logo, 3 < ϕ (3) (Hipótese de indução) suponhamos que n < ϕ (n)

• •

como 0 < n < ϕ (n), segue que n2 < ϕ (n)2 isto é, n2 < ϕ (n +1) + n. Dai, n2 – n < ϕ (n +1) Mas n + 1 < n2 – n se e somente se 0 < n2 – 2n – 1 se e somente se 0 < n2 – 2n + 1 – 2 se e somente se 0 < (n – 1)2 – 2. Esta última desigualdade é verdadeira se n ≥ 3 Portanto, se n ≥ 3, n +1 < n2 – n < ϕ (n + 1) e dai n +1 < ϕ (n + 1). Pelo princípio de indução, segue que n < ϕ (n) para todo n ≥ 3 como para todo n ∈ N –{0} e daí, Portanto,

n≤ n

n ≤ ϕ (n) para todo n ∈ N –{0}

1998 < ϕ (1998) 1998 < ϕ (1997)2 – 1997 1997 +

1998 < ϕ (1997)2

1997 + 1998 < ϕ (1997) pois 0 < 1997 < ϕ (1997). Prosseguindo desta maneira, chegaremos a

1 + 2 + 3 + ... + 1998 < ϕ (1) = 2 8)

Considere um torneio de xadrez envolvendo brasileiros e argentinos em que cada jogador joga contra todos os outros

EUREKA! N° 2, 1998

55


Sociedade Brasileira de Matemática

exatamente uma vez. Ao final do torneio, cada jogador obteve metade dos pontos que conquistou jogando contra brasileiros e metade jogando contra argentinos. Prove que o número total de jogadores do torneio é um quadrado perfeito (obs: cada vitória vale 1 ponto, empate 1/2 ponto e derrota 0 ponto). SOLUÇÃO

Sejam k o número de brasileiros e n o número de argentinos no torneio. Cada jogador brasileiro jogou k – 1 partidas contra brasileiros. Observe que o número de vitórias, o número de empates e o número de derrotas (de cada jogador brasileiro contra jogadores brasileiros) somadas deve ser igual a k – 1. (i)

Seja s o número total de vitórias ocorridas entre brasileiros e E o número de empates, logo: 2s + E = k ( k – 1); pois o número de vitórias é igual ao número de derrotas.

(ii)

Usando a mesma idéia do item (i) para os argentinos, temos: 2s' + E' = n ( n – 1); onde s' é o número total de vitorias entre argentinos e E' o número total de empates entre argentinos.

(iii)

Sejam P = s +

, os totais de pontos obtidos nos 2 2 itens (i) e (ii) entre brasileiros e entre argentinos, respectivamente.

(iv)

Suponha agora que os jogos entre brasileiros e argentinos; logo cada brasileiro joga n partidas com os argentinos e cada argentino jogou k partidas com os brasileiros. Seja p o total de vitórias que os brasileiros obtiveram com os argentinos e q o total de empates que os brasileiros obtiveram com os argentinos, logo 2p + q = nk. Como o total de pontos de cada brasileiro, metade foi contra q brasileiros e outra metade entre argentinos, temos P = p + 2 Sejam p' o total de vitórias que os argentinos obtiveram contra os brasileiros e q' o total de empates que os argentinos obtiveram contra os brasileiros, logo:

E

e P' = s +

EUREKA! N° 2, 1998

56

E'


Sociedade Brasileira de Matemática

2p' + q' = n k e também P' = p' +

q'

2

.

De (i), (ii), (iii) e (iv) temos:

E q   P = s + 2 = p + 2 ⇒ 2 s + E = 2 p + q   P ' = s '+ E ' = p'+ q ' ⇒ 2s '+ E = 2 p '+ q '  2 2

(v)

Somando (v) teremos:

2 s +

E + 2 s '+

E ' = 2 p + q + 2 p'+ q'

↓ ↓ ↓ k(k–1) + n(n –1) = nk

+

↓ nk

n + k = (n – k)2 ou seja o total de jogadores é um quadrado perfeito: Nota: Para cada n, k com n + k = (n – k)2 é possível construir torneios com k brasileiros e n argentinos satisfazendo as condições do enunciado. Note

t2 + t t2 −t também que se n + k = t então n = e k= . 2 2 2

9)

Prove que todo número racional positivo pode ser escrito como soma de um certo número de frações distintas de numerador 1.

SOLUÇÃO

(i)

Seja inicialmente a fração

1 n

p q

<

1 n −1

p q

< 1 , logo ∃ n ∈ N tal que

, observe que para n ≥ 2, temos:

p q

=

1 n

+

np − q nq

np − q

.

até nq encontrarmos a fração inicial como uma soma de frações com Nós podemos repetir o processo inicial para a fração

EUREKA! N° 2, 1998

57


Sociedade Brasileira de Matemática

numeradores iguais a 1; observe também que np – q < p, ou seja o numerador da fração

np − q é menor do que o numerador da nq

fração original e já que os numeradores dessas frações não podem decrescer indefinidamente, este procedimento deverá terminar com um número finito de frações com numeradores iguais a 1. Resta então mostrar que essas frações são todas distintas; se não vejamos:

np − q p 1 1 1 1 1 = − < − = ≤ (n ≥ 2); nq q n n − 1 n n(n − 1) n np − q

é escrita como uma soma de frações de nq numeradores iguais a 1, todos os denominadores dessas frações são maiores do que n, mostrando portanto que essas frações são todas distintas. então quando

(ii)

Seja

1+

1 2

p > 1 , então ∃ n ∈ N tal que: q +

1 3

+ ... +

1 n

p q

< 1+

1 2

+

1 3

+ ... +

1 n +1

θ θ 1 , com < < 1, 2 3 4 ϕ n +1 q n ϕ θ usando o ítem (i) podemos expandir como uma soma finita de ϕ

logo:

p

= 1+

1

+

1

+

1

+ ... +

1

+

frações unitárias cujos denominadores são maiores que "n + 1". Soluções dos problemas 1, 4, 8 e 9 enviadas por Carlos Alberto da Silva Victor, Nilópolis, Rio de Janeiro-RJ. Solução do problema 7 enviada por Manuel João de Jesus Almeida, Rio de Janeiro-RJ. Agradecemos também a participação de Carlos Eduardo Cardoso Borges, Wayne L. Silva de Paula, Marco Rogério Vieira e Vicente Wilson Moura Gaeta. Continuamos esperando as soluções dos problema 2, 3, 5 e 6.

EUREKA! N° 2, 1998

58


Sociedade Brasileira de Matemática

PROBLEMAS PROPOSTOS Convidamos o leitor a enviar soluções dos problemas propostos e sugestões de novos problemas para os próximos números.

10)

Suponha que temos k moedas, todas iguais exceto por uma que tem peso ligeiramente diferente das anteriores (não se sabe se maior ou menor), e uma balança de dois pratos.

a)

Mostre que se k ≤

b)

c)

3n − 3

é possível determinar com n pesagens 2 qual é a moeda diferente, e se ela é mais leve ou mais pesada que as outras. 3n − 1 Mostre que se k = é possível determinar com n pesagens 2 qual é a moeda diferente, mas nem sempre é possível dizer se ela é mais leve ou mais pesada que as outras. Mostre que se k >

3n − 1 2

não é sempre possível determinar qual é

a moeda diferente.

11)

Determine todas as soluções de xy = yx com x e y racionais positivos.

12)

a) Prove que se n ∈ N e 2n + 1 é um número primo então n é uma potência de 2. b) Prove que se a, n ∈ N, n ≥ 2 e an –1 é primo, então a = 2 e n é primo.

13)

Dado n ∈ N determine determine o maior k ∈ N tal que existam conjuntos A1, A2,…, Ak contidos em {1, 2, …, n} de forma que Ai ⊄ Aj para todo i ≠ j.

EUREKA! N° 2, 1998

59


Sociedade Brasileira de Matemática

14)

(Problema proposto por Antonio Luiz Santos): Determine o número 1 1 1 com x e y inteiros positivos. de soluções de + = x y 1998

15)

Considere uma seqüência de triângulos retângulos AnBnCn no plano cuja hipotenusa seja BnCn, com as seguintes condições:

i) ii) iii)

A1 B1 = A1C1 = 1 Bn + 1 = Bn e An + 1 = Cn para todo n ∈ N. An+1 Cn+1 é congruente à altura de An em relação a BnCn. Mostre que qualquer ponto do plano pertence a infinitos triângulos AnBnCn .

Você sabia… Que há 6 poliedros regulares no espaço euclidiano de 4

dimensões mas apenas 3 em 5 ou mais dimensões ?? Em dimensão n há o simplexo, com n + 1 "faces" (que são simplexos) de dimensão n – 1, o hipercubo, com 2n "faces" ( que são hipercubos) de dimensão n – 1 e o hiperoctaedro, dual do hipercubo, com 2n "faces" (que são simplexos) de dimensão n – 1. Em dimensão 4, além desses há o C24, que tem 24 "faces" octaédricas, o C120, que tem 120 "faces"dodecaédricas e o C600, que tem 600 "faces" tetraédricas. Lembre-se que em 3 dimensões há 5 poliedros regulares: o tetraedro (caso particular do simplexo), o cubo (caso particular do hipercubo), o octaedro (caso particular do hiperoctaedro), o dodecaedro, que tem 12 faces pentagonais, e o icosaedro, que tem 20 faces triangulares.

EUREKA! N° 2, 1998

60


Sociedade Brasileira de Matemática

AGENDA OLÍMPICA

OLIMPÍADA BRASILEIRA DE MATEMÁTICA Primeira Fase – Sábado, 6 de junho Segunda Fase – Sábado, 12 de setembro Terceira Fase – Sábado, 24 de outubro (níveis 1, 2 e 3) Domingo, 25 de outubro (nível 3). ♦ OLIMPÍADA IBEROAMERICANA DE MATEMÁTICA 13 a 20 de setembro de 1998 República Dominicana.

Você sabia… Que é possível pentear um toro (superfície em forma de pneu) cabeludo mas não uma esfera cabeluda sem deixar rodamoinhos??

EUREKA! N° 2, 1998

61


Sociedade Brasileira de Matemática

COORDENADORES REGIONAIS Alberto Hassen Raad Antônio C. Rodrigues Monteiro Amarísio da Silva Araújo Angela Camargo Antônio C. do Patrocínio Ariosto de Oliveira Lima Benedito T. Vasconcelos Freire Carlos A. Bandeira Braga Claudio Arconcher Egnilson Miranda de Moura Élio Mega Florêncio F. Guimarães F. Francisco Dutenhefner Gisele de A. Prateado G. Ivanilde H. Fernandes Saad João B. de Melo Neto João F. Melo Libonati José Carlos Pinto Leivas José Luis Rosas Pinho José Paulo Carneiro José Vieira Alves Leonardo Matteo D'orio Licio Hernandes Bezerra Luzinalva M. de Amorim Marco Polo Marcondes Cavalcante França Mario Jorge Dias Carneiro Ma-To-Fú Pablo Rodrigo Ganassim Paulo H. Cruz Neiva de L. Jr. Reinaldo Gen Ichiro Arakaki Ricardo Amorim Sergio Claudio Ramos Tadeu Ferreira Gomes Wagner Pereira Lopes

(UFJF) (UFPE) (UFV) (Centro de Educação de Adultos CEA) (IMECC/UNICAMP) (UFPI) (UFRGDN) (UFPB) (Col. Leonardo da Vinci) (Col. Agrícola do Bom Jesus) (Col. ETAPA) (UFES) (UFMG ) (UFGO) (U. Católica Dom Bosco) (UFPI) (Grupo Educ. IDEAL) (URG) (UFSC) (USU) (UFPB) (Parque de Material Aeronáutico de Belém) (UFSC) (UFBA) (Colégio Singular) (UF Ceará) (UFMG) (UEM) (L. Albert Einstein) (Esc. Tec.Everardo Passos) (INPE) (Centro Educ. Logos) (IM-UFRGS) (U. do Estado da Bahia) (Esc. Tec. Fed. de Goiás)

EUREKA! N° 2, 1998

62

Juiz de Fora-MG Recife-PE Viçosa-MG Blumenau-SC Campinas-SP Parnaíba-PI Natal-RN João Pessoa-PB Jundiaí-SP Bom Jesus-PI São Paulo-SP Vitória-ES BH-MG Goiânia-GO Campo Grande-MS Teresina-PI Belém-PA Rio Grande-RS Florianópolis-SC Rio de Janeiro-RJ Campina Grande-PB Belém-PA Florianópolis-SC L. de Freitas-BA Santo André-SP Fortaleza-CE BH-MG Maringá-PR Rio das Pedras-SP Piracicaba-SP S.J.Campos-SP Nova Iguaçu-RJ Porto Alegre-RS Juazeiro-BA Jataí-GO


CONTEÚDO AOS LEITORES

2

OLIMPÍADA BRASILEIRA DE MATEMÁTICA Problemas de treinamento para a terceira fase

3

XIX OLIMPÍADA BRASILEIRA DE MATEMÁTICA (1997) Problemas e soluções da segunda fase sênior

8

13a. OLIMPÍADA IBEROAMERICANA DE MATEMÁTICA República Dominicana 1998 - Problemas e resultados

18

39a. OLIMPÍADA INTERNACIONAL DE MATEMÁTICA Soluções

20

ARTIGOS O PRINCÍPIO DA INDUÇÃO Elon Lages Lima

26

FRAÇÕES CONTÍNUAS, REPRESENTAÇÕES DE NÚMEROS E APROXIMAÇÕES Carlos Gustavo Moreira

44

SOLUÇÕES DE PROBLEMAS PROPOSTOS EUREKA Nos. 1 e 2

56

PROBLEMAS PROPOSTOS

61

COORDENADORES REGIONAIS

62


Sociedade Brasileira de Matemática

AOS LEITORES 1998 tem sido um bom ano para o programa brasileiro de Olimpíadas de Matemática. Tivemos em torno de 40.000 participantes na primeira fase, ganhamos mais uma medalha de ouro na Olimpíada Internacional de Matemática e fomos o país com maior soma de pontos na Olimpíada Iberoamericana de Matemática. Esperamos concluí-lo com uma terceira fase da Olimpíada Brasileira de Matemática que faça jus aos resultados até agora obtidos, estimulando ainda mais a imaginação criativa dos jovens competidores, propiciando a descoberta de novos talentos para a matemática e, em particular, criando as bases para as equipes brasileiras que participarão nas olimpíadas internacionais do ano que vem. Esperamos que os números da revista Eureka! que apresentamos este ano sejam úteis para aumentar o número de participantes da Olimpíada Brasileira de Matemática e que permitam a todos os classificados chegarem bem preparados à terceira fase, além de contribuir para o enriquecimento da cultura matemática de nossa comunidade acadêmica e escolar. Esta Eureka! 3 está mais difícil que as anteriores, entre outras razões, por ter boa parte de seu material dedicado à preparação para a terceira fase do terceiro nível. Grande parte do material das Eureka! 1 e 2 é adequada à preparação para a terceira fase dos primeiros dois níveis, mas no terceiro nível a prova (como mostra a segunda fase sênior da Olimpíada Brasileira de Matemática do ano passado, aqui resolvida) costuma ser mais técnica, de modo que resolvemos usar a Eureka! 3 para oferecer aos participantes da terceira fase uma preparação adequada, com problemas mais difíceis e bem diferentes dos que usualmente se estudam nas escolas. A terceira fase será realizada nas seguintes datas. Sábado 24 de outubro

1o. nível 2o. nível 3o. nível (primeira prova).

Sábado 14 de novembro

3o. nível (segunda prova).

Comitê Editorial. EUREKA! N° 3, 1998

2


Sociedade Brasileira de Matemática

OLIMPÍADA BRASILEIRA DE MATEMÁTICA Problemas de treinamento para a terceira fase

1)

Sejam três pontos A, B e C pertencentes a uma circunferência de ∧

centro O tais que AOB < BOC. Seja D o ponto médio do arco AC que contém o ponto B. Seja K o pé da perpendicular a BC por D. Prove que AB + BK = KC.

2)

Prove que existe uma seqüência a0, a1, …, ak, …, onde cada ai é um algarismo (ou seja, ai ∈ {0, 1, 2, 3, 4, 5, 6, 7, 8, 9}) com a0 = 6, tal que para cada inteiro positivo n o número xn = a0 + 10a1 + 100a2 + … + 10n–1 an–1 (cuja representação decimal é an–1 an–2 …a1a0) é tal que x n2 − x n é divisível por 10n.

3)

Seja A = {x1 < x2 <…< xn} um conjunto de números inteiros positivos tal que se x e y são dois números naturais que não pertencem a A então x + y não pertence a A. Provar que xi ≤ 2i – 1 para i = 1, 2,…, n.

4)

Considere a seqüência (xn) todo n ∈ N, x n + 2

n∈N

definida por x1 = 19, x2 = 98 e, para

1  , se x n +1 ≠ 0 xn − x n +1 = 0, se x = 0. n +1 

Prove que existe n ∈ N tal que xn = 0 e encontre o menor n com essa propriedade. 5)

6)

Sejam ABC um triângulo, M o pé da bissetriz interna do ângulo A e N o pé da bissetriz interna do ângulo B. Suponha que MN seja bissetriz do ângulo AMC. Calcule o ângulo A. Ache todas as soluções reais de [x ] +

( [ y ] denota o único inteiro tal que

EUREKA! N° 3, 1998

3

[ 1998x ] = 1998

[ y ] ≤ y < [ y ] + 1) .


Sociedade Brasileira de Matemática

7)

Mostre

que

o

produto

de

todos

os

números

da

forma

± 1 ± 2 ± 3 ± ... ± 100 é o quadrado de um número inteiro.

Soluções 1)

Sejam AB = x, BD = y; marcamos D' tal que D'C = y. Então D'D = x por ser D ponto médio de AC e resulta DD' // BC. Se K' é o pé da perpendicular a BC por D', então temos AB = DD' = KK' e BK = K'C AB + BK = KK'+ K'C = KC.

2) O primeiro termo é a0 = 6; então x1 = 6 e x12 − x1 = 36 − 6 = 30 , que é divisível por 101. Seja n ≥ 1. Suponhamos que existem a0, a1,…,an–1 tais que x n = a 0 + 10a1 + 10 2 a 2 + ... + 10 n −1 a n −1 verifica que x n2 − x n é divisível por 10n (ou seja x n2 − x n = 10 n r , com r ∈ N) Temos que encontrar an tal que x n +1 = a 0 + 10a1 + 10 2 a 2 + ... + 10 n −1 a n −1 + 10 n a n = x n + 10 n a n seja tal que x n2+1 − x n +1 é divisível por 10 n +1.

EUREKA! N° 3, 1998

4


Sociedade Brasileira de Matemática

xn2+1 − xn+1 = ( xn + 10n an ) 2 − ( xn + 10n an ) = xn2 + 2xn 10n an +102n an2 − xn −10n an = = ( xn2 − xn ) + 10n (2xn an − an ) + 102n an2 = 10n r +10n (2xn an − an ) +102n an2 = = 10n (r + 2xn an − an ) + 102n an2 . Assim, xn2+1 − xn+1 e divisível por 10n+1 ⇔ r + 2xn an − an é divisível por 10 ⇔ r + (2xn −1)an é divisível por 10. Dado que a0 = 6, temos que xn = 10t + 6 com t ∈ N; então 2xn = 10h + 2. (com h = 2t + 1). Logo r + (2xn – 1)an é divisível por 10 ⇔ r + (10h + 1)an é divisível por 10 ⇔ r + an é divisível por 10. Sempre existe um único inteiro an entre 0 e 9 de modo tal que isto se verifique.

Obs: A seqüência (an) começa por 6, 7, 3, 9, 0, 1, 7, 8, 7, 1, 8, 0, 0, 4, 7, 3… Assim, por exemplo, x10 = 1787109376. Problema extra: Prove que a seqüência (an) obtida não é periódica nem pré-periódica. 3) Suponhamos que o enunciado é falso, ou seja que existe k tal que xk > 2k – 1, 1 ≤ k ≤ n. Então os conjuntos B1 = {1, xk – 1}, B2 = {2, xk – 2}, …, Bk = {k, xk – k} são disjuntos dois a dois e seus elementos são menores que xk. Além disso, para cada j, 1 ≤ j ≤ k, j ∈ A ou xk – j ∈ A, pois no caso contrario, ou seja, se j ∉ A e xk – j ∉ A, teríamos que xk = j + (xk – j) ∉ A. Portanto, para cada j, 1 ≤ j ≤ k, A ∩ Bj ≠ ∅, donde A tem pelo menos k elementos menores que xk , absurdo.

4) Se xn+1 ≠ 0, temos xn+2 xn+1 = xn+1 xn –1. Definindo yn = xn xn+1 temos yn+1 = yn–1 para todo n tal que xn+1 ≠ 0. Como y 1 = x1 x2 = 19 ⋅ 98 = 1862, temos yk = 1863 – k enquanto yk –1 for diferente de 0, e portanto y1862 = 1 e EUREKA! N° 3, 1998

5


Sociedade Brasileira de Matemática

y1863 = 0 ⇒ x1862 x1863 = 1 e x1863 x1864 = 0. Assim, x1863 ≠ 0 e x1864 = 0, donde 1864 é o menor n tal que xn = 0.

5)

AN

=

c

BM

=

c

⇒ MC =

ab

, e como MC d b+c MA AN c bc MN é bissetriz de AMˆ C devemos ter = = , donde MA = MC CN a b+c ab pela lei dos senos aplicada aos triângulos ABC e ABM (pois MC = b +1 Pelo teorema das bissetrizes,

NC

a

e

ac

temos senA = a = b + c = BM = sen ( A / 2 ) , e portanto sen (A/2) = sen A senB

b

bc

MA

senB

b+c

2π A π  A 1 . = ⇒ = ⇒ A= 2 3 3 2 2

= 2 sen (A/2) cos (A/2) ⇒ cos 

6) [x] + 1998x é sempre inteiro. Seja x0 a solução de x + 1998x = 1998, ou

[

]

seja x0 = 999 (3 – 5) = 763,1... e 1998x0 = 1998− x0 = 999( 5 − 1) = 1234,8...

[ ] [ 1998x ] = 1997. A função

Temos x + 0

0

de uma unidade quando x ou

f ( x ) = [x ] +

[ 1998x ] aumenta

1998 x torna-se inteiro. Os próximos valores

de x maiores que x0 para os quais x e 1998 x são inteiros são respectivamente 764 e 12352 / 1998 < 764. Assim, f (12352 /1998) = 763 + 1235 = 1998 e f (764) = 764 + 1235 = 1999 (de fato 1998 ⋅ 764 < 1236). Como f (x) é não-decrescente, o conjunto das soluções é o intervalo  1235 2  ,764  = [763 ,3758758758 ..., 764 ).   1998 

EUREKA! N° 3, 1998

6


Sociedade Brasileira de Matemática

7) O número referido no enunciado é o quadrado do produto de todos os 299 números da forma 1 ± 2 ± 3 ± ... ± 100 (no produto do enunciado cada um desses números aparece uma vez, assim como seu simétrico). Neste último produto, obtemos uma soma de termos do tipo

(σ , a , )(σ 1

2

a )...(σ 2

m

a ), com m ≤ 2 99 , a , a ,...a ∈ {2,3,...,100} e m

1

2

m

σ j ∈{-1,1}, ∀j. Fixamos α 2 , α 3 ,..., α 100 ∈ N com α 2 + α 3 + ... + α 100 ≤ 2 99 , e consideramos

todos os termos como acima que têm exatamente α k valores de a j = k ,

para 2 ≤ k ≤ 100 . Se todos os α j são pares esses termos são todos inteiros. Se algum deles (digamos α r ) é ímpar, podemos associar de forma bijetiva a cada termo desses o termo obtido trocando os sinais de todos os σ j para os quais a j = r. Assim, a cada termo associamos o seu simétrico, e portanto, nesse caso a soma dos termos considerados é 0. Assim, o produto de todos os números da forma 1 ± 2 ± 3 ± ... ± 100 é um inteiro, e portanto o produto do enunciado é um quadrado perfeito.

Você sabia…

que são conhecidos 51539600000

casas decimais de

π, calculadas por Y. Kamada e D.

Takahashi, da Universidade de Tokio em 1997? E que em 21/8/1998 foi calculada pelo projeto Pihex a 5000000000000a. casa binária de

π?

Consulte a home-page http://www.cecm.sfu.ca/pi

EUREKA! N° 3, 1998

7


Sociedade Brasileira de Matemática

XIX OLIMPIADA BRASILEIRA DE MATEMÁTICA (1997) Problemas e soluções da segunda fase sênior PROBLEMA 1

Duas circunferências de raios R e r e centros O e O', respectivamente, intersectam-se nos pontos P e P'. Seja l a reta que passa por P e P'. Determine em função de R e r, o menor valor que pode assumir a soma das distâncias de l a O e O'. PROBLEMA 2

Dizemos que um conjunto A ⊂ N satisfaz a propriedade P(n) se A tem n elementos e A + A = {x + y tal que x ∈ A e y ∈ A} tem

n( n + 1) 2

elementos.

Dado A ⊂ N finito definimos o diâmetro de A como sendo a diferença entre o maior e o menor elemento de A. Seja f (n) o menor diâmetro que um conjunto A satisfazendo P(n) pode ter. Mostre que

n

2

4

3

≤ f ( n ) < n para todo

n ≥ 2. (Se o seu tempo de prova não estiver esgotado, tente melhorar esta estimativa. Por exemplo, tente mostrar que f (p) < 2p2, para todo número primo p.) PROBLEMA 3

a)

Prove que não existem funções f : R → R e g : R → satisfazendo g ( f (x) ) = x3 e f ( g(x) ) = x2 para todo x ∈ R.

b)

Exiba funções f : ( 1, ∞ ) → ( 1, ∞ ) e g :( 1, ∞ ) → (1, ∞ ) tais que

g ( f(x) ) = x3 e f ( g(x) ) = x2 , para todo x ∈ (1, ∞ ).

EUREKA! N° 3, 1998

8

R


Sociedade Brasileira de Matemática

PROBLEMA 4

Seja Fn definido por F1 = 1, F2 = 1 e Fn+2 = Fn+1 + Fn , para todo n ≥ 1. Seja

Vn = Fn2 + Fn2+ 2 , n ≥ 1. Mostre que, para todo n inteiro positivo, Vn , Vn+1 e Vn+2 são lados de um triângulo de área 1/2. PROBLEMA 5

Sejam c ∈ Q, f (x) = x2 + c. Definimos f 0 (x) = x , f n+1 (x) = f ( f n (x)), n ∈ N . Dizemos que x ∈ R é pré-periódico se { f n (x), n ∈ N } é finito. Mostre que { x ∈ Q | x é pré-periódico } é finito. PROBLEMA 6

Seja f uma função do plano no plano que satisfaz d (P,Q) = 1 ⇒ d (f (P), f(Q)) = 1 para todos os pontos P e Q do plano. Mostre que d (f (P), f(Q)) = d (P, Q) para todos os pontos P e Q do plano. (d (X,Y) denota a distância entre X e Y). SOLUÇÕES 1)

Considere a circunferência de raio R fixa, cujo centro O está sobre uma reta s. O problema se resume a determinar a posição de O' em s que minimiza a soma d das distâncias de O e O' a A . Claramente, A é perpendicular a s. Seja I o ponto de intersecção de s com A . Temos dois casos a considerar:

OO' ≥ OI. Neste caso, d = OO' e d é mínimo quando I = O'. OO' ≤ OI. Neste caso, considere O'' ≠ O' em s tal que O'I = IO'' ( O'' é simétrico de O' em relação à A ). Assim, temos que d = OO'' ≥ OI (primeiro caso) e d é mínimo quando O' = I = O''. Em ambos casos, temos que d é mínimo quando O' = I . Por Pitágoras, este

(i) (ii)

mínimo é igual a

R2 − r 2 .

EUREKA! N° 3, 1998

9


Sociedade Brasileira de Matemática

2)

Dado um conjunto finito A ⊂ N , denotaremos por d(A) o diâmetro de A. Temos duas desigualdades a provar:

f (n) ≥

n2

, para todo n ≥ 2. 4 Vamos supor, por absurdo, que exista um conjunto A = {a1, a2, …,an}, n ≥ 2,

(i)

n2 . Como A satisfaz P(n), 4

tal que A satisfaz P(n), a1 < a2 < …< an e d(A) <

A + A = {a1 + a2, a1 + a2, …,an + an} tem

n( n + 1)

elementos. Como 2 a1 + a1 < a1 + a2 < … < an + an, temos que (an + an) – (a1 + a1) + 1 ≥ n( n + 1) n2 n − 2 n2 n2 ⇒ an – a1 ≥ + ≥ ⇒ d ( A) ≥ , o que é uma 2 4 4 4 4 contradição. Isto demonstra (i). (ii)

f (n) < n 3 , para todo n ≥ 2.

Como {0, 1} satisfaz P(2), temos que f(2) ≤ 1 < 23. Agora, vamos supor que f (n) < n 3 para algum n ≥ 2. Seja An = {a1, a2, …, an} ⊂ N tal que An satizfaz P(n) e d(An) = f(n) < n3. Sem perda de generalidade, podemos supor que 0 = a1 < a2 < … < an = d(An), bastando para isto subtrair de cada elemento de An o menor de seus elementos. Agora, queremos achar an + 1 ∈ N – An tal que An + 1 = {a1, a2, …, an + 1} satisfaça P(n +1) e

d (An + 1) < (n + 1)3. Como An + An tem

n( n + 1) 2

elementos e

An +1 + An +1 = ( An + An ) ∪ {ai + a n +1 1 ≤ i ≤ n + 1},

temos

que

P (n + 1) se e somente se, ai + a j a n +1 ∉ P = {ai + a j − a k 1 ≤ i, j , k ≤ n} ∪ { 1 ≤ i, j ≤ n}. Como 2 n(n + 1) n( n + 1) , temos que a n +1 ≤ n 3 + , pois basta escolher P ≤ n3 + 2 2 a n +1 ∈ N − An

an+1

como

e

o

An+1

menor

satisfaz

natural

que

EUREKA! N° 3, 1998

10

não

está

em

P.

Assim,


Sociedade Brasileira de Matemática

f (n + 1) ≤ d ( An +1 ) < (n + 1) 3 . Por indução finita em n, temos que (ii) é verdade, o que completa nossa demonstração. Vamos ainda, verificar que, para p primo ímpar, f ( p ) < 2 p 2 . Para isto, A = {k + 2 pg ( k ),0 ≤ k ≤ p − 1}, onde construímos o conjunto

g ( k ) = k 2 (mod p), 0 ≤ g(k) ≤ p – 1. Temos d ( A) ≤ p − 1 + 2 p ( p − 1) = 2 p 2 − p − 1 < 2 p 2 e se tivéssemos i + 2 pg (i ) + j + 2 pg (i ) = r + 2 pg ( r ) + s + 2 pg ( s ), então i + j + 2 p ( g (i ) + g (i )) = r + s + 2 p ( g (r ) + g ( s )) ⇒ i + j =

= r + s , g (i ) + g ( j ) = g ( r ) + g ( s ) Assim,

i − r = s − j e i 2 + j 2 ≡ r 2 + s 2 (mod p), logo (i − r )(i + r ) ≡ ( s − j )( s + j )(mod p ) ⇒ i − r ≡ s − j ≡ 0(mod p ) ou i + r ≡ s + j (mod p ) . Portanto i = r e s = j ou i = s e r = j. Com um pouco de teoria dos Corpos, é possível provar, utilizando um elegante argumento devido a Bose-Chowla, que, de fato, temos f ( p) < p2 para p primo. Seja K = Z/pZ o corpo com p elementos e L ⊃ K um corpo com p2 elementos. Seja θ um gerador do grupo (cíclico) multiplicativo de L, ou seja, tal que { θ k , k ∈ Z} = L − {0}. Para cada m ∈ K, θ + m ∈ L – {0, 1}, e, portanto, existe am ∈ Z, 0 < am < p2 – 1 tal que θ am = θ + m. O conjunto

A = {a m ,0 ≤ m ≤ p − 1}

tem

diâmetro

no

máximo

p2 − 3 < p2 e

a + a = a + a ⇒ (θ + i)(θ + j ) = (θ + r )(θ + s) ⇒ (i + j − r − s)θ + (ij − rs) = 0. i

j

r

s

Como θ ∉ K, temos i + j = r + s e ij = rs ⇒ {i, j} = {r , s}.

3) a)

Vamos supor, por absurdo, que existam funções f, g : R → R satisfazendo, para todo x ∈ R,

( I ) g ( f ( x)) = x 3 e ( II ) f ( g ( x)) = x 2

EUREKA! N° 3, 1998

11


Sociedade Brasileira de Matemática

Agora Logo

x, y ∈ R , f ( x ) = f ( y ) ⇒ g ( f ( x )) = g ( f ( y )) ⇒ x 3 = y 3 ⇒ x = y. f é injetora. Ainda, de (I) e (II), temos

( f ( x) 2 = f ( g ( f ( x)) = f ( x 3 ) ⇒ f (0) 2 = f (0), f (−1) 2 = f (−1) e f (1) 2 = f (1), logo { f ( 0 ), f ( 1 ), f ( − 1 )} ⊂ { 0 ,1} o que é um contradição (pois f é injetora e, portanto, { f (0), f (1), f (–1)} tem 3 elementos).

(b) Vamos supor, por enquanto, que existam funções 3 2 f , g : (1,+∞) → (1,+∞) tais que g ( f ( x)) = x e f ( g ( x )) = x , para todo x ∈ (1, ∞). Agora, considere as funções x

φ ( x) = log 2 (log 2 g (2 2 )) x

ψ ( x) = log 2 (log 2 f (2 2 )) Temos

φ D ψ ( x) = log 2 (log 2 g (2 2

log 2 (log 2 f ( 2 2

x

x

))

)) = log 2 (log 2 g ( f (2 2 ))) =

x

log 2 (log 2 (2 2 ) 3 ) = log 2 3 ⋅ 2 x = x + log 2 3

φ D ψ ( x) = log 2 (log 2 f (2 2

log 2 (log 2 g ( 2 2

x

x

))

)) = log 2 (log 2 f ( g (2 2 ))) =

x

log 2 (log 2 (2 2 ) 2 ) = log 2 2 x +1 = x + 1 Supondo que φ ( x) = ax + b todo x ∈ R,

e ψ ( x) = cx + d ,

devemos ter, para

φ D ϕ ( x) = acx + ad + b = x + log 2 3 ψ D φ ( x) = acx + d + bc = x + 1 Podemos escolher, por exemplo, a = log 2 3, b = 0, c = log 3 2 e d = 1. (ou seja, φ ( x) = x log 2 3 e ψ ( x ) = x log 3 2 + 1 De (A), temos x

g (2 2 ) = 2 2

φ ( x)

φ (log2 (log2 x ))

⇒ g ( x) = 2 2

= 22

EUREKA! N° 3, 1998

12

log 2 3. log 2 log 2 x

= 2 log 2 x

log 2 3


Sociedade Brasileira de Matemática

e de (B) x

f (2 2 ) = 2 2

ψ (x)

ψ (log2 (log2 x ))

⇒ f ( x) = 2 2

1+ log2 log2 x . log3 2

= 22

= 2 2 log 2 x

log3 2

É fácil verificar que as funções acima estão definidas em (1, ∞ ) e satisfazem as condições do enunciado. Elas fornecem, portanto, uma possível solução para o item b).

4) Primeiramente, notemos que, para n ≥ 0, Fn Fn + 2 − Fn2+1 = (−1) n +1 . De fato,

F1 F3 − F22 = 1 ⋅ 2 − 12 = (−1) 2 e por indução supondo que Fn Fn + 2 − Fn2+1 = ( −1) n +1 temos que

Fn +1 Fn + 3 − Fn2+ 2 = Fn +1 ( Fn + 2 + Fn +1 ) − Fn2+ 2 = Fn2+1 − Fn + 2 ( Fn + 2 − Fn +1 ) = − ( Fn Fn + 2 − Fn2+1 ) = −(−1) n +1 = (−1) n + 2 . Dividimos o problema em dois casos; indicados pelas seguintes figuras: (i)

Fn

Vn Vn+2

Fn+2 Fn+1

Vn+1 Fn+2

Fn+4

Fn+3

Se A é a área do triângulo sombreado, de lados Vn, Vn+1 e Vn+2, temos

A=

1 2

Fn + 2 Fn + 4 2

=

1 2

+

Fn Fn + 2 2

+ Fn +1 Fn + 2 +

Fn +1 Fn +3 2

Fn+2 Fn+4 = 1+ Fn+2 (Fn + 2Fn+1 ) + Fn+1Fn+3 = 1+ Fn+2 (Fn+2 + Fn+1 ) + Fn+1Fn+3 = EUREKA! N° 3, 1998

13


Sociedade Brasileira de Matemática

= 1 + Fn + 2 Fn +3 + Fn +1 Fn + 3 = 1 + Fn2+3 ⇔ Fn + 2 Fn + 4 − Fn2 +3 = 1, o que ocorre sempre que n é ímpar. (ii) Vn

Fn

Vn+1

Fn+2

Vn+2 Fn+1 Fn+2

Fn+3

Fn+4

Se A é a área do triângulo sombreado, de lados Vn, Vn+1 e V n+2, temos analogamente que

F F F F 1 1 FF ⇔ n + 2 n + 4 = − + n n + 2 + Fn +1 Fn + 2 + n +1 n +3 ⇔ 2 2 2 2 2 Fn + 2 Fn + 4 − Fn2+3 = −1, o que ocorre sempre que n é ímpar. A=

Em qualquer dos casos, temos que a área do triângulo de lados Vn, Vn+1 e 1 Vn+2 é . 2

5) Se x > c + 1, então 2

x 2 − x = x ( x − 1) > c + c ≥ c ⇒ x 2 + c ≥ x 2 − c > x e, portanto,

f

n +1

( x) > f n ( x) > c + 1 para todo n ≥ 0. Logo, se x é pré-períodico,

então x ≤ c + 1 (*). Agora, sejam c =

r

, onde ( r , s ) = 1, e x =

s com p, q, r , s ∈ Z e q, s > 0. Temos EUREKA! N° 3, 1998

14

p q

, onde ( p, q ) = 1,


Sociedade Brasileira de Matemática

s( x 2 + c) = Se x 2 + c =

sp 2 q2

+r

u , u , v ∈ Z, v ≠ 0, então v

q2 su sp 2 = 2 + r ⇒ svp 2 = q 2 (su − rv) ⇒ q 2 svp2 ⇒ q 2 sv ⇒ sv ≥ q 2 ⇒v ≥ . v s q Se q > s, o denominador v de x 2 + c é maior ou igual a denominador de x isto é, o denominador de

q2

> q, que é o s f n+1 ( x ) é maior que o

denominador de f n ( x ), ∀n ≥ 0, e, portanto, se x é pré-periódico, então seu denominador é no máximo s (**). De (*) e (**), segue que há apenas um número finito de pontos préperiódicos racionais.

6) Em primeiro lugar, observe que as imagens dos vértices de um triângulo equilátero de lado 1 formam também um triângulo eqüilátero de lado 1. Assim, dados dois triângulos eqüiláteros de lado 1 com um lado em comum, os vértices opostos ao lado comum podem ter mesma imagem ou imagens diferentes distando AA' =

3. Em outras palavras, se A e A' são pontos tais que

3. então d ( f ( A), f ( A' )) ∈ {0, 3}. Vamos mostrar que, de fato,

d ( f ( A), f ( A' )) = 3. Se f ( A) = f ( A' ), então tomando B com AB = 1 e A' B = 3 , teríamos d ( f ( A), f ( B)) = 1 ⇔ d ( f ( A' ), f ( B)) = 1, o que seria absurdo. Assim, d(A,A' ) = d( f (A), f (A' )) = 3 ⇒d( f (A), f (A' )) = 3. Desta forma qualquer reticulado triangular formado por vértices de triângulos eqüiláteros de lado 1 de interiores disjuntos e cobrindo o plano é preservado por f, no seguinte sentido: a imagem deste reticulado também será outro reticulado do mesmo tipo. Em particular, pontos a distância n são levados em pontos também à distância n, n ∈ N. Este último fato mostra que triângulos de lados 1, n 2 + n − 1 que têm área

n 2 − n +1

e

3 / 4 são preservados pela função f , já que seus vértices estão em reticulado triangular de lado 1.

EUREKA! N° 3, 1998

15


Sociedade Brasileira de Matemática

1

B 1

C 1

A n AB = n 2 − n + 1 AC = n 2 + n + 1

Utilizando um procedimento análogo ao anterior, vamos agora considerar a imagem dos vértices de dois triângulos deste tipo com o lado de medida

n 2 + n + 1 em comum. Sendo X e Y os vértices destes triângulos opostos ao lado comum, temos novamente que XY = ε n ⇒ d ( f ( X ), f (Y )) = 0 ou

d ( f ( X ), f (Y )) = XY = ε n , onde εn =

3 n2 + n +1

é o dobro da altura dos triângulos considerados em relação ao lado comum. Vamos demonstrar que os pontos à distância ε n têm, de fato, imagens distintas. Seja kn tal que k n ε n < 1 ≤ ( k n + 1)ε n .

d ( A0 , A1 ) = ε n , considere pontos para d ( Ai , Ai +1 ) = ε n 0 ≤ i ≤ kn e

Sendo

Ai , 2 ≤ i ≤ k n + 1

tais

d ( A0 , Akn +1 ) = 1

que

Temos

d ( f ( A0 , f ( Akn +1 )) = 1 e, portanto, kn

1 ≤ ∑ d ( f ( Ai ), f ( Ai +1 )) ≤ d ( f ( A0 ), f ( A1 )) + k n ε n , i =0

Se d ( f ( A0 ), f ( A1 )) fosse 0, então 1 ≤ k n ε n < 1, o que seria absurdo assim,

XY = ε n ⇒ d ( f ( X ), f (Y )) = ε n .

Como

kε n ⇒ d ( f ( X ), f (Y )) = kε n para k ∈ N.

EUREKA! N° 3, 1998

16

antes,

temos

que

XY

=


Sociedade Brasileira de Matemática

Agora, suponha que existam X e Y tais que d ( f ( X ), f (Y )) ≠ d ( X , Y ). Sejam n ∈ N tal que 4 ∈n < d ( f ( X ), f (Y )) − d ( X , Y ) e P ∈ R 2 com

d ( P, X )

εn

∈ N, d ( P, Y ) < 2ε n .

Tome Q ∈ R 2 com d ( P, Q ) = d (Y , Q ) = ε n ⇒ d ( f ( P ), f (Q )) =

d ( f (Y ), f (Q )) = ε n ⇒ d ( f ( P ), f (Y )) ≤ 2ε n e como d ( P, X ) = d ( f ( P), f ( X )), temos d ( f ( X ), f (Y )) − d ( X , Y ) ≤ d ( f ( X ), f (Y )) − d ( f ( X ), f ( P)) +

d ( X , P) − d ( X , Y ) ≤ d ( f (Y ), f ( P)) + d ( P, Y ) < 4ε n , absurdo. Obs: As funções f : R2 → R2 que satisfazem as condições do enunciado são chamadas isometrias, e são composições de translações com rotações e / ou reflexões.

Você sabia…

Que o número de pessoas que já

apertaram a mão de outras pessoas um número ímpar de vezes é par

??

EUREKA! N° 3, 1998

17


Sociedade Brasileira de Matemática

13a. OLIMPÍADA IBEROAMERICANA DE MATEMÁTICA República Dominicana 1998 - Problemas e resultados Primeiro dia Duração da Prova: 4 h e 30 minutos. PROBLEMA 1

São dados 98 pontos sobre uma circunferência. Maria e José jogam alternadamente da seguinte maneira: cada um deles traça um segmento unindo dois dos pontos dados que não tenham sido unidos entre si anteriormente. O jogo termina quando os 98 pontos tenham sido usados como extremos de um segmento pelo menos uma vez. O vencedor é a pessoa que faz o último traço. Se o José começa o jogo, quem pode garantir a sua própria vitória? PROBLEMA 2

A circunferência inscrita no triângulo ABC é tangente aos lados BC, CA e AB nos pontos D, E e F, respectivamente. AD corta a circunferência num segundo ponto Q. Demonstrar que a reta EQ passa pelo ponto médio de AF se e somente se AC = BC . PROBLEMA 3

Encontrar o menor número natural n com a seguinte propriedade: entre quaisquer n números distintos do conjunto {1, 2, …, 999} pode-se escolher quatro números diferentes a, b, c, d, tais que a + 2b + 3c = d. Segundo dia Duração da Prova: 4 h e 30 minutos. PROBLEMA 4

Em volta de uma mesa redonda estão sentados representantes de n países (n ≥ 2), satisfazendo a seguinte condição: se duas pessoas são do mesmo país, então, seus respectivos vizinhos da direita não podem ser de um mesmo país. Determinar, para cada n, o número máximo de pessoas que pode haver em volta da mesa. EUREKA! N° 3, 1998

18


Sociedade Brasileira de Matemática

PROBLEMA 5

Encontrar o maior valor possível n para que existam pontos distintos P1, P2, P3, … , Pn no plano, e números reais r1, r2, … , rn de modo que a distância entre quaisquer dois pontos diferentes Pi e Pj seja ri + rj. PROBLEMA 6

Seja λ a raiz positiva da equação t2 − 1998t − 1 = 0. Define-se a sucessão x0, x1, x2, … , xn , … por:  xo = 1   x n +1 = [λx n ], para n = 0,1,2,... Encontrar o resto da divisão de x1998 por 1998.

Nota: [x] indica a parte inteira de x, ou seja, [x] é o único inteiro k tal que k ≤ x < k + 1. RESULTADOS A equipe Brasileira teve uma excelente participação na 13a. Olímpíada Iberoamericana de Matemática realizada em República Dominicana de 18 a 27 de setembro na qual participaram 18 países. Os países que obtiveram maior soma de pontos foram:

BRASIL CHILE ARGENTINA PERU MÉXICO ESPANHA

132 pontos 127 pontos 120 pontos 117 pontos 115 pontos 112 pontos

O Resultado da Equipe Brasileira BRA 1 BRA 2 BRA 3 BRA 4

Murali Srinivasam Vajapeyam Emanuel Augusto de Souza Carneiro Fabricio Shigueu Catae Mauricio Pereira Carrari

Cada um dos seis problemas da prova vale 7 pontos. EUREKA! N° 3, 1998

19

Prata - 32 pontos Ouro - 37 pontos Ouro - 35 pontos Bronze - 28 pontos


Sociedade Brasileira de Matemática

39a. OLIMPÍADA INTERNACIONAL DE MATEMÁTICA Soluções PROBLEMA 1

No quadrilátero convexo ABCD, as diagonais AC e BD são perpendiculares e os lados opostos AB e DC não são paralelos. Sabemos que o ponto P, onde se intersectam as mediatrizes de AB e DC, está no interior de ABCD. Prove que ABCD é um quadrilátero inscritível se, e somente se, os triângulos ABP e CDP têm áreas iguais.

SOLUÇÃO

Suponha primeiro que ABCD ∩

seja inscritivel. Como AC ⊥ BD temos

AB + CD = π . Claramente o centro O do círculo circunscrito pertence às mediatrizes de AB e DC, logo P = O, e como área de

OAB =

∩ ∩ 1 1 2 r sen AB = r 2 sen CD = área de OCD (onde r é o raio do 2 2

círculo), vale a primeira implicação. Suponha agora que ABCD não seja inscritível. Suponha sem perda de generalidade que PC < PA. Seja Q o ponto de interseção de AC e BD. Prolongamos QC e QD até intersectarmos o círculo de centro p e raio PA = PB em novos pontos C’ e D’ . Como AC’ e BD’ são perpendiculares, pela primeira implicação sabemos que área de PAB = área de PC’D’, mas C’D’ > CD ( C’D’ é hipotenusa do triângulo retângulo QC’D’, de catetos maiores que o triângulo retângulo QCD, do qual CD é hipotenusa), e d(P, C’D’) > d (P, CD) (de fato, C’ e D’ estão no mesmo semiplano determinado pela reta CD , distinto do semiplano ao qual pertence P, e d (P, C’D’) = d(P, M), onde M é o ponto médio de C’D’, e portanto pertence ao mesmo semiplano que C’ e D’ , logo d(P, CD) < d(P, M) = = d (P, C‘D’ )). Portanto área de PC’D’ > área de PCD, absurdo, pois estamos supondo que área de PAB = área de PCD.

PROBLEMA 2

Numa competição, existem a concorrentes e b juízes, onde b ≥ 3 é um inteiro ímpar. Cada juiz avalia cada um dos concorrentes, classificando-o como "aprovado" ou "reprovado". Suponha que k é um número tal que as EUREKA! N° 3, 1998

20


Sociedade Brasileira de Matemática

classificações dadas por dois juízes quaisquer coincidem no máximo para k k b −1 . concorrentes. Prove que ≥ a 2b SOLUÇÃO

Para cada um dos candidatos, se j é o número de juizes que o aprovam, o número de pares de juízes que tem julgamentos coincidentes em relação a ele é C 2j + C b2− j ≤ C b2+1 + C b2−1 = 2

2

(b − 1) 2 , de modo que o número total de 4

a(b − 1) 2 , que, por outro 4 b(b − 1) . Assim, lado, por hipótese, deve ser no máximo k ⋅ C b2 = k 2 (b − 1) a(b − 1) 2 k b −1 . devemos ter kb ≥ ⇒ ≥ 2 4 2b a pares de julgamentos coincidentes é no máximo

PROBLEMA 3

Para qualquer inteiro positivo n, seja d(n) o número de divisores positivos de n (incluindo 1 e n). Determine todos os inteiros positivos k tais que

d (n 2 ) = k para algum n. d ( n)

SOLUÇÃO α

Obsevemos inicialmente que se n = p1α1 p 2α 2 ... p k k ( pi primos distintos) então d (n) = (1 + α 1 )(1 + α 2 )...(1 + α k ). Assim, d ( n 2 ) / d ( n) =

(1 + 2α 1 )(1 + 2α 2 )...(1 + 2α k )

. Como o numerador é (1 + α 1 )(1 + α 2 )...(1 + α k ) ímpar, se o resultado for inteiro deve ser ímpar (e todos os αi devem ser pares). Vamos mostrar que qualquer número natural ímpar é da forma desejada. Para isso, devemos mostrar que todo número ímpar pode ser escrito como 2r + 1 produto de frações da forma , r ∈ N, não necessariamente distintas. r +1 Faremos isso por indução. Seja m um número ímpar, e seja 2s a maior EUREKA! N° 3, 1998

21


Sociedade Brasileira de Matemática

potência de 2 que divide m + 1. Temos portanto m = 2 s +1 q + 2 s − 1 para algum q ∈ N, donde m(2 s − 1) 2 2 s (2q + 1) − 2 s +1 (q + 1) + 1 2 2 s (2q + 1) − 2 s +1 (q + 1) + 1 m= × = = 2 s −1 2s −1 2s −1 2 (2q + 1) − 2 s (q + 1) + 1

2 2 s −1 (2q + 1) − 2 s (q + 1) + 1 2 2 s +1 (2q + 1) − 4(q + 1) + 1 × ⋅ ⋅ ⋅ × × (2q + 1). 2 2 s − 2 (2q + 1) − 2 s −1 (q + 1) + 1 2 s (2q + 1) − (2q + 1) Como 2q + 1 < 2s + 1 q + 2s – 1 = m, por hipótese de indução, 2q + 1 se 2r + 1 , e portanto m também. escreve como produto de frações da forma r +1 ×

PROBLEMA 4

Determine todos os pares (a, b) de inteiros positivos tais que ab2 + b + 7 divide a2b + a + b. SOLUÇÃO

a 2b + a + b é inteiro então ab 2 + b + 7 b(a 2 b + a + b) − a (ab 2 + b + 7) b 2 − 7a = é inteiro. Como ab 2 + b + 7 ab 2 + b + 7 b 2 − 7a b 2 − 7a Se b 2 − 7 a < b 2 < ab 2 + b + 7 temos que < 1 . =0 ab 2 + b + 7 ab 2 + b + 7 teremos b2 = 7a, donde b é múltiplo de 7 (digamos b = 7t ), e (7t)2 = 7a nos dá a = 7t2. É fácil ver que (a, b) = (7t2, 7t) satisfaz as condições do enunciado para todo t inteiro positivo (temos nesse caso a 2b + a + b = t ). ab 2 + b + 7 b 2 − 7a b 2 − 7a 2 Se < 0 devemos ter b < 7a e ≤ −1 (pois é inteiro), e ab 2 + b + 7 ab 2 + b + 7 portanto 7a > 7a − b2 ≥ ab2 + b + 7 ⇒ 7a > ab2 ⇒ b2 < 7 ⇒ b = 1 ou b = 2. b 2 − 7a 1 − 7a 57 = = −7 + Se b = 1, 2 , e devemos ter que a + 8 divide a+8 ab + b + 7 a + 8 57, com a inteiro positivo ⇒ a + 8 = 19 ou a + 8 = 57 ⇒ a = 11 ou Se

EUREKA! N° 3, 1998

22


Sociedade Brasileira de Matemática

a = 49. Para a = 11 e b = 1 temos b = 1 temos

a 2 b + a + b 133 = = 7, e para a = 49 e ab 2 + b + 7 19

a 2 b + a + b 2451 = = 43. 57 ab 2 + b + 7

4 − 7a b 2 − 7a Se b = 2, = . Como 4 – 7a > – 18 – 8a = – 2 (4a + 9), se 2 ab + b + 7 4a + 9 4 − 7a

é inteiro negativo, devemos ter

4a + 9 4 − 7a 4a + 9

= −1 ⇒ 4 − 7 a = −4 a − 9 ⇒ a =

13 3

∉ N.

Assim, as soluções são dadas por (a, b) = (7t 2 ,7t ), t ∈ N; (a, b) = ( 11,1 ) e (a, b) = (49,1).

PROBLEMA 5

Seja I o incentro do triângulo ABC. A circunferência inscrita no triângulo ABC é tangente aos lados BC, CA e AB nos pontos K, L e M, respectivamente. A reta que passa por B, paralela ao segmento MK, intersecta as retas LM e LK nos pontos R e S, respectivamente. Prove que o ângulo ∠RIS é agudo.

SOLUÇÃO

__ __ __ Como BM = BK e BI é bissetriz de MBˆ K temos BI ⊥ MK , e portanto __

__

BI ⊥ RS . Queremos mostrar que RIˆS é agudo, o que é equivalente a 2 2 2 RI + SI > RS , o que equivale a 2

2

2

BR + BI + BS + BI

2

2

2

> ( BR + BS ) 2 = BR + 2 BR BS + BS ,

e portanto devemos provar que

BI

2

> BR BS .

π − Bˆ Se Aˆ = BAˆ C , B = ABˆ C e C = BCˆ A, temos KBˆ S = MBˆ R = , 2 EUREKA! N° 3, 1998

23


Sociedade Brasileira de Matemática

KSˆB =

π − Aˆ 2

(e portanto

SKˆ B =

π − Cˆ 2

) e MRˆ B =

π − Cˆ 2

(e portanto

π − Aˆ ). Assim, os triângulos MBR e SBK são semelhantes e BMˆ R = 2 BR BM

=

BK BS

, donde

BR BS = BM BK = BM

2

< BI

2

(pois

BI

é

hipotenusa do triângulo retângulo BMI ).

PROBLEMA 6

Considere todas as funções f definidas no conjunto N dos inteiros positivos, com valores no mesmo conjunto, que satisfazem f (t 2 f ( s )) = s ( f (t )) 2 , para todos s e t em N. Determine o menor valor possível de f(1998)

SOLUÇÃO

Dizemos que h : N → N é estritamente multiplicativa se h(xy) = h(x) h(y), para quaisquer x, y ∈ N, e dizemos que h é uma involução se h(h(x)) = x para todo x ∈ N. É facil ver que se f satisfaz a involução estritamente multiplicativa então f satisfaz a condição do enunciado: f (t2 f (s)) = (f (t)2 f (f (s)) = s (f (t))2. Podemos definir f : N → N estritamente multiplicativa por

f ( p1α1 p 2α 2 ... p αk k ) = f ( p1 ) α1 ... f ( p k )α k ( pi primos distintos), onde f (2) = 3, f (3) = 2, f (37) = 5, f (5) = 37 e f (p) = p, para todo p primo não pertencente a {2, 3, 5, 37}, e teremos f (1998) = f ( 2 ⋅ 33 ⋅ 37 ) = f (2) f (3)3 f (37) = 3 ⋅ 23 ⋅ 5 = 120. Vamos provar que 120 é menor valor possível para f (1998). Fazendo t = 1 temos f (f (s )) = s f (1)2, ∀ s ∈ N. Em particular, f é injetiva, pois f (s) = f (u) ⇒ f (f (s)) = f ( f (u)) ⇒ s f (1)2 = u f (1)2 ⇒ s = u. Temos ainda f (t 2 f (1)) = f (t)2 para todo t ∈ N ( fazendo s = 1), e portanto temos f (t2 f (s) 2) = f (t2 f (s2 f (1))) = s2 f (1) f (t)2, e fazendo s = f (u) temos f (t2 ( f ( f (u))2)= f (u)2 f (1) f (t)2. Assim, provamos que f (t 2 u2 f (1)4) = = ( f (u) f (t))2 f (1), para quaisquer u, t ∈ N. Portanto, se ut = xy, f (t 2 u2 f (1)4) = f (x2 y2 f (1)4), logo ( f (u) f (t))2 f (1) = ( f (x) f (y))2 f (1) ⇒ f (u) f (t) = f (x) f (y). Como x2 ⋅ 1 = x ⋅ x, f (x2) f (1) = EUREKA! N° 3, 1998

24


Sociedade Brasileira de Matemática

f (x)2, ∀x ∈ N. Se pk é uma potência de primo que divide f (1), e p r é a maior potência de p que divide f (x) para todo x ∈ N, temos que f (x)2 é múltiplo de  r +k   2  

pr ⋅ pk ⇒ f (x) é múltiplo de p 

onde α  denota o menor inteiro que é

maior ou igual a α, para todo x ∈ N, o que é absurdo se r < k (pois teríamos

r + k  k  2  > r ). Logo p divide f (x) para todo x ∈ N, e portanto f (1) divide f (x), para todo x ∈ N. Como xy ⋅ 1 = x ⋅ y , f (xy) f (1) = f (x) f (y) ⇒ f ( xy ) f ( x ) f ( y ) f ( x) temos que g é = ⋅ . Definindo g : N → N, g (x) = f (1) f (1) f (1) f (1) estritamente multiplicativa, g é injetiva, g (1) = 1 e g (x) ≤ f (x) para todo x ∈ N. Temos g(1998) = g(2 ⋅ 33 ⋅ 37) = g(2) g(3)3 g(37). Observemos agora que g(2), g(3) e g(37) devem ser naturais distintos maiores que 1, e não podemos ter {2, 4} ⊂ {g(2), g(3), g(37)}, pois se g( p) = 2 e g(q) = 4 com {p, q} ⊂ {2, 3, 37} teríamos g(p2) = g(p)2 = g(q) ⇒ p2 = q, absurdo. Assim g(1998) = g(2) g(3)3 g(37) = g(2) g(3) g(37) g(3)2 ≥ 2 ⋅ 3 ⋅ 5 ⋅ g(3)2 ≥ 2 ⋅ 3 ⋅ 5 ⋅ 22 = 120, logo f (1998) ≥ 120, como afirmamos ❑

Você sabia…

Que o matemático Harald Bohr (irmão do físico Niels Bohr), famoso por sua teoria das funções quase-periódicas, era um consagrado jogador de futebol? Ele jogou no meio-de-campo da seleção da Dinamarca, que ganhou a medalha de prata nos jogos Olímpicos de Londres, em 1908, quando seu time derrotou a França pela contagem de 17 a 1

(!).

EUREKA! N° 3, 1998

25


Sociedade Brasileira de Matemática

O PRINCÍPIO DA INDUÇÃO Elon Lages Lima ♦ Nível Avançado. INTRODUÇÃO

O Princípio da Indução é um eficiente instrumento para a demonstração de fatos referentes aos números naturais. Por isso deve-se adquirir prática em sua utilização. Por outro lado, é importante também conhecer seu significado e sua posição dentro do arcabouço da Matemática. Entender o Princípio da Indução é praticamente o mesmo que entender os números naturais. Apresentamos abaixo uma breve exposição sobre os números naturais, onde o Princípio da Indução se insere adequadamente e mostra sua força teórica antes de ser utilizado na lista de exercícios propostos ao final. 1. A SEQÜÊNCIA DOS NÚMEROS NATURAIS

Os números naturais constituem um modelo matemático, uma escala padrão, que nos permite a operação de contagem. A seqüência desses números é uma livre e antiga criação do espírito humano. Comparar conjuntos de objetos com essa escala abstrata ideal é o processo que torna mais precisa a noção de quantidade; esse processo (a contagem) pressupõe portanto o conhecimento da seqüência numérica. Sabemos que os números naturais são 1, 2, 3, 4, 5,… A totalidade desses números constitui um conjunto, que indicaremos com o símbolo N e que chamaremos de conjunto dos naturais. Portanto N = {1, 2, 3, 4, 5,…}. Evidentemente, o que acabamos de dizer só faz sentido quando já se sabe o que é um número natural. Façamos de conta que esse conceito nos é desconhecido e procuremos investigar o que há de essencial na seqüência 1, 2, 3, 4, 5… . Deve-se a Giussepe Peano (1858-1932) a constatação de que se pode elaborar toda a teoria dos números naturais a partir de quatro fatos básicos, conhecidos atualmente como os axiomas de Peano. Noutras palavras, o conjunto N dos números naturais possui quatro propriedades fundamentais, das quais resultam, como conseqüências lógicas, todas as afirmações verdadeiras que se podem fazer sobre esses números. Começaremos com o enunciado e a apreciação do significado dessas quatro proposições fundamentais a respeito dos números naturais. EUREKA! N° 3, 1998

26


Sociedade Brasileira de Matemática

2. OS AXIOMAS DE PEANO

Um matemático profissional, em sua linguagem direta e objetiva, diria que o conjunto N dos números naturais é caracterizado pelas seguintes propriedades:

A. B. C. D.

Existe uma função s : N → N, que associa a cada n ∈ N um elemento s(n) ∈ N, chamado o sucessor de n. A função s : N → N é injetiva. Existe um único elemento 1 no conjunto N, tal que 1 ≠ s(n) para todo n ∈ N. Se um subconjunto X ⊂ N é tal que 1 ∈ N e s(X) ⊂ X (isto é, n ∈ X ⇒ s(n) ∈ X), então X = N.

Observe que, como estamos chamando de N o conjunto dos números naturais, a notação n ∈ N significa que n é um número natural. As afirmações A, B, C e D são os axiomas de Peano. A notação s(n) é provisória. Depois de definirmos adição, escreveremos n + 1 em vez de s(n). Como concessão à fraqueza humana, nosso matemático nos faria a gentileza de reformular os axiomas de Peano em linguagem corrente, livre de notação matemática. E nos diria então que as afirmações acima significam exatamente o mesmo que estas outras:

A'. B'. C'. D'.

Todo número natural possui um único sucessor, que também é um número natural. Números naturais diferentes possuem sucessores diferentes. (Ou ainda: números que têm o mesmo sucessor são iguais.) Existe um único número natural que não é sucessor de nenhum outro. Este número é representado pelo símbolo 1 e chamado de "número um". Se um conjunto de números naturais contém o número 1 e, além disso, contém o sucessor de cada um de seus elementos, então esse conjunto coincide com N, isto é, contém todos os números naturais.

A partir daí, retomamos a palavra para dizer que o sucessor de 1 chama-se "dois", o sucessor de dois chama-se "três", etc. Nossa civilização progrediu ao ponto em que temos um sistema de numeração, o qual nos EUREKA! N° 3, 1998

27


Sociedade Brasileira de Matemática

permite representar, mediante o uso apropriado dos símbolos 0, 1, 2, 3, 4, 5, 6, 7, 8 e 9, todos os números naturais. Além disso, nossa linguagem também fornece nomes para os primeiros termos da seqüência dos números naturais. (Números muito grandes não têm nomes específicos, ao contrário dos menores como "mil novecentos e noventa e oito". Quem sabe, por exemplo, o nome do número de átomos do universo?) Voltando a usar a notação s(n) para o sucessor do número natural n, teremos então 2 = s(1), 3 = s(2), 4 = s(3), 5 = s(4), etc. Assim, por exemplo, a igualdade 2 = s(1) significa apenas que estamos usando o símbolo 2 para representar o sucessor de 1. A seqüência dos números naturais pode ser indicada assim: s s s s s 1 → 2 → 3 → 4 → 5 → ⋅⋅⋅ As flechas ligam cada número ao seu sucessor. Nenhuma flecha aponta para 1, pois este número não é sucessor de nenhum outro. O diagrama acima diz muito sobre a estrutura do conjunto N dos números naturais. 3. O AXIOMA DA INDUÇÃO

Um dos axiomas de Peano, o último, possui claramente uma natureza mais elaborada do que os demais. Ele é conhecido como o axioma da indução. Faremos dele uma análise detida, acompanhada de comentários. O significado informal do axioma D é que todo número natural pode ser obtido a partir de 1 por meio de repetidas aplicações da operação de tomar o sucessor. Assim, por exemplo, 2 é o sucessor de 1, 3 é o sucessor do sucessor de 1, etc. Para se entender melhor o axioma da indução é util examinar o exemplo, no qual N = {1, 2, 3,…} mas a função s : N → N é modificada, pondo-se s(n) = n + 2. Então, se começarmos com 1 e a este número aplicarmos repetidamente a operação de tomar o "sucessor" (nesta nova acepção) obteremos s(1) = 3, s(3) = 5, s(5) = 7, etc., e nunca chegaremos a qualquer número par. Portanto, o diagrama s s s s s s 1 → 3 → 5 → ⋅⋅⋅ 2  → 4 → 6 → ⋅⋅⋅

exibe uma função injetiva s : N → N para a qual não é verdade que todo número natural n pode ser obtido, a partir de 1, mediante repetidas aplicações da operação de passar de k para s(k). EUREKA! N° 3, 1998

28


Sociedade Brasileira de Matemática

Dentro de um ponto de vista estritamente matemático, podemos reformular o axioma da indução do seguinte modo: Um subconjunto X ⊂ N chama-se indutivo quando s(X) ⊂ X, ou seja, quando n ∈ X ⇒ s(n) ∈ X, ou ainda, quando o sucessor de qualquer elemento de X também pertence a X. Dito isto, o axioma da indução afirma que o único subconjunto indutivo de N que contém o número 1 é o proprio N. No exemplo acima, os números ímpares 1, 3, 5, … formam um conjunto indutivo que contém o elemento 1 mas não é igual a N. O papel fundamental do axioma da indução na teoria dos números naturais e, mais geralmente, em toda a Matemática, resulta do fato de que ele pode ser visto como um método de demonstração, chamado o Método de Indução Matemática, ou Princípio da Indução Finita, ou Princípio da Indução, conforme explicaremos agora. Seja P uma propriedade que se refere a números naturais. Um dado número natural pode gozar ou não da propriedade P. Por exemplo, seja P a propriedade de um número natural n ser sucessor de outro número natural. Então 1 não goza da propriedade P, mas todos os demais números gozam de P. O Princípio da Indução diz o seguinte:

Princípio da Indução: Seja P uma propriedade referente a números naturais. Se 1 goza de P e se, além disso, o fato de o número natural n gozar de P implica que seu sucessor s(n) também goza, então todos os números naturais gozam da propriedade P. Para ver que o Princípio da Indução é verdadeiro (uma vez admitidos os axiomas de Peano) basta observar que, dada a propriedade P cumprindo as condições estipuladas no enunciado do Princípio, o conjunto X dos números naturais que gozam da propriedade P contém o número 1 e é indutivo. Logo X = N, isto é, todo número natural goza da propriedade P. As propriedades básicas dos números naturais são demonstradas por indução. Comecemos com um exemplo bem simples. Exemplo 1. Entre afirmação de que provaremos agora. número natural n, afirmação n ≠ s(n).

os axiomas de Peano não consta explicitamente a todo número é diferente do seu sucessor, a qual Seja P esta propriedade. Mais precisamente, dado o escrevamos P(n) para significar, abreviadamente, a Então P(1) é verdadeira, pois 1 ≠ s(1), já que 1 não é

EUREKA! N° 3, 1998

29


Sociedade Brasileira de Matemática

sucessor de número algum; em particular, 1 não é sucessor de si próprio. Além disso, se supusermos P(n) verdadeira, isto é, se admitimos que n ≠ s(n), então s(n) ≠ s(s(n)), pois a função s : N → N é injetiva. Mas a afirmação s(n) ≠ s(s(n) significa que P(s(n)) é verdadeira. Assim, a verdade de P(n) acarreta a verdade de P(s(n)). Pelo Princípio da Indução, todos os números naturais gozam da propriedade P, ou seja, são diferentes de seus sucessores. Nas demonstrações por indução, a hipótese de que a propriedade P é válida para o número natural n (da qual deve decorrer que P vale também para s(n)) chama-se hipótese de indução. O Princípio da Indução não é utilizado somente como método de demonstração. Ele serve também para definir funções f: N → Y que têm como dominio o conjunto N dos números naturais. Para se definir uma função f : X → Y exige-se em geral que seja dada uma regra bem determinada, a qual mostre como se deve associar a cada elemento x ∈ X um único elemento y = f(x) ∈ Y. Entretanto, no caso particular em que o domínio da função é o conjunto N dos números naturais, a fim de definir uma função f : N → Y não é necessário dizer, de uma só vez, qual é a receita que dá o valor f(n) para todo n ∈ N. Basta que se tenha conhecimento dos seguintes dados: (1) O valor f (1); (2) Uma regra que permita calcular f (s(n)) quando se conhece f (n). Esses dois dados permitem que se conheça f (n) para todo número natural n. (Diz-se então que a função f foi definida por recorrência.) Com efeito, se chamarmos de X o conjunto dos números naturais n para os quais se pode determinar f (n), o dado (1) acima diz que 1 ∈ X e o dado (2) assegura que n ∈ X ⇒ s(n) ∈ X. Logo, pelo axioma da indução, tem-se X = N. Obs. : Uma função f : N → Y cujo domínio é o conjunto dos números naturais chama-se uma seqüência ou sucessão de elementos de Y. A notação usada para uma tal seqüência é (y1, y2,…,yn,…), onde se usa yn em vez de f(n) para indicar o valor da função f no número n. O elemento yn . 4. ADIÇÃO E MULTIPLICAÇÃO DE NÚMEROS NATURAIS

A adição e a multiplicação de números naturais são exemplos de funções definidas por recorrência. EUREKA! N° 3, 1998

30


Sociedade Brasileira de Matemática

Para definir a adição, fixaremos um número natural arbitrário k e definiremos a soma k + n para todo n ∈ N. Fixado k, a correspondência n → k + n será uma função f: N→ N, f(n) = k + n, chamada "somar k". Ela se define por recorrência, a partir dos seguintes dados: (S1) k + 1 = s(k) (S2) k + s(n) = s(k + n). Portanto, k + 1 é, por definição, o sucessor de k. E, se conhecermos k + n, saberemos o valor de k + s(n): por definição, tem-se k + s(n) = s(k + n). Isto nos permite conhecer k + n para todo n ∈ N (e todo k ∈ N). Usando as notações definitivas n + 1 em vez de s(n) e (k + n) + 1 em vez de s(k + n), a igualdade (S2) se escreve assim: (S2') k + (n + 1) = (k + n) +1. Assim, as igualdades (S1) e (S2) ou, equivalentemente, (S1) e (S2') definem por recorrência a soma k + n de dois números naturais quaisquer k e n. A multiplicação de números naturais se define de modo análogo à adição. Fixado arbitrariamente um número natural k, a multiplicação por k associa a todo número mnatural n o produto n ⋅ k, definido por indução da seguinte maneira: (P1) 1⋅ k = k. (P2) (n + 1) k = n⋅k + k. O produto n⋅k escreve-se também nk e lê-se "n vezes k". A definição acima diz portanto que uma vez k é igual a k e n + 1 vezes k é igual a n vezes k mais (uma vez) k . Assim, por definição, 2 ⋅ k = k + k, 3 ⋅ k = k + k + k, etc. Usa-se indução para provar as propriedades básicas da adição e da multiplicação de números naturais. Entre elas, destacam-se as seguintes, válidas para quaisquer k, n, p ∈ N: Associatividade: k + (n + p) = (k + n) + p e k ⋅ (n ⋅ p) = (k ⋅ n)⋅ p Comutatividade: k+n=n+k e k⋅n=n⋅k Lei do Corte: k+n=k+p⇒n=p e k⋅n=k⋅p⇒n=p Distributividade: k (n + p) = k ⋅ n + k ⋅ p. Omitiremos as demonstrações destes fatos. O leitor pode considerálas como exercícios sobre o método da indução. 5. ORDEM

A adição de números naturais permite introduzir uma relação de ordem em N. Dados os números naturais m, n diremos que m é menor do que EUREKA! N° 3, 1998

31


Sociedade Brasileira de Matemática

n, e escreveremos m < n, para significar que existe p ∈ N tal que n = m + p. Neste caso, diz-se também que n é maior do que m e escreve-se n > m para exprimir que se tem m < n. A notação m ≤ n significa que m < n ou m = n. Por definição, tem-se portanto m < m + p para quaisquer m, p ∈ N. Em particular, m < m + 1. Segue-se também da definição que 1 < n para todo número natural n ≠ 1. Com efeito, pelo axioma C, n ≠ 1 implica que n é sucessor de algum número natural m, ou seja, n = m + 1 = 1 + m, logo n > 1. Assim, 1 é o menor dos números naturais. Provaremos a seguir as propriedades básicas da relação de ordem m < n que definimos. A primeira delas é a transitividade.

Teorema 1. (Transitividade.) Se m < n e n < p, então m < p. Demonstração: Se m < n, n < p então n = m + k, p = n + r, logo p = (m + k) + r = m + (k + r), portanto m < p. Outra importante propriedade de relação de ordem é que, dados dois números naturais diferentes m, n, ou se tem m < n ou então n < m. Esta propriedade pode ser reformulada de outra maneira, como segue. Diremos que os números naturais m, n são comparáveis quando se tem m = n, m < n ou n < m. Podemos então enunciar o seguinte teorema. Teorema 2. (Comparabilidade.) Todo número natural n é comparável com qualquer número natural m. Demonstração: Isto se prova por indução. O número 1 é comparável com qualquer outro número natural pois já sabemos que 1 < m para todo m ≠ 1. Suponhamos agora que o número n seja comparável com todos os números naturais. Mostremos, a partir daí, que n + 1 também tem essa propriedade. Com efeito, seja m ∈ N tomado arbitrariamente. Sabemos que se tem m < n, m = n ou n < m. Examinemos cada uma dessas possibilidades: Se for m < n então m < n + 1 por transitividade, pois sabemos que n < n + 1. Se for m = n, então m < n + 1. Se for n < m então m = n + p. Neste caso, há duas possibilidades. Ou se tem p = 1, donde m = n + 1, ou então p > 1, logo p = 1 + p', e daí m = (n + 1) + p' e concluímos que n + 1 < m. Em qualquer hipótese, vemos que n + 1 é comparável com qualquer número natural m. Por indução, fica provada a comparabilidade de quaisquer números naturais m, n. A comparabilidade dos números naturais é complementada pela proposição abaixo.

EUREKA! N° 3, 1998

32


Sociedade Brasileira de Matemática

Teorema 3. (Tricotomia.) Dados m, n ∈ N, qualquer das afirmações m < n, m = n, n < m exclui as outras duas. Demonstração: Se tivéssemos m < n e m = n, então seria m = m + p, donde m + 1 = m + p + 1 e, cortando m, concluiríamos que 1 = p + 1, um absurdo, pois 1 não é sucessor de p. Portanto m < n (e analogamente, n < m) é incompatível com m = n. Do mesmo modo, se tivéssemos m < n e n < m, então teríamos n = m + p e m = n + k, do que resultaria n = n + k + p, logo n + 1 = n + k + p + 1 e, cortando n, concluiríamos que 1 = k + p + 1, um absurdo. O teorema seguinte mostra que n e n + 1 são números consecutivos. Teorema 4. Não existem números naturais entre n e n + 1. Demonstração: Se fosse possível ter n < p < n + 1, teríamos p = n + k e n + 1 = p + r, logo n + 1 = n + k + r. Cortando n, obteríamos 1 = k + r. Por definição, isto significaria k < 1, o que é absurdo, pois já vimos que k ≠ 1 ⇒ k > 1. A conexão entre a relação de ordem e as operações de adição e multiplicação é dada pelo seguinte teorema: Teorema 5. (Monotonicidade.) Se m < n, então m + p < n + p e mp < np. Demonstração: Usando a definição de <, temos que m < n ⇒ n = m + k ⇒ n + p = (m + k) + p ⇒ m + p < n + p. Analogamente, m < n ⇒ n = m + k ⇒ np = mp + kp ⇒ np >mp. A recíproca da monotonicidade é a Lei do Corte para desigualdades: m + p < n + p ⇒ m < n e mp < np ⇒ m < n. O leitor poderá prová-la por absurdo, usando a tricotomia e a própria monotonicidade. 6. BOA ORDENAÇÃO

Dado o subconjunto A ⊂ N, diz-se que o número natural a é o menor (ou primeiro) elemento de a quando a ∈ A e, além disso, a ≤ x, para todos os elementos x ∈ A. Por exemplo, 1 é o menor elemento de N. De agora em diante, dado n ∈ N, indicaremos com In o conjunto dos números naturais p tais que 1 ≤ p ≤ n. Assim, I1 = {1}, I2 = {1, 2}, I3 = {1, 2, 3} etc. As propriedades da relação de ordem m < n, demonstradas na seção anterior para os números naturais (exceto o Teorema 4 que vale apenas para EUREKA! N° 3, 1998

33


Sociedade Brasileira de Matemática

números inteiros), são igualmente válidas para os números inteiros, racionais e, mais geralmente, para números reais quaisquer. Existe, porém, uma propriedade de suma importância que é válida para a ordem entre os números naturais, mas sem equivalente para números inteiros, racionais ou reais.

Teorema 6. (Princípio da Boa Ordenação.) Todo subconjunto não-vazio A ⊂ N possui um menor elemento. Demonstração: Sem perda de generalidade, podemos admitir que 1 ∉ A, pois caso contrário 1 seria evidentemente o menor elemento de A. O menor elemento de A, cuja existência queremos provar, deverá ser da forma n + 1. Devemos pois encontrar um número natural n tal que n +1 ∈ A e, além disso, todos os elementos de A são maiores do que n, logo maiores do que 1, 2, …, n. Noutras palavras, procuramos um número natural n tal que In ⊂ N – A e n + 1 ∈ A. Com esse objetivo, consideramos o conjunto X = {n ∈ N; In ⊂ N – A}. Portanto, X é o conjunto dos números naturais n tais que todos os elementos de A são maiores do que n. Como estamos supondo que 1 ∉ A, sabemos que 1 ∈ X. Por outro lado, como A não é vazio, nem todos os números naturais pertencem a X, ou seja, temos X ≠ N. Pelo axioma D, vemos que o conjunto X não é indutivo, isto é, deve existir algum n ∈ X tal que n + 1 ∉ X Isto significa que todos os elementos de A são maiores do que n mas nem todos são maiores do que n + 1. Como não há números naturais entre n e n + 1, concluímos que n + 1 pertence a A e é o menor elemento de A.

O Princípio da Boa Ordenação pode muitas vezes ser usado em demonstrações, substituindo o Princípio da Indução. Vejamos um exemplo. Dissemos anteriormente que um subconjunto X ⊂ N chama-se indutivo quando n ∈ X ⇒ n + 1 ∈ X, ou seja, quando X contém o sucessor de cada um dos seus elementos. O Princípio da Indução afirma que se um conjunto indutivo X contém o número 1 então X contém todos os números naturais. Vamos usar o Princípio da Boa Ordenação para provar que se um conjunto indutivo X contém o número a, então X contém todos os números naturais maiores do que a. A prova desta afirmação se faz por absurdo, como ocorre em geral quando se usa a boa ordenação. Suponhamos então que existam números EUREKA! N° 3, 1998

34


Sociedade Brasileira de Matemática

naturais, maiores do que a, não pertencentes ao conjunto indutivo X. Seja b o menor desses números. Como b > a, podemos escrever b = c + 1, onde, pela definição de b, tem-se necessariamente c ∈ X. Mas, como X é indutivo, isto obriga que b = c + 1 ∈ X, uma contradição. A proposição qua acabamos de demonstrar pode ser enunciada da seguinte forma:

Teorema 7: (Princípio da Indução Generalizado.) Seja P uma propriedade referente a números naturais, cumprindo as seguintes condições: (1) O número natural a goza da propriedade P; (2) Se um número natural n goza da propriedade P então seu sucessor n + 1 também goza de P. Então todos os números naturais maiores do que ou iguais a a gozam da propriedade P. Exemplo 2. Vejamos uma situação simples onde se emprega o Princípio da Indução Generalizado. Trata-se de provar que 2n + 1 < 2n, para todo n ≥ 3. Esta afirmação, (que é falsa para n = 1 ou n = 2), vale quando n = 3. Supondo-a válida para um certo n ≥ 3, mostremos que daí decorre sua validez para n + 1. Com efeito, 2(n + 1) + 1 = (2n + 1) + 2 < 2n + 2 < 2n + 2n = 2n + 1. (Na primeira desigualdade, usamos a hipótese de indução.) Exemplo 3. Usando a desigualdade 2n + 1 < 2n, qua acabamos de provar para n ≥ 3, podemos demonstrar que n2 < 2n para todo n ≥ 5, empregando novamente o Princípio da Indução Generalizado. Com efeito, vale 52 < 25 pois 25 < 32. Supondo válida a desigualdade n2 < 2n para um certo valor de n ≥ 5, daí segue-se que (n + 1)2 = n2 + 2n + 1 < 2n + 2n + 1 (pela hipótese de indução) < 2n + 2n (pelo exemplo anterior) = 2n + 1. Portanto P(n) ⇒ P(n + 1). Pelo Princípio de Indução Generalizado, segue-se que P(n) vale para todo n ≥ 5. Evidentemente, a desigualdade n2 < 2n é falsa para n = 1, 2, 3, 4. O teorema abaixo contém outra aplicação do Princípio da Boa Ordenação. Teorema 8. Toda função monótona não-crescente f: N → N é constante a partir de um certo ponto. ( Isto é, existe n0 ∈ N tal que f(n) = f(n0), para todo n ≥ n0.) Demonstração: Seja n0 o menor elemento do conjunto X = {f(1), f(2), …, f(n),…}. Então n > n0 ⇒ f(n) ≤ f(n0) (porque a função f é não-crescente) o que acarreta que f(n) = f(n0) (porque f(n0) é o menor elemento de X).

EUREKA! N° 3, 1998

35


Sociedade Brasileira de Matemática

Corolário: Toda seqüência decrescente n1 > n2 > … de números naturais é finita. Com efeito, do contrário, pondo f(k) = nk, obteríamos uma função estritamente decrescente f : N → N. 7. SEGUNDO PRINCÍPIO DA INDUÇÃO

Em algumas situações, ao tentarmos fazer uma demonstração por indução, na passagem de n para n + 1, sentimos necessidade de admitir que a proposição valha não apenas para n e sim para todos os números naturais menores do que ou iguais a n. A justificativa de um raciocínio desse tipo se encontra no Teorema 9: (Segundo Princípio da Indução.) Seja X ⊂ N um conjunto com a seguinte propriedade: Dado n ∈ N, se todos os números naturais menores do que n (I) pertencem a X, então n ∈ X. O segundo Princípio da Indução afirma que um conjunto X ⊂ N com a propriedade (I) coincide com N. Demonstração: Com efeito, supondo, por absurdo, que X ≠ N, isto é, que N – X ≠ ∅, seja n o menor elemento do conjunto N – X, ou seja, o menor número natural que não pertence a X. Isto quer dizer que todos os números naturais menores do que n pertencem a X. Mas então, pela propriedade (I), n pertence a X, uma contradição. Segue-se que N – X = ∅ e X = N.

Obs. : Se um conjunto X ⊂ N goza da propriedade (I), para que um número natural n não pertencesse a X seria necessário que existisse algum número natural r < n tal que r ∉ X. Em particular, se n = 1, como não existe número natural menor do que 1, a hipótese 1 ∉ X não pode ser cumprida. Noutras palavras, (I) já contém implicitamente a afirmação de que 1 ∈ X. Assim, ao utilizar o Segundo Princípio da Indução, não é preciso estipular que X contém o número 1. Toda propriedade P que se refira a números naturais define um subconjunto X ⊂ N, a saber, o conjunto dos números naturais que gozam da propriedade P. (E reciprocamente, todo conjunto X ⊂ N define uma propriedade referente a números naturais, a saber, a propriedade de pertencer a X.) Deste modo, "propriedade" e "conjunto" são noções equivalentes. Por isso, é natural que o Segundo Princípio da Indução possua a formulação seguinte, onde ele aparece como o EUREKA! N° 3, 1998

36


Sociedade Brasileira de Matemática

Teorema 10: (Segundo método de demonstração por indução.) Seja P uma propriedade referente a números naturais. Dado n ∈ N, se a validade de P para todo número natural menor do que n implicar que P é verdadeira para n, então P é verdadeira para todos os números naturais. Demonstração: Com efeito, nas condições do enunciado, o conjunto X dos números naturais que gozam da propriedade P satisfaz a condição (I) do Segundo Princípio da Indução, logo X = N e P vale para todos os números naturais. Aplicaremos agora o Segundo Princípio da Indução para demonstrar um fato geométrico. No exemplo a seguir, usamos os números naturais como instrumento de contagem, isto é, como números cardinais, pois empregamos expressões do tipo um polígono de n lados". (Vide seção 6.) Sabe-se que, traçando diagonais internas que não se cortam, pode-se decompor qualquer polígono em triângulos justapostos. Isto é evidente quando o polígono é convexo: basta fixar um vértice e traçar as diagonais a partir dele. Se o polígono não é convexo, a prova requer mais cuidados. (Vide "Meu Professor de Matemática", pag. 109.) O leitor pode experimentar com um polígono não-convexo e verificar qua há muitas maneiras diferentes de decompô-lo em triângulos justapostos mediante diagonais internas. Mas vale o resultado seguinte, no qual usaremos o Segundo Princípio da Indução. Exemplo 4. Qualquer que seja a maneira de decompor um polígono P, de n lados, em triângulos justapostos por meio de diagonais internas que não se intersectam, o número de diagonais utilizadas é sempre n – 3. Com efeito, dado n, suponhamos que a proposição acima seja verdadeira para todo polígono com menos de n lados. Seja então dada uma decomposição do polígono P, de n lados, em triângulos justapostos, mediante diagonais internas. Fixemos uma dessas diagonais. Ela decompõe P como reunião de dois polígonos justapostos P1, de n1 lados, e P2, de n2 lados, onde n1 < n e n2 < n, logo a proposição vale para os polígonos P1 e P2. Evidentemente, n1 + n2 = n + 2.

EUREKA! N° 3, 1998

37


Sociedade Brasileira de Matemática

P2

P1

As d diagonais que efetuam a decomposição de P se agrupam assim: n1 – 3 delas decompõem P1, n2 – 3 decompõem P2 e uma foi usada para separar P1 de P2. Portanto d = n1 – 3 + n2 – 3 + 1 = n1 + n2 – 5. Como n1 + n2 = n + 2, resulta que d = n – 3. Isto completa a demonstração.

Observações: 1.

Para habituar-se com o método de demonstração por indução é preciso praticá-lo muitas vezes, a fim de perder aquela vaga sensação de desonestidade que o principiante tem quando admite que o fato a ser provado é verdadeiro para n, antes de demonstrá-lo para n + 1.

2.

Pratique também (com moderação) o exercício de descobrir o erro em paradoxos que resultam do uso inadequado do método de indução. Vejamos dois desses sofismas:

Exemplo 5. Todo número natural é pequeno. Ora, 1 certamente é pequeno. E se n é pequeno, n + 1 não vai subitamente tornar-se grande, logo também é pequeno. (O erro aqui consiste em que a noção "número pequeno" não é bem definida.) Exemplo 6. Toda função f : X → Y, cujo domínio é um conjunto finito X, é constante. Isto é obviamente verdadeiro se X tem apenas 1 elemento. Supondo a afirmação verdadeira para todos os conjuntos com n elementos, seja f : X → Y definida num conjunto X com n + 1 elementos. Considere um elemento a ∈ X. Como X' = X – {a} tem n elementos, f assume o mesmo valor c ∈ Y em todos os elementos de X'. Agora troque a por um outro elemento b ∈ X'. Obtém-se X'' = X – {b} um conjunto com n elementos EUREKA! N° 3, 1998

38


Sociedade Brasileira de Matemática

(entre os quais a). Novamente pela hipótese de indução, f é constante e igual a c em X''. Logo f (a) = c e daí f : X → Y é constante. (Aqui o erro reside no uso inadequado da hipótese de indução. O raciocínio empregado supõe implicitamente que X tem pelo menos 3 elementos. Na realidade, não vale a implicação P(1) ⇒P(2).) O perigo de fazer generalizações apressadas relativamente a asserções sobre números naturais fica evidenciado com o seguinte exemplo:

Exemplo 7. Considere o polinômio p(n) = n2 – n + 41 e a afirmação "o valor de p(n) é sempre um primo para n = 0, 1, 2, 3, …". Embora isso seja verdadeiro para n = 0, 1, 2, …, 40, temos p(41) = 412 – 41 + 41 = 412 não é primo, logo a afirmação não é verdadeira. Semelhantemente, a expressão q(n) = n2 – 79n + 1601 fornece primos para n = 1, 2, …, 79, mas q(80) = 802 – 79 ⋅ 80 + 1601 = 1681 não é primo, pois é divisível por 41. A moral da história é: Só aceite que uma afirmação sobre os números naturais é realmente verdadeira para todos os naturais se isso houver de fato sido demonstrado! 8. NÚMEROS CARDINAIS

Vamos agora mostrar como se usam os números naturais para contar os elementos de um conjunto finito. O Princípio da Indução será essencial. Lembremos que, dado n ∈ N, escrevemos In = {p ∈ N; p ≤ n}, portanto In = {1, 2, …, n}. Uma contagem dos elementos de um conjunto não-vazio X é uma bijeção f : In → X. Podemos pôr x1 = f(1), x2 = f(2),…, xn = f(n) e escrever X = {x1, x2,…xn}. Diz-se então que X possui n elementos. O conjunto X chama-se um conjunto finito quando existe n ∈ N tal que X possui n elementos. Um exemplo óbvio de conjunto finito é In. Evidentemente, a função identidade f: In → In é uma contagem dos elementos de In. Um exemplo de conjunto infinito é o proprio conjunto N dos números naturais, pois nenhuma função f : In → N pode ser sobrejetiva, não importa qual n se tome. De fato, dada f, tomamos k = f(1) + f(2) +…+ f(n) e vemos que k > f(x) para todo x ∈ In, logo k ∉ f(In), e f não é sobrejetiva. A fim de que não haja ambigüidade quando se falar do número de elementos de um conjunto finito X, é necessário provar que todas as EUREKA! N° 3, 1998

39


Sociedade Brasileira de Matemática

contagens de X fornecem o mesmo resultado. Noutras palavras, dado o conjunto X, os números naturais m, n e as bijeções f : Im → X, g : In → X, devemos mostrar que se tem m = n. Começamos observando que se f e g são bijeções, então φ = g–1 ο f : Im → In também é uma bijeção. Basta portanto provar o seguinte:

Teorema 11. Dados m, n ∈ N, se φ : Im → In é uma bijeção, então m = n. Demonstração. Com efeito, chamemos de X o conjunto dos números naturais n que têm a seguinte propriedade: só existe uma bijeção φ : Im → In quando m = n. Evidentemente, 1 ∈ X. Suponhamos agora que n ∈ X. Dada uma bijeção φ: Im+1 → In+1, duas coisas podem acontecer. Primeira: φ(m + 1) = n + 1. Neste caso, a restrição φ|Im : Im → In é uma bijeção, logo m = n, donde m + 1 = n + 1. Segunda: φ(m + 1) = b, com b < n + 1. Neste caso, consideramos a = φ –1(n + 1) e definimos uma nova bijeção ψ : Im + 1 → In + 1, pondo ψ (m + 1) = n + 1, ψ(a) = b e ψ(x) = φ(x) para os demais elementos x ∈ Im + 1. Então recaímos no caso anterior e novamente concluímos que m + 1 = n + 1. Isto mostra que n ∈ X ⇒ n + 1 ∈ X, logo X = N e a unicidade do número cardinal de um conjunto finito fica demonstrada. Agora os números naturais não são apenas elementos do conjuntopadrão N, mas servem também para responder perguntas do tipo "quantos elementos tem o conjunto X?,"ou seja, podem ser usados também como números cardinais. A adição de números naturais se relaciona com a cardinalidade dos conjuntos por meio da seguinte proposição.

Teorema 12: Sejam X, Y conjuntos finitos disjuntos. Se X tem m elementos e Y tem n elementos, então X ∪Y tem m + n elementos. Demonstração: Com efeito, se f : Im → X e g : In → Y são bijeções, definimos uma bijeção h : Im+n → X ∪Y por h (x) = f (x) se 1 ≤ x ≤ m e h(x) = g(x) + m se m + 1 ≤ x ≤ m + n, o que conclui a demonstração. Prova-se, por indução, que todo subconjunto de um conjunto finito X é também finito e seu número de elementos é menor do que ou igual ao de X (Veja E.L.Lima, "Análise Real", vol 1, pag. 5.) E conveniente incluir, por definição, o conjunto vazio entre os conjuntos finitos e dizer que o seu número de elementos é zero. Embora zero EUREKA! N° 3, 1998

40


Sociedade Brasileira de Matemática

não seja um número natural, ele passa a ser o número cardinal do conjunto vazio. Seguem-se algumas proposições que devem ser demonstradas por indução ou boa ordenação. Os dez últimos exercícios foram sugeridos pelo Professor A. C. Morgado.

Exercícios: 1. Construa um esquema de setas começando com os números ímpares, seguidos dos números pares divisíveis por 4 em ordem decrescente e, por fim, os pares não divisíveis por 4 em ordem crescente. Noutras palavras, tome X = N e defina s : X → X pondo s(n) = n + 2 se n não é divisível por 4, s(n) = n – 2 se n for múltiplo de 4. Mostre que s : X → X cumpre os axiomas A, B, C mas não D. 2. Defina, por recorrência, uma função f : N → N estipulando que f (1) = 3 e f (n + 1) = 5. f (n) + 1. Dê uma formula explícita para f (n). 3. Dê uma fórmula explícita para f : N → N sabendo que f(1) = 1, f(2) = 5 e f (n + 2) = 3f (n + 1) – 2f (n). 4. Seja X ⊂ N um conjunto indutivo não-vazio. Mostre que existe a ∈ N tal que X = {n ∈ N; n ≥ a}. n(n + 1)(2n + 1) 5. Prove, por indução, que 12 + 2 2 + ... + n 2 = . 6 6. Num polígono com n ≥ 6 lados, o número de diagonais é maior do que n. 7. Prove, por indução que [(n + 1)/n]n < n, para todo n ≥ 3. (Sugestão: Observe que (n + 2)/(n + 1) < ( n + 1)/n e eleve ambos os membros desta desigualdade à potência n + 1.) Conclua daí que a seqüência 1, 2 , 3 3 , 4 4 , 5 5 ,... é decrescente a partir do terceiro termo. 8. Prove, por indução a desigualdade de Bernoulli: (1 + a)n > 1 + na quando 1 + a > 0. n

 (n + 1) 2  9. Para todo n ∈ N, ponha x n =   e prove, por indução que se  n ( n + 2)  n+2 tem x n < . Conclua, a partir daí, que a seqüência de termo geral n +1 n  n + 1   é crescente.  n 

EUREKA! N° 3, 1998

41


Sociedade Brasileira de Matemática 3

n n + 2 Sugestão: observe que x n +1 =  ⋅ xn .  ⋅ n + n +3 1   10. Use a distributividade de duas maneiras diferentes para calcular (m + n )(1 + 1) e aplique em seguida a Lei do Corte para obter uma nova prova de que m + n = n + m. 11. Um conjunto S ⊂ N, não-vazio, é limitado superiormente, se existe um natural k tal que para todo natural x ∈ S, então x ≤ k. Mostre que S possui um maior elemento. (Isto é, existe m ∈ S tal que x ≤ m, para todo x ∈ S.) 12. Demonstre que a soma dos n primeiros números ímpares é n2, ou seja, que 1 + 3 + 5 +…+ (2n – 1) = n2. 13. Prove que 2n – 1 é múltiplo de 3, para todo número natural n par. 14. Demonstre que, para todo número natural n, vale 1  1   1  1  1 + 1 + 1 + ...1 +  ≤ n + 1. 1 2 3 n      15. Demonstre que 1 −

1 1 1 1 1 1 1 1 + − + .. + − = + + ... + . 200 199 200 101 102 2 3 4

1 2   16. Determine An se A =   2 4 17. Demonstre, usando o Princípio da Indução Finita, que  p   p + 1  p + n   p + n + 1   +   + ... +   =  .  p  p   p   p  Este resultado é comumente conhecido por Teorema das Colunas. (Por quê?). p 1 3 7 18. Considere a seqüência , , ,..., n ,..., onde 1 2 5 qn p n +1 = p n + 2q n e q n +1 = p n + q n . Demonstre que a) m.d.c (pn, qn) = 1;

EUREKA! N° 3, 1998

42


Sociedade Brasileira de Matemática

b)

pn é o inteiro mais próximo de

de

(1 + 2 ) n e qn é o inteiro mais próximo 2

2 (1 + 2 ) n . 4

19. [A Torre de Hanói.] São dados três suportes A, B e C. No suporte A estão encaixados n discos cujos diâmetros, de baixo para cima, estão em ordem estritamente decrescente. Mostre que é possível, com 2n – 1 movimentos, transferir todos os discos para o suporte B, usando o suporte C como auxiliar, de modo que jamais, durante a operação, um disco maior fique sobre um disco menor. 20. Demonstre que 2n < n!, para n ≥ 4. 21. Demonstre que 2n3 > 3n2 + 3n + 1 para n ≥ 3. 22. Considere n retas em um plano. Mostre que o "mapa" determinado por elas pode ser colorido com apenas duas cores sem que duas regiões vizinhas tenham a mesma cor.

EUREKA! N° 3, 1998

43


Sociedade Brasileira de Matemática

FRAÇÕES CONTÍNUAS, REPRESENTAÇÕES DE NÚMEROS E APROXIMAÇÕES Carlos Gustavo Moreira ♦ Nível Avançado. INTRODUÇÃO

A teoria de frações contínuas é um dos mais belos temas da matemática elementar, sendo ainda hoje assunto de pesquisa recente (incluindo a do autor destas linhas). O objetivo deste artigo é servir como referência didática em português a nível secundário sobre o assunto. Nas inclusões N ⊂ Z ⊂ Q ⊂ R a passagem de Q para R é sem dúvida a mais complicada conceitualmente, e a representação de um número real está diretamente ligada à propria noção de número real. De fato, o conceito de número natural é quase um conceito primitivo no ensino secundário. Já um número inteiro é um número natural com um sinal que pode ser + ou –, e um número racional é a razão entre um número inteiro e um natural não nulo. Por outro lado, dizer o que é um número real é tarefa bem mais complicada, mas há coisas que podemos dizer sobre eles. Uma propriedade essencial de R é que todo número real pode ser bem aproximado por números racionais. Efetivamente, dado x ∈ R, existe k ∈ Z (k = [x]) tal que 0 ≤ x – k < 1. Podemos escrever a representação decimal de x – k = 0, a1a2…an…, ai ∈ {0, 1, …, 9}, o que significa que se rn = an + 10.an–1 + 100.an–2 +…+ 10n–1 . a1,

rn 10

n

≤x−k<

rn + 1 10

n

, e portanto k +

rn 10 n rn

então

é uma boa aproximação racional de

1 , que é um 10 10 n número bem pequeno se n for grande. A representação decimal de um número real fornece pois uma seqüência de aproximações por racionais cujos denominadores são potências de 10. x, no sentido que o erro x − (k +

n

)

EUREKA! N° 3, 1998

44

é menor que


Sociedade Brasileira de Matemática

Dado qualquer x ∈ R e q natural não nulo existe p ∈ Z tal que , e portanto x −

p p +1 ≤x< q q

p p +1 1 1 < e x− ≤ . Em particular há aproximações q q q q

de x por racionais com denominador q com erro menor que

1

. A q representação decimal de x equivale a dar essas aproximações para os denominadores q que são potências de 10, e tem méritos como sua praticidade para efetuar cálculos que a fazem a mais popular das representações dos números reais. Por outro lado, envolve a escolha arbitrária da base 10, e oculta freqüentemente aproximações racionais de x muito mais eficientes do que as que exibe. Por exemplo, 22 1 314 355 1 3141592 π− < <π− < <π− e π− 7 700 100 113 3000000 1000000 22 355 mostram que e são melhores aproximações de π que aproximações 7 113 decimais com denominadores muito maiores, e de fato são aproximações muito mais espectaculares do que se podia esperar. O objetivo deste artigo é apresentar uma outra maneira de representar números reais, que sempre fornece aproximações racionais surpreendentemente boas, e de fato fornece todas essas aproximações excepcionalmente boas, além de ser natural e conceitualmente simples: a representação por frações contínuas. Dado x ∈ R definimos [x] como o único inteiro tal que [x] ≤ x < [x] + 1). Definimos recursivamente 1 , para todo n ∈ N . α 0 = x, a n = [α n ], e, seα n ∉ Z , α n +1 = α n − an Se, para algum n, αn = an temos

x = α 0 = a0 +

1 a1 +

=: [ a ; a1 , a 2 ..., a n ].

1

0

a 2 + ... +

1 an

Se não denotamos EUREKA! N° 3, 1998

45


Sociedade Brasileira de Matemática

x = a0 +

1 1 a1 + a 2 + ...

=: [ a 0 ; a1 , a 2 ...].

O sentido dessa última notação ficará claro mais tarde. A representação acima se chama a representação por frações contínuas de x.

Curiosidade: O denominador da n-ésima aproximação em base B de um número real é Bn. Já o denominador qn da n-ésima aproximação por fração contínua de x depende de x. Apesar disso, para quase todo real x, n

q n converge a e π

n

x−

pn qn

2

/ 12 ln 2

= 3,27582291872... (meu número real preferido!) e

converge a e −π

2

/ 6 ln 2

= 0,093187822954...

Observação: Os αn (como funções de x) são funções distintas do tipo ax + b com a, b, c, d inteiros. Se a fração contínua de x é periódica, ou seja, cx + d se αn + k = αn, n ∈ N, k ∈ N*, então x será raiz de uma equação do segundo grau com coeficientes inteiros, ou seja, será um irracional da forma r + s , r , s ∈ Q. A recíproca é verdadeira (de fato já foi enunciada no artigo de José Paulo Carneiro na RPM, ver referências), mas sua prova é mais difícil, e será apresentada no Apêndice. Se x ∈ Q, sua representação será finita, e seus coeficientes an vêm do algoritmo de Euclides: x=

p q

, q>0

p = a 0 q + r0

0 ≤ r0 < q

q = a1 r0 + r1

0 ≤ r1 < r0

r0 = a 2 r1 + r2

0 ≤ r2 < r1

#

#

rn − 2 = a n rn −1

EUREKA! N° 3, 1998

46


Sociedade Brasileira de Matemática

Isso já é uma vantagem da representação por frações contínuas (além de não depender de escolhas artificiais de base), pois o reconhecimento de racionais é mais simples que na representação decimal.

Seção 1: Reduzidas e boas aproximações. Seja x = [a0; a1, a2, …]. Sejam pn ∈ Z, qn ∈ N* primos entre si tais que pn = [a0; a1, a2, …, an], n ≥ 0. O seguinte resultado será fundamental no qn que seguirá.

Proposição: (pn) e (qn) satisfazem a recorrência pn+2 = an+2 pn+1+ pn e qn+2 = an+2 qn+1 +qn, para todo n ≥ 0. Temos ainda p0 = a0, p1 = a0a1 +1, q0 = 1,q1 = a1. Além disso, pn+1 qn – pnqn+1 = (–1)n, ∀n ≥ 0. Prova: Por indução em n, provaremos que se tk > 0, para k > 1 então [t0; t1, t2, …, tk] =

xk onde as seqüências (xm) e (ym) são definidas por yk

x0 = t0, y0 = 1, x1 = t0t1 + 1, y1 = t0, xn+2 = tn+2 xn+1 + xn, yn+2 = tn+2 yn+1 + yn, ∀n. Suponha que a afirmação seja válida para k = n. Para k = n+1 temos

[t0; t1, t2, …, tn, tn+1] = [t0; t1, t2, …, tn +

(t n +

1 t n +1

]=

1

) x n −1 + x n −2 t n +1 t (t x + x ) + x n −1 t n +1 x n + x n −1 . = n +1 n n −1 n −2 = 1 t n +1 (t n y n −1 + y n − 2 ) + y n −1 t n +1 y n + y n −1 (t n + ) y n −1 + y n −2 t n +1 Por outro lado as igualdades • p1q0 – p0q1 = (a0a1 +1) – a0a1 = 1 • pn+2 qn+1 – pn+1 qn+2 = (an+2 pn+1 + pn) qn+1 – (an+2 qn+1 + qn) pn+1 = = – ( pn+1 qn – pnqn+1) EUREKA! N° 3, 1998

47


Sociedade Brasileira de Matemática

mostram que pn+1 qn – pnqn+1 = (–1)n, ∀n ∈ N, o que implica em particular que os pn, qn dados pelas recorrências acima são primos entre si.

Corolário: x =

α n p n −1 + p n − 2 α n q n −1 + q n − 2

αn =

e

p n−2 − q

n−2

α

q n −1α − p n −1

, ∀n ∈ N.

Prova: A primeira igualdade é conseqüência direta da prova, e a segunda é conseqüência direta da primeira pois x = [a0; a1, a2, …, an–1, αn]. Note que as reduzidas de ordem par são menores e as de ordem ímpar maiores que x = [a0; a1,…].

Teorema 1: x −

pn qn

1 1 < 2 , ∀n ∈ N. q n q n +1 q n

Além disso, x −

pn qn

<

p 1 ou x − n +1 2 q n +1 2q n

<

1 , ∀n ∈ N. 2q n2+1

Prova: x sempre pertence ao segmento de extremos

p n +1 pn e cujo qn q n +1

comprimento é:

p n +1 p n − q n +1 q n

=

( −1) n q n q n +1

=

1 q n q n +1

⇒ x−

pn 1 1 ≤ < 2. qn q n q n +1 q n

Além disso, se

x− ≥

pn p p p 1 1 1 ≥ 2 e x − n+1 ≥ 2 então = x − n + x − n+1 qn qn+1 qn qn+1 2qn 2qn+1 qnqn+1

1 2q n2

+

1 2q n2 +1

⇒ q n +1 = q n , absurdo ❑

Observação: De fato

x−

pn qn

<

1 q n q n +1

EUREKA! N° 3, 1998

48

<

1 a n +1 q n2

. Quanto

maior for


Sociedade Brasileira de Matemática

pn de x. O próximo resultado nos dá qn

an+1 melhor será a aproximação

explicitamente o erro da aproximação de x por

pn qn

.

Proposição: x−

q pn ( −1) n , onde β n +1 = n −1 = [0; a n , a n −1 , a n − 2 ,..., a1 ]. = 2 qn q n (α n +1 + β n +1 )q n

Demonstração: Temos α n +1 =

α n +1 + β n +1 =

p n −1 − q n −1 x . Portanto, qn x − pn

p pn −1 − qn −1x qn −1 pn −1qn − pn qn −1 (−1)n + = = ⇒ x− n = qn x − pn qn qn (qn x − pn ) qn (qn x − pn ) qn

q n (q n x − p n ) q n2

=

(−1) n (α n +1 + β n +1 )q n2

Como aplicação podemos provar o seguinte.

Teorema (Hurwitz, Markov): Para todo α irracional, n ≥ 1 p  pn−1 pn pn+1 p 1 ∈ , , para pelo menos um racional α− < q  qn−1 qn qn+1 q 5q 2 particular α −

temos  . Em 

p 1 tem infinitas soluções racionais p/q. < q 5q 2

Demonstração: Suponha que o teorema seja falso. Então existe α irracional, n ≥ 1 com α n + β n ≤ 5,α n +1 + β n +1 ≤ 5 e α n + 2 + β n + 2 ≤ 5. Devemos portanto ter an = an+1 = an+2 = 1 (todos são claramente no máximo 2, e se algum ak é igual a 2 com k ∈ {n, n + 1, n + 2}, teríamos 1 α k + β k ≥ 2 + > 5 , absurdo.) 3 Seja x = 1/αn+2 e y = βn+1. As desigualdades acima se traduzem em 1 1 1 1 + ≤ 5, 1 + x + y ≤ 5 e + ≤ 5 . Temos x 1+ y 1+ x y

EUREKA! N° 3, 1998

49


Sociedade Brasileira de Matemática

1+ x + y ≤ 5 ⇒ 1+ x ≤ 5 − y ⇒ y( 5 − y) ≥ 1 ⇒ y ≥

portanto

x ≤ 5 −1− y ⇒

1 1 1 1 5 + ≥ + = 1+ x y 5 − y y y( 5 − y), 5 −1 . 2

outro

lado

5 −1 , 2

e

portanto

e

devemos

ter

q 5 −1 , o que é absurdo pois y = β n +1 = n −1 ∈ Q. qn 2

Obs: em particular provamos que α − p < q

p , para todo α irracional. q propriedade, De fato, se racionais

ε > 0, α =

1 5q

2

tem infinitas soluções

5 é o maior número com essa

p 1+ 5 1 e α− < , temos 2 q ( 5 + ε )q 2 1−

  1+ 5  1+ 5  1  − p q 1 − 5  − p < − p < ⇒ q  q      2 2 2 ( 5 + ε )q      

ou seja , p 2 − pq − q 2 < 1 + 5 − p − 5 2

é

temos

1 1 1 1 5 + ≥ + = x 1+ 4 5 − 1 − y 1 + 4 (1 + y )( 5 − 1 − y )

portanto (1 + 4)( 5 − 1 − y ) ≥ 1 ⇒ y ≤

y=

Por

e

pequeno,

e

1+ 5 p − − 5 2 q

1+

5 2

q

p q

é

muito

5 2

5 +ε

p q

,

( 5 + ε ). Se q é grande, 1/q

próximo

( 5 + ε ) é muito próximo de

5 5 +ε

de

0,

2

donde

< 1, absurdo, pois

p 2 − pq − q 2 ≥ 1 (de fato p2 – pq – q2 é um inteiro não nulo, pois se EUREKA! N° 3, 1998

50


Sociedade Brasileira de Matemática 2

 p  p p 1 + 5 1 − 5  p – pq – q = 0 teríamos   −   − 1 = 0 ⇒ ∈  , , q  2 2  q q p absurdo, pois ∈ Q .) q 2

2

Outra maneira de ver que, para todo ε > 0,

1+

5 2

p 1 < q ( 5 + ε )q 2

tem

p ∈ Q é observar que as melhores q p 1+ 5 aproximações racionais de são as reduzidas n de sua fração 2 q

apenas um número finito de soluções

n

contínua [1, 1, 1, 1, …] (ver seção 2 e exemplos), para as quais temos p 1+ 5 1 − n = , com α n + 1 + β n + 1 se aproximando cada q 2 (α n + 1 + β n +1 ) q n2 n

vez mais de

[1;1,1,1...] + [0;1,1,1,...] =

1+ 5 5 −1 + = 5. 2 2

Exemplos: •

• •

π = [3; 7, 15, 1, 292, 1, 1, 1, 2, 1, 3, 1, 14, 2, 1,…], portanto p0 p 22 p 2 333 p 3 355 = 3, 1 = , = , = ,... q0 q1 7 q 2 106 q 3 113 e = [2; 1, 2, 1, 1, 4, 1, 1, 6, 1, 1, 8,…, 1, 1, 2n, …], (isso não é fácil de provar.) 2 = [1;2,2,2,...] pois 2 =1+

1 2 +1

1

=1+ 2+

1 2 +1

1

=1+

2+

EUREKA! N° 3, 1998

51

= ...

1

2+

1 2 +1


Sociedade Brasileira de Matemática

1+

5 2

= [1;1,1,1,...] pois 1 +

5 2

=1+

1 1+

5 2

1

=1+ 1+

= ...

1 1+

5 2

Isso prova em particular que

2 e 1 + 5 são irracionais, pois sua fração 2

contínua é infinita.

Seção 2: Boas aproximações são reduzidas. O próximo teorema (e seu Corolário 2) caracteriza as reduzidas em termo do erro reduzido da aproximação de x por p/q, o qual é, por definição, a razão entre x − p / q e o erro máximo da aproximação por falta com denominador q, que é 1/q. Assim, o erro reduzido da aproximação de x por p/q é qx − p .

Teorema 2: q n x − p n < qx − p , ∀p, q ∈ Z,0 < q ≤ q n ,

p pn ≠ . q qn

Além disso, q n x − p n ≤ qx − p , ∀p, q ∈ Z,0 < q < q n +1 .

Prova:

p pn 1 1 se q < qn+1, e assim − ≥ > q qn qq n q n q n +1

p está fora do q

p p  intervalo  n , n +1  . Portanto,  q n q n +1 

x−

 p p p p p ≥ min  − n , − n +1 q  q q n q q n +1

 1 1 ⇒ qx − p ≥ ≥ qn x − pn . ≥ q n +1  qq n + 1

p n +1 , donde an+1 ≥ 2, e qn+1 > 2qn, q n +1 pois numa fração contínua finita, como no algaritmo de Euclides, o último coeficiente an é sempre maior que 1. Nesse caso, se q ≤ qn , teremos Além disso, se vale a igualdade, então x =

EUREKA! N° 3, 1998

52


Sociedade Brasileira de Matemática

x−

1 1 1 1 p p p q −q p ≥ x − n − n+1 − n ≥ − = n+1 > ⇒ qx − p > ≥ qn x − pn . qn qn+1 qn qqn qnqn+1 qqnqn+1 qqn+1 qn+1 q

Corolário 1: x −

pn p < x − , ∀q < q n . qn q

Corolário 2: Se qx − p < q ' x − p ' , ∀ q ' ≤ q , p ≠ q ' então p/q é uma reduzida q

q'

da fração contínua de x.

Prova: Tome n tal que q n ≤ q < q n +1 . Teremos q n x − p n ≤ qx − p , e portanto p/q = p n /q n ❑

p Teorema 3: Se x − p < 1 então é uma reduzida da fração contínua de x. 2 q

2q

q

Prova: Seja n tal que qn < q ≤ qn+1. Suponha que

p p n +1 ≠ . Então, temos q q n +1

duas possibilidades: q p 1 1 a) ≥ . q ≥ n +1 ⇒ x − ≥ 2 q qq n +1 2q 2 b) q< =

1 1 qn +1 p p p p p ⇒ qn +1 > 2qn ⇒ x − ≥ n − − nm − n ≥ − = 2 q qn q qnm qn qqn qn qn +1

1 1 qn +1 − q > > 2. qqn qn +1 2qqn 2q

Apêndice: Frações contínuas periódicas Nesta seção provaremos que os números reais com fração contínua periódica são exatamente as raízes de equações do segundo grau com coeficientes inteiros. Lembramos que na representação de x por fração contínua, an, αn são definidos por recursão por EUREKA! N° 3, 1998

53


Sociedade Brasileira de Matemática

α 0 = x, a n = [α n ], α n +1 =

1 . α n − an

e temos p − q n−2 x α n = n−2 , ∀n ∈ N . q n −1 x − p n −1 Isso dá uma prova explícita do fato de que se a fração contínua de x é periódica, então x é raiz de uma equação do segundo grau com coeficientes inteiros. De fato, se αn + k = αn , n ∈ N, k ∈ N* então p n − 2 − q n − 2 x p n + k − 2 − q n + k −2 x = ⇒ qn−1 x − pn−1 qn+k −1 x − pn+k −1

(qn−1qn+k −2 − qn−2 qn+k −1 ) x 2 + ( pn+k −1qn−2 + pn−2 qn+k −1 − pn+k −2 qn−1 − pn−1qn+k −2 ) x + pn−1 pn+k −2 − pn−2 pn+k −1 = 0. Note que o coeficiente de x2 é não-nulo, pois

q n −1 é uma fração irredutível q n−2

(de fato p n −1 q n − 2 − p n − 2 q n −1 = (−1) n ) de denominador qn–2 e

q n + k −1 é uma q n+k −2

fração irredutível de denominador qn+k–2 > qn–2 , donde q qn −1 ≠ n + k −1 ⇒ q n −1 q n + k − 2 − q n − 2 q n + k −1 ≠ 0. qn − 2 q n + k − 2 Vamos provar agora um resultado devido a Lagrange segundo o qual se x é uma irracionalidade quadrática, isto é, se x é um irracional do tipo r + s, r , s ∈ Q, s > 0 então a fração contínua de x é periódica, i. e, existem n ∈ N, k ∈ N* com αn + k = αn . Neste caso, existem a, b, c inteiros tais que

ax 2 + bx + c = 0 , com b 2 − 4ac > 0 e b 2 − 4ac irracional. Como vimos na seção 1, p α + p n−2 x = n −1 n , e portanto q n −1α n + q n − 2  p α + p n−2 ax 2 + bx + c = 0 ⇒ a  n −1 n  q n −1α n + q n − 2

2

 p α + p n−2   + b  n −1 n  q α +q n−2   n −1 n

⇒ A n α n2 + B n α n + C n = 0,

onde EUREKA! N° 3, 1998

54

 +c= 0  


Sociedade Brasileira de Matemática

An = ap n2−1 + bp n −1 q n −1 + cq n2−1 Bn = 2ap n −1 p n − 2 + b( p n −1 q n − 2 + p n − 2 q n −1 ) + 2cq n −1 q n − 2 C n = ap n2− 2 + bp n − 2 q n − 2 + cq n2− 2. Note que Cn = An–1. Vamos provar que existe M > 0 tal que 0 < A n ≤ M para todo n ∈ N, e portanto 0 < C n ≤ M , ∀n ∈ N :

 p  An = ap n2−1 + bp n −1 q n −1 + cq n2−1 = aq n2−1  x − n −1   q n −1  

 p   x − n −1 , q n −1  

onde x e x são as raízes de a, X2 + bX + c = 0, mas

x−

(

 pn−1 p p p 1 < 2 ≤ 1 ⇒ An = aqn2−1 x − n−1 x − n−1 ≤ a x − x + x − n−1  qn−1 qn−1 qn−1 qn−1 qn−1 

)

 ≤  

≤ a x − x + 1 =: M . Notemos agora que Bn2 − 4 An C n = b 2 − 4ac, ∀n ∈ N. De fato,

Bn2 − 4 An C n = ( p n −1 q n − 2 − p n − 2 q n −1 ) 2 (b 2 − 4ac) = b 2 − 4ac. Portanto, Bn2 ≤ 4 AnCn + b2 − 4ac = 4M 2 + b2 − 4ac ⇒ Bn ≤ M ' = 4M 2 + b2 − 4ac,∀n ∈ N. Provamos assim que An, Bn e Cn estão uniformemente limitados, donde há apenas um número finito de possíveis equações An X2 + BnX + Cn = 0, e portanto de possíveis valores de αn. Assim, necessariamente αn+k = αn para alguma escolha de n ∈ N, k ∈ N*.

Referências: • N. Beskin - Frações contínuas - Iniciação à Matemática - Editora Mir. • José Paulo Q. Carneiro - Um processo finito para a raiz quadrada – Revista do Professor de Matemática 34, 1997, pp. 36-44. • C.D. Olds - Continued Fractions - New Mathematical Library - Random House. • A. M. Rockett, P. Szüsz - Continued Fractions - World Scientific.

EUREKA! N° 3, 1998

55


Sociedade Brasileira de Matemática

SOLUÇÕES DE PROBLEMAS PROPOSTOS EUREKA! Nos. 1 e 2 Publicamos aqui algumas das respostas enviadas por nossos leitores.

2)

Em uma pista circular há postos de gasolina, e o total de gasolina que há nos postos é exatamente o suficiente para um carro dar uma volta. Prove que existe um posto de onde um carro com o tanque inicialmente vazio pode partir e conseguir dar uma volta completa na pista (parando para reabastecer nos postos).

Solução Sejam P1, P2,…,Pn os postos de gasolina, li a quantidade de gasolina no posto Pi e ci a quantidade de gasolina necessária para ir de Pi a Pi+1, para i = 1, 2,…, n (convenção: para 1 ≤ k ≤ n, Pn+k : = Pk ). Por hipótese, n

n

∑ l =∑ c . i

Suponha que exista k com 1 ≤ k ≤ n e

i

i =1

i =1

k

k

∑ l <∑ c i

i =1

i

(se não

i =1

existe tal k podemos dar a volta começando em P1). Tome k0 com 1≤ k0 ≤ n k0

tal que

∑ (l

i

− c i ) seja o menor possível. Afirmamos que podemos dar a

i =1

volta começando em Pk0 +1 . De fato, se não for assim, existe r com 1≤ r ≤ n e k0 + r

(l i − ci ) < 0, mas então teríamos

k0 + r

(l i − ci ) <

i =1

i = k 0 +1 k0 + r

absurdo (se k0+r > n temos

(l i − ci ) =

∑ (l

i

− ci ), o que é um

i =1

ko + r − n

i =1

3)

k0

∑ i =1

(l i − ci ), pois

n

∑ (l

i

− ci ) = 0).

i =1

Prove que existe n ∈ N tal que os 1000 primeiros dígitos de n1998 são iguais a 1.

Solução Seja n ∈ N tal que

EUREKA! N° 3, 1998

56


Sociedade Brasileira de Matemática

111 ..

11 α 0α 1α 2 ...α p ; onde α i é tal que 0 ≤ α i ≤ 9;

n1998 =

1000 algarismos

i = 1, 2, 3, …, p. Seja também k = 111…11, daí: s 1998 < (k + 1) . 10s, k .10s ≤ n1998 ≤ k 9999 ...

9, logo k . 10 ≤ n s algarismos

Precisamos garantir que há algum n ∈ N que satisfaça a desigualdade acima; seja então s = 1998 . p : k . 101998 . p ≤ n1998 < (k + 1) . 101998 . p ⇒

n < 1998 k + 1. 10 p observe que se tomarmos n = 10 p .1998 k + 1; onde z  = maior inteiro menor ou igual a z, e p suficientemente grande satisfaremos a condição do enunciado. 1998

k .10 p ≤ n < 1998 k + 1.10 p ⇒

1998

k≤

Conclusão: ∃ n ∈ N tal que n1998 é escrito como no enunciado. 5)

Sejam a > 0 e P1P2P3P4P5 uma poligonal aberta contida em um dos semiplanos determinados pela reta P1 P5 . Prove que existem pontos P6 e P7 no plano, com P5 P6 = a, de modo que é possível ladrilhar o plano com infinitos ladrilhos congruentes ao heptágono P1P2P3P4P5P6P7.

Solução Traçe a paralela a P3P2 passando por P1. O ponto P7 pertencerá a essa reta e

teremos P1 P7 = P3 P2 . O ponto P6 pertencerá à paralela a P3P4 passando por

P5 e satisfará P5 P6 = a, ou seja, P5 P6 =

→ a ⋅ P3 P4 . P3 P4

Rodando o heptágono H = P1P2P3P4P5P6P7 de 180° em torno do ponto médio de P1P2 obtemos o heptágono H' = P1'P2'P3'P4'P5'P6'P7' com P1' = P2 , P2' = P1 , P3' = P7 , P7' = P3. Transladando infinitas vezes os

heptágonos H e H' por k . P3 P6 , k ∈ Z, cobrimos uma faixa dentada, que,

transladada infinitas vezes por m . P4 ' P5 , m ∈ Z, nos permite cobrir o plano. EUREKA! N° 3, 1998

57


Sociedade Brasileira de Matemática

Desta

forma,

cobrimos

o

plano

com

os

heptágonos

H + K . P3 P6 + m . P4 ' P5 e H' + k . P3 P6 + m . P4 ' P5 , k ∈ Z, de interiores disjuntos e todos congruentes a H. 6)

Mostre que toda seqüência com n2+1 elementos possui uma subseqüência crescente com n+1 elementos ou uma subseqüência decrescente com n+1 elementos.

Solução Dada uma seqüência a1, a2,…, a n 2 +1 de números reais, definimos para 1 ≤ i ≤

n2+1 o número f (i) como sendo o número máximo de termos de uma subseqüência decrescente de a1, a2,…, a n 2 +1 começando em ai. Suponha que não exista nenhuma subseqüência decrescente de n +1 elementos. Então f (i) ≤ n para todo i, e portanto f (i) só pode assumir os n valores 1, 2, …, n. Assim, existem 1 ≤ i1 < i2 <…< in+1 com f (i1) = f (i2) = … = f (in+1), mas nesse caso devemos ter a i1 ≤ a i2 ≤ ... ≤ a n 2 +1 , com n + 1 termos.

Obs. 1: Mostra-se com um argumento análogo que toda seqüência com mn+1 elementos possui uma subseqüência crescente de m+1 elementos ou uma subseqüência decrecente de n+1 elementos (de fato que existe uma seqüência crescente de m+1 elementos ou uma seqüência estritamente decrescente de n+1 elementos.) Obs. 2: O resultado (e sua generalização na obs. 1) é o melhor possível. De fato, dados m, n ∈ N, a seqüência de mn termos n, n–1, n–2, …, 1, 2n, 2n–1, 2n–2, …, n+1, 3n, …, 2n+1, …, mn, mn–1, …, (m–1) n+1 não contém nenhuma seqüência crescente de mais de m elementos nem nenhuma seqüência decrescente de mais de n elementos.

12)

a) Prove que se n ∈ N e 2n + 1 é um número primo então n é uma potência de 2. b) Prove que se a, n ∈ N, n ≥ 2 e an –1 é primo, então a = 2 e n é primo.

EUREKA! N° 3, 1998

58


Sociedade Brasileira de Matemática

Solução a) Sabemos que ∀n ∈ N pode ser escrito da seguinte forma: n = 2 k ⋅ p onde k ∈ N e p é ímpar.

( )

k

k

p

k

Seja n = 2n + 1, logo n = 2 2 ⋅ p + 1 = 2 2 + 1, fazendo λ = 2 2 ⇒ x = λ p + 1. Se p é im ímpar maior do que 1, teremos: x = λ p + 1 = (λ + 1)(λ p −1 − λ p − 2 + λ p −3 − ... + 1) e, como x é primo, ele não poderá ser fatorável em um produto de fatores diferentes de 1. Basta então observar que o segundo fator da multiplicação acima não é igual a 1 com p ímpar maior do que 1, mas isso segue de λ p > λ ⇒ λ p + 1 ≠ λ + 1. Logo k

devemos ter necessariamente x = 2 2 + 1, ou seja n = 2k.

b) Seja y = an – 1 = (a – 1)(an–1 + an–2 +…+ a +1) primo: i) ii)

Vamos verificar inicialmente que a deve ser igual a 2. De fato a – 1 = 1, já que o segundo fator não pode ser igual a 1 (a ≥ 1). Suponha que n não seja primo, n = k1 . k2 com k1 ≥ 2 e k2 ≥ 2, Logo

y = 2 k1 ⋅k 2 − 1 = (2 k1 ) k 2 − 1 = ( 2 k1 − 1) ⋅ (2 k1

( k 2 −1)

+ 2 k1 ( k 2 − 2) + ... + 2 k1 + 1), obser-

ve que 2 k1 − 1 ≥ 3 e 2 k1 ( k 2 −1) + ... + 2 k1 + 1 > 3 e conseqüentemente não teremos y primo, logo n não pode ser escrito como acima; donde n é primo.

14)

Determine o número de soluções de positivos.

Solução Temos 1998x + 1998y = xy . Somando 19982 dos dois lados temos xy – 1998x – 1998y + 19982 = 19982, logo x(y – 1998) – 1998(y – 1998) = 19982, donde (x – 1998) (y – 1998) = 19982.

EUREKA! N° 3, 1998

59

1

x

+

1

y

=

1 1998

com x e y inteiros


Sociedade Brasileira de Matemática

Desta forma o número de soluções é o mesmo que a quantidade de sistemas da forma abaixo que possamos obter:

 x − 1998 = a   y − 1998 = b  2 ab = 1998 com a observação de que os pares (x, y) solução devam ser inteiros e positivos, devemos ter

a + 1998 > 0 a > −1998 ⇒  b + 1998 > 0 b > −1998 logo, só servem a e b positivos, já que se –1998< a < 0 e –1998 < b < 0 implica ab < 19982. O número de soluções é, portanto, o número de divisores positivos de 19982 = 22. 36. 372, que é dado por (2 + 1) (6 + 1) (2 + 1) = 63.

Soluções dos problemas 2, 5 e 6 enviadas por Zoroastro Azambuja Neto. Soluções dos problemas 3 e 12 enviadas por Carlos Alberto da Silva Victor. Solução do problema 14 enviada por Vicente Wilson Moura Gaete e André Luiz Arruda Marques. Continuamos esperando as soluções dos problemas 10, 11, 12, 13 e 15.

EUREKA! N° 3, 1998

60


Sociedade Brasileira de Matemática

PROBLEMAS PROPOSTOS Convidamos o leitor a enviar soluções dos problemas propostos e sugestões de novos problemas para os próximos números.

16)

Seja l a reta {( x, y ) ∈ R 2 y = 0}, C1 o círculo centrado em

i) ii)

1 1 1 1 (0, ) de raio e C 2 o círculo centrado em (1, ) de raio . 2 2 2 2 Seja F o conjunto de círculos em R2 com as seguintes propriedades: {C1, C2} ⊂ F Se C e C’ pertencem a F, são tangentes entre si e tangentes a l ~

então todo círculo C tangente aos dois círculos C e C’ e à reta l pertence a F. iii)

~

Se F é um conjunto de círculos satisfazendo as propriedades i) e ii) ~

então F ⊂ F . Determine o conjunto dos pontos de tangência dos círculos C ∈ F com a reta l.

17)

Dado n ∈ N, uma partição π de n é uma lista ordenada π = (a , a ,...,a ) , r, a , a ,..., a ∈N* com a ≤ a ≤ ...≤ a e a + a + ...+ a = n. 1

2

r

1

2

r

1

2

r

1

2

r

Seja Pn o conjunto das partições de n. Para π ∈ Pn , definimos A(π) como o número de termos iguais a 1 em π ( ou seja , A(π ) =

# {i ∈ {1,2,..., r} ai = 1}), e B(π) como o número de termos distintos na partição π (ou seja, B (π) = # {a1, a2, …, ar}). A(π ) = B (π ) para todo n ∈ N. Prove que

π ∈Pn

π ∈Pn

18)

Seja α a maior raiz real da equação x3 – 3x2 + 1 = 0. Prove que [α2004] é divisível por 17. Obs: [y] é o único inteiro tal que [y] ≤ y < [y] + 1.

19)

a) Determine o número máximo de regiões em que n retas podem dividir o plano. b) Determine o número máximo de regiões em que n planos podem dividir o espaço.

EUREKA! N° 3, 1998

61


Sociedade Brasileira de Matemática

COORDENADORES REGIONAIS Alberto Hassen Raad Antônio C. Rodrigues Monteiro Amarísio da Silva Araújo Angela Camargo Antônio C. do Patrocínio Ariosto de Oliveira Lima Benedito T. Vasconcelos Freire Carlos A. Bandeira Braga Claudio Arconcher Egnilson Miranda de Moura Élio Mega Florêncio F. Guimarães F. Francisco Dutenhefner Gisele de A. Prateado G. Ivanilde H. Fernandes Saad João B. de Melo Neto João F. Melo Libonati Jorge Ferreira José Carlos Pinto Leivas José Luis Rosas Pinho José Paulo Carneiro José Vieira Alves Leonardo Matteo D'orio Licio Hernandes Bezerra Luzinalva M. de Amorim Marco Polo Marcondes Cavalcante França Mario Jorge Dias Carneiro Pablo Rodrigo Ganassim Paulo H. Cruz Neiva de L. Jr. Reinaldo Gen Ichiro Arakaki Ricardo Amorim Sergio Claudio Ramos Tadeu Ferreira Gomes Valdenberg Araújo da Silva Wagner Pereira Lopes

(UFJF) Juiz de Fora-MG (UFPE) Recife-PE (UFV) Viçosa-MG (Centro de Educação de Adultos CEA) Blumenau-SC (IMECC/UNICAMP) Campinas-SP (UFPI) Parnaíba-PI (UFRN) Natal-RN (UFPB) João Pessoa-PB (Col. Leonardo da Vinci) Jundiaí-SP (Col. Agrícola do Bom Jesus) Bom Jesus-PI (Col. ETAPA) São Paulo-SP (UFES) Vitória-ES (UFMG ) BH-MG (UFGO) Goiânia-GO (U. Católica Dom Bosco) Campo Grande-MS (UFPI) Teresina-PI (Grupo Educ. IDEAL) Belém-PA (UEM) Maringá-PR (URG) Rio Grande-RS (UFSC) Florianópolis-SC (USU) Rio de Janeiro-RJ (UFPB) Campina Grande-PB (Parque de Material Belém-PA Aeronáutico de Belém) (UFSC) Florianópolis-SC (UFBA) L. de Freitas-BA (Colégio Singular) Santo André-SP (UF Ceará) Fortaleza-CE (UFMG) BH-MG (L. Albert Einstein) Piracicaba-SP (Esc. Tec.Everardo Passos) S. J.Campos-SP (INPE) S.J.Campos-SP (Centro Educ. Logos) Nova Iguaçu-RJ (IM-UFRGS) Porto Alegre-RS (U. do Estado da Bahia) Juazeiro-BA (U. Federal de Sergipe) São Cristovão-SE (Esc. Tec. Fed. de Goiás) Jataí-GO

EUREKA! N° 3, 1998

62


Issuu converts static files into: digital portfolios, online yearbooks, online catalogs, digital photo albums and more. Sign up and create your flipbook.